Combined Deck: 24.1, 23.2, 23.1, 22.2, 22.1, 21.1, 21.2, 20.1, 20.2 Flashcards
MCQs with remembered answeres from the last 5yrs of released stems for the ANZCA part 2 exam
2hyp2.1A high mixed venous oxygen saturation (SvO2) is most likely to be associated with
a. COPD
b. PE / Tamponade
c. Acute MI
d. Severe liver failure
e. Sepsis
d. Severe liver failure
but could also be
e. Sepsis
LIFTL:
INTERPRETATION
High SvO2
- increased O2 delivery (increased FiO2, hyperoxia, hyperbaric oxygen)
- decreased O2 demand (hypothermia, anaesthesia, neuromuscular blockade)
- high flow states: sepsis, hyperthyroidism, severe liver disease
Low SvO2
- decreased O2 delivery:
1. decreased Hb (anaemia, haemorrhage, dilution)
2. decreased SaO2 (hypoxaemia)
3. decreased Q (any form of shock, arrhythmia)
- increased O2 demand (hyperthermia, shivering, pain, seizures)
- Causes of High SvO2 despite evidence of End-organ Hypoxia:
1. microvascular shunting (e.g. sepsis)
2. histotoxic hypoxia (e.g. cyanide poisoning)
3. abnormalities in distribution of blood flow
Anesthesia Monitoring Of Mixed Venous Saturation:
https://www.ncbi.nlm.nih.gov/books/NBK539835/
In sepsis, ScvO2 less than 70% or SvO2 lower than 65% correlate with poor prognosis.[2] In application, certain studies have shown that maintaining a goal ScvO2 greater than 70% leads to reduced mortality.[11] Therefore, ScvO2 is used to guide treatment algorithms in the Surviving Sepsis Campaign (SSC).
Studies have shown that normal to higher levels of mixed venous oxygen saturation in patients with clinically worsening sepsis do not rule out tissue hypoxia due to the inability to utilize O2.[11][7] Therefore, several studies support the conclusion that abnormally low or high ScvO2 correlates with higher mortality in patients with septic shock.
20.1 A patient has foam sclerotherapy to treat a number of varicose veins. Following the procedure she stands, immediately loses consciousness and develops a unilateral limb weakness. The most likely mechanism is
a. Anaphylaxis
b. Intracranial bleed
c. Paradoxical gas embolus
d. Thromboembolic stroke
c. Paradoxical gas embolus
Although liquid-injection sclerotherapy is the criterion standard, foam sclerotherapy is becoming a popular alternative because of its efficacy and success rate.1 A potential complication of foam sclerotherapy is the formation of gas microemboli in the brain, which can lead to neurologic deficits.
https://www.degruyter.com/document/doi/10.7556/jaoa.2016.063/html?lang=en
20.2 If group A Rh-ve fresh frozen plasma is not available for use in an A Rh-ve patient, of the following your next best choice should be
a. A+
b. B-
c. AB+
d. O+
e. O-
a. A+
Group A Plasma component preference
1st choice: A
2nd Choice: AB
3rd Choice: B
[a] If the patient is a female of childbearing potential, O RhD negative red cells should be used until the patient’s blood group is established.
[b] Group A platelets with the A2 subgroup don’t express significant amounts of A antigen and are therefore preferable to other group A platelets when transfusing group O and B recipients.
[c] Apheresis platelets that have a low titre anti-A/B or pooled platelets pose a lower risk of haemolysis when transfusing ABO incompatible components.
[d] Plasma components that have low titre anti-A/B pose a lower risk of haemolysis when transfusing ABO incompatible components.
[e] Group A plasma may be used as per local institutional policies.
If no A, use AB Rh + cryo (Ie; no anti A or anti B)
Cryo incompatible can be given, but large volumes are high risk for DIC
https://litfl.com/cryoprecipitate/
20.1 In the treatment of diabetic ketoacidosis, the most important initial therapeutic intervention is to
a) Electrolyte correction
b) Insulin
c) IV hydration
d) Bicarbonate
IV hydration
Fluid first (hartmanns or saline w k+) then insulin
BJA Developments in the management of diabetic ketoacidosis 2015
Diabetic ketoacidosis (DKA) is a medical emergency and bedside capillary ketone testing allows timely diagnosis and identification of successful treatment.
> 0.9% saline with premixed potassium chloride should be the main resuscitation fluid on the general wards and in theatre; this is because it complies with National Patient Safety Agency recommendations on the administration of potassium chloride.
> Weight-based fixed rate i.v. insulin infusion (FRIII) is now recommended rather than a variable rate i.v. insulin infusion (VRIII).
> The blood glucose must be kept above 14 mmol litre−1 with the FRIII.
> Precipitating factor(s) needs to be identified and treated. Surgery and also critical care may be indicated to manage the patient presenting with DKA.
22.2 The nerve labelled by the arrow marked P in the diagram is the
- Ulnar Nerve
- Axillary Nerve
- Median Nerve
- Medial Cutaneous nerve of the forearm
- Long Thoracic Nerve
- Dorsal Scapular Nerve
- Radial Nerve
- Suprascapular nerve
- Musculocutaneous Nerve
- Long Thoracic Nerve
22.2 The most likely diagnosis for the following electrocardiograph is
(comment that this was like a 2015A repeat - ECG below is from that paper + 2022 recalled options)
a. AF with BBB
b. sinus tachy with BBB
c. ventricular tachycardia
d. torsades
b. sinus tachy with BBB
The most correct answer would be Trifasicular block:
RBBB with LAD (RBBB with left anterior hemiblock) and 1st degree heart block
Barash 8E 2017:
The term bifascicular block often refers to block in the right bundle and one of the two major fascicles of the left bundle. RBBB with left anterior hemiblock is present when the ECG shows an RBBB with a left axis deviation (usually greater than −60 degrees) in the absence of an inferior myocardial infarction. Complete RBBB with right axis deviation (greater than 90 degrees) is indicative of RBBB and left posterior hemiblock in the absence of a lateral myocardial infarction or evidence of right-sided heart failure. The term trifascicular block is used to describe first-degree AV block in the presence of bifascicular block.
Is it necessary to insert a temporary pacemaker before general anesthesia for an asymptomatic patient with bifascicular or trifascicular block?
The risk for progression to complete heart block in asymptomatic patients with bifascicular block is low. Further, no clinical characteristics have been identified that accurately predict the risk of development of complete heart block. Therefore, routine PPM implantation in patients with asymptomatic bifascicular block is not recommended. Observations made in the perioperative period have suggested that development of complete heart block during general anesthesia is also rare; therefore, it is generally not recommended that patients undergo temporary pacemaker insertion before general anesthesia. However, it is advisable to have an external pacemaker available in the operating room.
23.1 The initial treatment of a trigeminocardiac reflex during skull base surgery should be
a. Tell surgeons to stop stimulus
b. Atropine
c. LA to site
a) Tell the surgeons to stop stimulus
https://www.ncbi.nlm.nih.gov/pmc/articles/PMC1821135/
https://jamanetwork.com/journals/jamaotolaryngology/fullarticle/1864754
Careful dissection for prevention and early intervention with stimulus removal and anticholinergic use as needed are paramount to ensure good outcomes
N.B
Trigeminocardiac reflex refers to the sudden development of bradycardia or even asystole with arterial hypotension from manipulation of any sensory branches of the trigeminal nerve. Although it has only rarely been associated with morbidity and tends to be self-limited with removal of the stimulus, it is an important phenomenon for head and neck surgeons to recognize and respond to
23.1 You are planning to extubate a patient following airway surgery. The patient has FAILED the cuff-leak test when
a. <110ml leak with cuff deflated
b. >110ml leak with cuff deflated
c. Audible leak with cuff deflated
d. No audible leak with cuff deflated
e. No audible leak with cuff pressure <30cm H2O
a. <110ml leak with cuff deflated
approach is to use 110 mL or 10% of tidal volume as the cut-off
https://litfl.com/cuff-leak-test/
21.1 The lung ultrasound finding most consistent with atelectasis is three or more
A. B lines
B. A lines
C. Comet tails
D. Z lines
E. Lung Pulse
comet tails or B-lines
useful resource: https://academic.oup.com/bjaed/article/16/2/39/2897763
Comet Tail artefact:
- a short path reverberation artefact that weakens with each reverberation, resulting in a vertical echogenic artefact that rapidly fades as it continues in to the ultrasound image.
https://litfl.com/comet-tail-artefact/
Short path reverberation artefact
- The ultrasound appearance of this artefact is a thin vertical bright or echogenic line that passes from the point of origin, to the deepest part of the ultrasound image.
- When appearing deep to the pleural line these are known as B-lines.
- Elsewhere in the body the identical artefact is known as ring down artefact.
- Where these artefacts fade quickly they are called comet tail artefacts
https://litfl.com/short-path-reverberation-artefact/
Radiopedia “B-line distribution corresponds with sub-pleural thickened interlobular septa” - more consistent with homogenous atelectasis
21.1 Infection control management of patients with carbapenemase-producing Enterobacteriaceae (CPE)
infection should include all of the following EXCEPT
a) isolation
b) contact precautions
c) droplet precautions
d) screening at risk patients with rectal swab and urine mcs
c) droplet precautions
https://www.safetyandquality.gov.au/sites/default/files/migrated/Recommendations-for-the-control-of-Carbapenemase-producing-Enterobacteriaceae.pdf
23.1 A patient will open her eyes in response to voice, speak with inappropriate words and
withdraw to a painful stimulus. Her Glasgow Coma Scale score is
a. 6
b. 7
c. 8
d. 9
e. 10
e. 10
20.1
a. Arndt blocker
b. Cohen blocker
c. Microlaryngeal tube
d. Husaker tuber
e. Parker Flex-tip tube
B Cohen Blocker
21.1 A transhiatal oesophagectomy is performed via a
a) laparotomy + right thoracotomy
b) laparotomy + left neck incision
c) laparotomy + left neck incision + Right thoractomy
d) Laparotomy + left thoractomy
midline laparotomy and left cervical incision
https://academic.oup.com/bjaed/article/17/2/68/2907833
Transhiatal oesophagectomy classically involves laparotomy and dissection of the lower oesophagus through an enlarged diaphragmatic hiatus, followed by removal of the oesophagus and re-anastomosis via a left cervical incision, thereby avoiding thoracotomy altogether (Fig. 2e).
- useful in patients with malignancies of the lower third of the oesophagus where thoracotomy is undesirable, such as those who have previously undergone thoracic surgery. - Dissection around the mediastinum is frequently associated with arrhythmias and ventricular compression causing hypotension (although this frequently occurs in transhiatal surgery, it is not uncommonly encountered during the thoracic phase of other approaches).
22.2 The Glasgow Coma Score of a patient whose best responses are: opening eyes to pain, making incomprehensible sounds, and withdrawing from pain is
a) 6
b) 8
c) 9
d) 10
e) 12
B) 8
E=2
V=2
M=4
Total= 8
Following a severe spinal cord injury, return of reflexes is usually seen after
a. <1 day
b. 1-3 days
c. 7 days
d. 1-4 weeks
e. >1 month
Answer: b, 1-3 days
BJA 2013 Initial Management of Acute Spinal Cord Injury
Spinal shock is the loss of reflexes below the level of SCI resulting in the clinical signs of flaccid areflexia and is usually combined with hypotension of neurogenic shock.
There is a gradual return of reflex activity when the reflex arcs below redevelop, often resulting in spasticity, and autonomic hyperreflexia.
This is a complex process and a recent four-phase classification to spinal shock has been postulated:
areflexia (Days 0 – 1),
initial reflex return (Days 1 – 3),
early hyperreflexia (Days 4 – 28), and
late hyperreflexia (1 – 12 months)
23.1 A man has symptomatic carbon monoxide poisoning. His pulse oximetry (SpO2) and arterial blood gas (PaO2) would be expected to show
a. Normal SpO2, Normal PaO2
b. Normal SpO2, reduced PaO2
c. Reduced SpO2, normal PaO2
d. Reduced SpO2, reduced PaO2
a. Normal - Normal
A normal or high oximetry reading should be disregarded because saturation monitors cannot differentiate between carboxyhaemoglobin and oxyhaemoglobin, owing to their similar absorbances.
The blood gas PO2 will also be normal in these cases (despite the presence of tissue hypoxia).
file:///Users/newuser/Downloads/BTS%20Guideline%20for%20oxygen%20use%20in%20adults%20in%20healthcare%20and%20emergency%20settings.pdf
21.1 The most common cause of postoperative visual loss after spinal surgery is
a. Central retinal artery occlusion
b. Central retinal vein occlusion
c. Ischemic optic neuropathy
d. Haemorrhage
e. corneal abrasion
c. Ischemic optic neuropathy
Cardiac: Anterior
Spinal: Posterior
ION
23.1 Therapeutic privilege is defined as
A. Withholding information to obtain consent
B. Getting presents and money for treating someone.
C. Not telling pt info because of their religious or cultural beliefs.
D. Withholding information to the patient if you think it will cause harm
D. Withholding information to the patient if you think it will cause harm
https://www.sciencedirect.com/topics/medicine-and-dentistry/therapeutic-privilege
“Therapeutic privilege,” also known as “therapeutic nondisclosure,” is defined as the withholding of relevant health information from the patient if nondisclosure is believed to be in the best interests of the patient (President’s Commission, 1982; Berger, 2005). The two most common justifications for such nondisclosure are that the disclosure would create incapacitating emotional distress and that disclosure would violate a patient’s personal, cultural, or other social requirements (Crawley et al., 2001; Berger, 2005).
20.2 A Jehovah’s Witness patient attends for a revision total hip replacement and is medically optimized. You consider she is high risk for the procedure but after extensive discussion agree to proceed, including agreeing that you will not give blood under any circumstances. Your decision can be justified on the basis of
a) Paternalism
b) Non maleficence
c) Autonomy
d) Beneficence
a) Autonomy
- Obligation to respect the decision-making capacities of persons.
Non-maleficence: Obligation to avoid causing harm
- If refused to proceed.
Paternalism: A set of attitudes and practices in which the health provider determines that a patient’s wishes or choices should not be honored.
- If transfused patient against their wishes
Beneficence: Obligation to provide benefits and to balance benefits against risks; obligation of physician to act for the benefit of the patient
- Controversial interpretation in this case. Both proceeding and refusing to do case may be acting for the benefit of the patient, depending on how you look at the scenario.
BJA: ‘MORAL balance’ decision-making in critical care
https://www.bjaed.org/article/S2058-5349(18)30145-8/fulltext
22.1 A four-year-old boy is in refractory ventricular fibrillation. The recommended dose of amiodarone is
80mg
Age + 4 x 2-> 4 + 4 x 2 =16kg
5 x 16mg =80mg
21.1 An 84-year-old woman with dementia presents for surgery for a breast lump. She lives in a care facility and is accompanied by the nurse manager from the facility and her son. Neither have a written legal authority to act on her behalf. Regarding consent for her surgery
a) Anaesthetic consent is implied in surgical consent
b) Son can’t consent
c) Legal guardian can’t consent
d) Not required if 2 Doctors are in agreement about the need for surgery
e) nil consent required if would be in patients interest/not against wishes
a) anaesthetic consent implied in surgical consent
or
e) nil consent required if would be in patients interest/ not against wishes
https://journals.sagepub.com/doi/pdf/10.1177/0310057X1003800504
The anaesthetist should give the parent or relative the same kind of information as a patient and recommend the appropriate procedure. The consent of a parent or relative is lawful authority to proceed. If the parent or relative does not wish to take this role, it may be necessary for a court or a guardianship body to appoint someone else to make the decision. This may take some time, and if the procedure is medically necessary and cannot be deferred, then reasonable treatment may be administered without consent (this is a principle of the common law and, in some jurisdictions, is also in legislation: for example, in New South Wales, the Guardianship Act 1987 (NSW) s 37; and in Victoria
22.2 Regarding cardiopulmonary exercise testing before major surgery, oxygen pulse is the
a. Arterial oxygen content at peak HR
b. Arterial oxygen saturation at mean HR?
c. Arterial oxygen saturation at peak HR
d. PaO2 at peak HR
e. Oxygen consumption/min divided by HR
e. Oxygen consumption/min divided by HR
VO2/HR: oxygen consumption divided by HR, known as the ‘oxygen pulse’ (ml beat–1)
https://www.bjaed.org/article/S2058-5349(19)30021-6/fulltext
The objective of CPET is to determine functional capacity in an individual.
Deficiencies in CPET-derived variables—specifically:
1. ventilatory anaerobic threshold (AT)
2. peak O2 consumption (VO2peak)
3. ventilatory efficiency for carbon dioxide (VE/VCO2)
—are associated with poor postoperative outcomes (mortality, morbidity, admission to intensive care, and length of hospital stay) after intra-abdominal surgery.
- Does the oxygen pulse increase with exercise?
The oxygen pulse is the VO2 divided by HR, and represents the product of the stroke volume and the arterial-venous oxygen difference. It can be seen in panel 2 and can be viewed as a surrogate for stroke volume, and as such should increase at the start of exercise before slowly reaching a plateau at its highest predicted value.
23.1 A feature that is atypical of multiple sclerosis is
A. Unilateral visual loss
B. Aphasia
C. Diplopia
D. Lower limb motor
E. Some sensory thing
B. Aphasia
UTD
22.2 An absolute contraindication to transoesophageal echocardiography is
A. Dysphagia
B. GORD
C. Oesophageal stricture
D. oesophageal webbing
E. oesophageal varices
C. Oesophageal stricture
https://www.asecho.org/wp-content/uploads/2014/05/2013_Performing-Comprehensive-TEE.pdf
22.2 A four-year-old boy is in refractory ventricular fibrillation. The recommended dose of amiodarone is
a) 40mg
b) 80mg
c) 120mg
d) 160mg
80mg
Age + 4 x 2-> 4 + 4 x 2 =16kg
5 x 16mg =80mg
16kg x 5mg/kg = 80mg
23.1 According to National Audit Project (NAP) 5, the incidence of awareness during general anaesthesia for lower segment caesarean section should be quoted as
a) 1:700
b) 1:3,000
c) 1:8,000
d) 1:19,000
e) 1:36,000
a) 1:670 (or 1:700)
23.1 According to the Fourth Consensus Guidelines for the Management of Post-operative Nausea and Vomiting (PONV) published in 2020, multimodal PONV prophylaxis should be implemented in adult patients
a. For everyone
b. 1 or more RF
c. 2 or more RF
d. 3 or more RF
e. 4 or more RF
b) 1 or more RF
20.1 In a Blalock–Taussig shunt, blood passes to the pulmonary artery via the
a. Aorta
b. Subclavian artery
c. IVC
d. SVC
e. Left atrium
B
22.2 The prevention of microbial contamination of living tissues or sterile materials is known as
a. disinfection
b. antisepsis
c. decontamination
d. asepsis
e. sterilisation
d. asepsis
Asepsis: the prevention of microbial contamination of living tissues or sterile materials.
Disinfection: the inactivation of non-sporing organisms using either thermal or chemical means.
Sterilisation: complete destruction of all micro-organisms, including spores.
https://www.anzca.edu.au/getattachment/e4e601e6-d344-42ce-9849-7ae9bfa19f15/PG28(A)-Guideline-on-infection-control-in-anaesthesia
22.1 The risk of a perioperative respiratory adverse event in a child is least likely to be increased by
a. Asthma
b. Infection 3 weeks ago
c. History of eczema
d. Passive smoking
History of eczema
APRICOT study
21.1 Predictors of successful awake extubation after volatile anaesthesia in infants include
a. 2mL/kg tidal volume,
b. grimacing
c. coughing
d. RR > 20
b. grimacing
conjugate gaze
facial grimace
eye opening
purposeful movement
tidal volume greater than 5 ml/kg
Source: SPANZA 2019 article
22.2 A patient with an acute subarachnoid haemorrhage arrives in the emergency department. Her Glasgow Coma Scale score is 10 and she has no motor deficit. A CT brain shows diffuse subarachnoid haemorrhage with no localised areas of blood > 1 mm thick, and no intracerebral nor intraventricular blood. Her World Federation of Neurosurgical Societies (WFNS) grade of subarachnoid haemorrhage is
a) 1
b) 2
c) 3
d) 4
e) 5
4
GCS 7-12
20.2 The normal response of serum growth hormone level to an oral glucose load is
A. Initially increases then normalises
B. Initially decreases then normalises
C. Initially increases and stays elevated
D. Initially decreases then stays decreased
E. No response
B. Initially decreases then normalises
Oral glucose tolerance test — The most specific dynamic test for establishing the diagnosis of acromegaly is an OGTT. When performing the test, we measure serum GH before and two hours after glucose administration; the criterion for the diagnosis of acromegaly is a GH concentration greater than 1 ng/mL. In normal subjects, serum GH concentrations fall to 1 ng/mL or less within two hours after ingestion of 75 g glucose. In contrast, the post-glucose values are greater than 2 ng/mL in over 85 percent of patients with acromegaly.
Following oral glucose administration in humans, a transient suppression of plasma GH levels for 2–3 h is observed followed by a delayed rise occurring at 3–5 h post glucose ingestion. This initial suppression seems to be related to a glucose-mediated increase in hypothalamic somatostatin release. Evidence supporting this hypothesis emerges from the findings that in healthy individuals, GH secretion in response to GHRH or GH secretagogue is diminished after an oral glucose load. Furthermore, the inhibitory effect of glucose is reversed with the acetylcholinesterase inhibitor pyridostigmine, a substance thought to suppress somatostatin release from the hypothalamus. These findings support the hypothesis that oral glucose load is associated with a somatostatin release into the hypophyseal portal blood suppressing GH levels. The delayed GH rise would result from a decrease in somatostatinergic tone and hence an increase in GHRH. Subsequently, the available pituitary stores of GH are released leading to a rebound rise in GH.
22.1 A risk factor for postoperative nausea and vomiting in adults is age less than
a. 20
b. 30
c. 40
d. 50
e. 60
50
4th consensus guidelines for management of PONV
An awake patient in the post-anaesthesia care unit complains of breathlessness. The FiO2 through the patient’s rebreather mask is 40%. An arterial blood gas taken at the time shows (ABG shown). The alveolar-arterial gradient (in mmHg) is approximately
Blood gas shows:
PaO2 135
PaCO2 48
SpO2 100%
The A-a gradient is:
A. 5
B. 30
C. 60
D. 90
E. 110
D 90
A-a = PAO2 - PaO2
Alveolar air equation gives PAO2
PAO2 = PiO2 - PaCO2 / R
PAO2 = 0.4 x (760 - 47) - 48 / 0.8
so, as PaO2 given as 135
A-a = 228 - 135 = 93
23.1 Application of a pacemaker magnet to a dual-chamber implanted pacemaker would be expected to convert the operating mode to
a. AOO
b. VOO
c. DOO
d. AAI
c) DOO
The pacing mode will be DOO when the programmed pacing mode is a dual chamber mode or an MVP mode (AAIR<=>DDDR, AAI<=>DDD), VOO when the programmed pacing mode is a single chamber ventricular mode, and AOO when the programmed pacing mode is a single chamber atrial mode.
21.2 You have been asked to provide general anaesthesia for a complex thoracic endovascular aortic aneurysm repair. After the placement of a lumbar drain the recommended safe time before the administration of intravenous heparin is
a) 1 hour
b) 4 hours
c) 6 hours
d) 12 hours
1 hour
ASRA: 1 hour
Although the occurrence of a bloody or difficult neuraxial needle placement may increase risk, there are no data to support mandatory cancellation of a case. Direct communication with the surgeon and a specific risk-benefit decision about proceeding in each case are warranted.
Currently, insufficient data and experience are available to determine if the risk of neuraxial haematoma is increased when combining neuraxial techniques with the full anticoagulation of cardiac surgery. We suggest postoperative monitoring of neurologic function and selection of neuraxial solutions that minimise sensory and motor block to facilitate detection of new/progressive neurodeficits.
NYSORA:
Administration of intravenous heparin intraoperatively should be delayed for at least 1 hour after epidural placement; a delay before administration of subcutaneous heparin is not required. In cases of full heparinization for CPB, additional precautions include delaying surgery for 24 hours in the event of a traumatic tap, tightly controlling the heparin effect and reversal, and removing catheters when normal coagulation is restored.
22.1 The dose of hydrocortisone that has equivalent glucocorticoid effect to 8 mg dexamethasone is
a) 12mg
b) 25mg
c) 50mg
d) 100mg
e) 200mg
200mg Hydrocortisone or 25mg Prednisolone
Conversion
Prednisone 1mg =
Hydrocortisone 4mg =
Dexamethasone 0.15mg =
Triamcinolone 0.8mg =
Methylprednisolone 0.8mg =
Betamethasone 0.15mg =
(https://litfl.com/corticosteroids-overview/)
23.1 Causes of exhaled carbon dioxide detection following oesophageal intubation include
all of the following EXCEPT
a. Massive bronchopleural fistula.
b. Carbonated drink.
c. Vigorous bag valve masking previously.
d. Previous gastric insufflation with CO2 for endoscopy.
e. Tracheoesophageal fistula.
A Massive bronchopleural fistula.
Nick Chrimes 2022 - Journal of Anaesthesia
‘Preventing unrecognised oesophageal intubation: a consensus guideline from the Project for Universal Management of Airways and international airway societies’
Causes of exhaled carbon dioxide detection despite oesophageal intubation
No alveolar ventilation occurring
-Prior ingestion of carbonated beverages or antacids
-Gastric insufflation of CO2 for upper gastrointestinal endoscopy
-Prolonged ventilation with facemask or poorly positioned supraglottic airway before attempting tracheal intubation
-Bystander rescue breaths
Some alveolar ventilation potentially occurring
-Tracheo-oesophageal fistula with tube tip proximal to fistula
-Proximal oesophageal intubation with uncuffed tube in a paediatric patient
19.1, 20.1 Soon after a peribulbar block, the patient’s eye rapidly becomes proptosed and tense, and the visual acuity is markedly decreased. A lateral canthotomy is indicated to:
a) Allow globe to continue to swell
b) Drain blood from behind eyeball
c) Allow the eye to proptose
d) Reduce pressure on the optic nerve
c) Allow the eye to proptose
Orbital Compartment Syndrome
The orbital compartment is a fixed space with limited capacity for expansion. If something like blood fills part of that space the pressure increases and may result in ischaemia of the optic nerve or the retina. A lateral canthotomy is a way of releasing this pressure.
You have up to approximately 2 hours before irreversible visual loss occurs. It may occur in less than 2 hours however, so speed is of the essence.
use local anesthetic but warn the patient that they may feel pain
Perform the canthotomy:
place the scissors across the lateral canthus and incise the canthus full thickness
Perform cantholysis:
Grasp the lateral lower eyelid with toothed forcepsPull the lower eyelid anteriorlyPoint the scissors toward the patient’s nose, place the blades either side of the lateral canthal tendon, and cut.
By cutting the canthal tendon,the counter pressure of the eyelid on the is relieved and the eye is allowed to proptose and pressure is relieved.
23.1 A 50-year-old man presents with a subarachnoid haemorrhage. He undergoes
cerebral angiography and the frontal view is shown below. His cerebral aneurysm is
in the
(exact image on exam)
a. Anterior choroidal
b. Anterior communicating artery
c. MCA
d. PCA
b) anterior communicating artery
https://case.edu/med/neurology/NR/SubarachnoidHemorrhageAComm3.htm
https://www.thieme-connect.com/products/ejournals/pdf/10.1055/s-0039-1681979.pdf
https://case.edu/med/neurology/NR/NRHome.htm (scroll down to subarachnoid imaging area)
21.1 The part of the lung that is typically divided into superior, medial, anterior, lateral and posterior bronchial segments is the
A. Right Upper lobe
B. Right Lower lobe
C. Left Upper lobe
D. Right Middle lobe
E. Lingula
RLL
1.Superior (apical bronchus 6)
-> most common site for foreign body or secretions to collect if patient laying flat in bed
20.2 The most common type of perioperative stroke is
a) Hypoxic
b) Thrombotic
c) Embolic
d) Hypotensive
e) Haemorrhagic
c) Embolic
Blue Book 2017
Perioperative Stroke
Epidemiology
A perioperative stroke is defined as one that occurs either intra-operatively or in the post-operative period within 30 days70. Perioperative strokes are associated with an increased length of stay and a six-fold increased mortality. Any combination of surgery and anaesthesia is associated with an increased risk of stroke irrespective of the type of surgery. This may relate to coagulation changes
The most common type of perioperative stroke is ischaemic stroke of embolic origin (heart or aorta). Hypotension is rarely the cause of perioperative stroke. Haemorrhagic stroke is uncommon which probably reflects the fact that severe hypertension during anaesthesia is a rare event, and anticoagulants have typically been withheld.
The risk of perioperative stroke varies depending on the type of the surgery and patients’ risk factors.
Procedural risk
Urgent surgery is associated with an increased risk of stroke when compared to elective surger.
Cardiac, vascular and brain surgeries are defined as “high-risk” as these have an increased risk of stroke when compared to other types of surgery. Valvular and aortic repair surgeries have a stroke risk as high as 8 to 10 per cent.
Perioperative strokes in non-high-risk surgery are relatively rare and are estimated to have an incidence of about 1/1000 cases80.
Patients’ risk factors
>Age
>history of previous stroke or transient ischaemic attack
>renal failure
>atrial fibrillation
>history of cardiovascular diseases
are identified risk factors for perioperative stroke.
Atrial fibrillation is associated with a two-fold increase in the risk of death and stroke after carotid endarterectomy.
21.1 In the morbidly obese the induction dose of propofol should be calculated based on
a. Lean body weight
b. Total body weight
c. Ideal body weight
d. Ideal body weight + 70%
Lean Body Weight
For infusion: Adjusted body weight
NDMB: Lean Body weight
Sux: Total body weight
Source: SOBA UK
A 50-year-old woman has had a headache for the last month which is relieved by lying flat. She has had no medical procedure to her spine such as epidural, spinal or lumbar puncture. Her brain magnetic resonance imaging (MRI) scan shows diffuse meningeal enhancement and brain sagging. Her neurologist suspects spontaneous intracranial hypotension and asks you to do an epidural blood patch. No spinal imaging has been performed to confirm a cerebrospinal fluid (CSF) leak. You should
A do LP to measure pressure if low do lumbar patch
B do blood patch at lumbar level with no further investigation
C do spine imaging if CSF leak present do blood patch at level
D do spine imaging if CSF leak present do lumbar blood patch
E refuse to do blood patch
B do blood patch at lumbar level with no further investigation
21.2 The image below on the left shows a normal central venous pressure (CVP) trace. The CVP
trace in the image below on the right is most consistent with
a) AF
b) MR
c) AR
d) TR
e) Pericardial constriction
TR
22.2 Despite an interscalene block being performed preoperatively for arthroscopic rotator cuff repair, a patient wakes up with posterior shoulder pain. The most appropriate procedure to consider would be a nerve block of the
a. Supraclavicular nerve
b. Suprascapular nerve
c. Medial pectoral
d. Vagus nerve
b. Suprascapular nerve
Suprascapula nerve (C5,6)
- innervates supra and infraspinatus
- comes off superior trunk of the brachial plexus, and is usually anaesthetised by an interscalene block
- sensory innervation to 70% posterior-superior shoulders and portion of the anterior axilla and the ACJ
Supraclavicular nerve (C3,4)
- provides sensory to the ‘cape’ of the shoulder
- component of the cervical plexus block
- lies outside the brachial plexus
- commonly missed during supraclavicular brachial plexus blocks
Subscapular nerve:
- subscapularis
- medial rotation shoulder
Dorsal scapular nerve:
- branch of the brachial plexus
- supplies rhomboid major muscle, rhomboid minor muscle, and levator scapulae muscle
- causes the scapula to be moved medially towards the vertebral column
- Dorsal scapular nerve syndrome can cause a winged scapula, with pain and limited motion
Thoracodorsal nerve:
- thoracodorsal nerve also branches from the posterior division of the brachial plexus
- this nerve innervates the latissimus dorsi muscle.
https://resources.wfsahq.org/atotw/the-shoulder-block/
21.1 A baby is brought to the emergency department three days after a term home birth. It has not been feeding well and has had few wet nappies. The child is grey in appearance and femoral pulses are difficult to palpate. You note an enlarged liver and marked tachycardia. Pulse oximetry reveals
saturations of 75% despite oxygen being administered. You suspect a duct-dependent circulation. The best initial management is
a) Intubation and controlled ventilation
b) 20ml/kg crystalloid bolus
c) Alprostadil (PGE1)
d) Stop administration of oxygen
c) Alprostadil (PGE1)
From Paediatric BASIC on CHD:
- Resuscitation of an infant or newborn in shock should follow a standard approach regardless of the aetiology.
- Any patient with a duct dependent lesion either for pulmonary blood flow, or systemic output, will require PGE1. The problem is that whether or not a duct dependent lesion is present is unclear in most cases. If CHD has been diagnosed antenatally, PGE1 should be started.
- The cyanosed neonate presenting with severe cyanosis (O2 <75) and/or in extremis should be started on PGE1; the assumption being that the duct has closed and needs to be reopened.
19.2 An 80-year-old woman is admitted to hospital with respiratory failure. Her arterial blood gas on oxygen 4 litres per minute via a Hudson mask is as follows: (ABG shown) Which of the following most accurately describes this blood gas result?
pH 7.2, pO2 91, pCO2 84, BE 16, HCO3- 43, Na 145
a) Metabolic alkalosis, acute resp acidosis + normal AG
b) Metabolic alkalosis resp acidaemia + abnormal AG
c) Mixed acidaemia
d) Respiratory Acidosis with incomplete compensation
e) Compensated Respiratory acidosis
d) Respiratory Acidosis with incomplete compensation
Uncertain of this answer, not enough info to calculate anion gap
pH 7.2 = acidaemia
pCO2 84 = respiratory acidosis
HCO3 43 = metabolic alkalosis as compensation
BE 16 = metabolic alkalosis
Boston rules:
Chronic fully compensated Respiratory acidosis
Expected compensation is 3-4 mmol/L rise for every 10mmHg rise in PCO2.
Expected metabolic compensation therefore is
HCO3 = 24 + 4 x ((84-40)/10)
= 24 + 4x (44/10)
= 24 + 4 x (4.4)
= 24 + 17.6
= 41.6
Metabolic acidosis
PaCO2 should be 1.5 x HCO3 + 8
= 72.5
Rules (from K.Brandis Acid-base rules anaesthesia mcq):
- 1 for 10 (acute resp acidosis), 4 for 10 (chronic resp acidosis)
- 2 for 10 (acute resp alkalosis), 5 for 10 (chronic resp alkalosis)
- 1.5xHCO + 8 = expected pCO2 in a metabolic acidosis
- 0.7xHCO3 + 22 = expected pCO2 in a metabolic alkalosis
https://www.anaesthesiamcq.com/AcidBaseBook/ab9_3.php
21.2 The main advantage of using noradrenaline (norepinephrine) over phenylephrine for the prevention of hypotension as a result of spinal anaesthesia for elective caesarean section is
a) Better APGAR
b) Better foetal acid-base balance
c) Less nausea & vomiting
d) Less maternal bradycardia
less bradycardia
20.1 ECG calibration, 10mm on Y axis is equal to:
a. 0.2 sec
b. 0.4sec
c. 1sec
d. 0.1mV
e. 1mV
b) 1mV
21.2 Stellate ganglion block is NOT contraindicated in patients with
a) Contralateral phrenic nerve palsy
b) Glaucoma
c) Recent MI
d) Arrhythmia
d) Arrhythmia
- caution if conduction disease however
Contraindications are current coagulopathy (or anticoagulated), recent myocardial infarction, pathologic bradycardia, and glaucoma.
Source Radiopaedia
Contralateral stellate ganglion/phrenic nerve block/neuropathy
22.1 A 68-year-old woman presents with a loud systolic murmur in the anaesthesia room before total
hip joint arthroplasty. A transthoracic echocardiogram is performed (image provided) and shows
a. AS
b. LVOT
c. MR
MR
22.1 The most reliable clinical indicator of opioid-induced ventilatory impairment (OIVI) is decreased
a. Sedation
b. Respiratory rate
SS /GCS
Repeat APMSE
21.1, 20.1 The coagulopathy that can result from intrahepatic cholestasis of pregnancy is due to
a. 2/7/9/10
b. All clotting factors made by the liver
c. Thrombocytopenia
d. Platelet dysfunction
e. Fibrinolysis
a. 2/7/9/10
Hypovitaminosis of Vitamin K
(Bile required for absorption)
Source: BMC Article
https://bmcpregnancychildbirth.biomedcentral.com/articles/10.1186/s12884-022-04875-w
21.2 A patient presents for endovascular clot retrieval after experiencing a right hemisensory loss and right homonymous hemianopia. The vessel most likely occluded is the left
a) ACA
b) MCA
c) PCA
d) AICA
e) PICA
Left PCA
22.1 A straight laryngoscope blade is likely to be more useful than the Macintosh blade when
performing direct laryngoscopy in patients with all of the following EXCEPT
A. Prominent maxillary Incisors
B. Undergoing manual inline stabilisation
C. Large Tongue
D. Floppy Epiglottis
E. Retrognathia
Not actually remembered answers
B undergoing manual inline stabilisation (posterior column problem, all others are anterior column issues)
https://www.anzca.edu.au/getattachment/9ef4cd97-2f02-47fe-a63a-9f74fa7c68ac/PS56BP-Guideline-on-equipment-to-manage-difficult-airways-Background-Paper
Straight laryngoscope blades:
These may be considered for patients with anterior column problems including:
- prominent maxillary incisors
- retrognathia
- large tongue and
- large floppy epiglottis
The Miller straight blade with its low profile produces a higher pressure on the submandibular tissues with the same force (pressure = force/area), and can be used to lift the epiglottis directly 75 to facilitate intubation.
There is evidence to support better success rates with straight blades as a rescue device when the Macintosh blade has failed. However, comparative studies of straight blades and videolaryngoscopy are lacking. As the paraglossal technique
for straight laryngoscope blades is different from Macintosh blades, training and ongoing volume of practice is recommended for optimal use. It should be
recognised that while straight blades provide better laryngoscopic views, the incidence of difficult intubation due to the narrower field of vision is increased .
McCoy (CLM) laryngoscope blades
When “McCoy” laryngoscope blades are in their flexed position, they apply pressure at the base of the tongue lifting the epiglottis anteriorly and are therefore, suitable for posterior column problems (e.g. manual inline stabilisation of head and neck) where the mandible and submandibular tissues are normal.
However, the effect of this levering action of McCoy blades has not been shown to consistently improve laryngeal view. When compared to Glidescope TM videolaryngoscopes, McCoy laryngoscope blades resulted in longer tracheal intubation times in bariatric patients.
Recommendations concerning Straight and McCoy laryngoscope blades:
Page 15 PG56(A)BP Difficult airway equipment BP 2021
Advanced equipment, such as videolaryngoscopy and the common availability of flexible bronchoscopes and intubation guides/bougies, may prove to be better alternatives to difficult airway management. While evidence is currently lacking, it is recommended that Straight and McCoy laryngoscope blades are not required in DATs unless operators have been trained in their use and have ongoing volume of practice (Weak recommendation for, level of evidence
moderate quality)
21.1, 23.1 In patients without other co-morbidities, bariatric weight loss surgery is indicated when the body mass index (kg/m2) is greater than
a. 35
Major updates (2022) to 1991 National Institutes of Health guidelines for bariatric surgery
Metabolic and bariatric surgery (MBS) is recommended for individuals with a body mass index (BMI) 35 kg/m2 , regardless of presence, absence, or severity of co-morbidities.
MBS should be considered for individuals with metabolic disease and BMI of 30-34.9 kg/m2
BMI thresholds should be adjusted in the Asian population such that a BMI 25 kg/m2 suggests clinical obesity, and individuals with BMI 27.5 kg/m2 should be offered MBS.
Long-term results of MBS consistently demonstrate safety and efficacy.
Appropriately selected children and adolescents should be considered for MBS.
https://www.soard.org/article/S1550-7289(22)00641-4/fulltext#:~:text=The%201991%20NIH%20Consensus%20Statement,surgery%20that%20is%20applied%20universally
Contraindications:
- Inflammatory disease of GI tract (ulcers, oesophagitis, Crohn’s)
- Upper GI bleeding
- Portal Htn
- Liver Cirrhosis
- Chronic Pancreatitis
- Laparascopic surgery may be technically difficult in patients weighing >180kg and may be considered a relative contraindication
21.2 Of the following, the lifestyle modification that is least effective in reducing essential
hypertension is
a) Stopping caffeine
b) Low salt diet
c) High potassium diet
d) Exercise
e) Alcohol cessation
a) Stopping caffeine
Eat a well-balanced diet that’s low in salt
Limit alcohol
Enjoy regular physical activity
Manage stress
Maintain a healthy weight
Quit smoking
Foods that are rich in potassium are important in managing high blood pressure (HBP or hypertension) because potassium lessens the effects of sodium. The more potassium you eat, the more sodium you lose through urine. Potassium also helps to ease tension in your blood vessel walls, which helps further lower blood pressure.
Source AHA
23.1 Diagnostic criteria for adult systemic inflammatory response syndrome include all of
the following EXCEPT
a. Leukopenia
b. Hypothermia
c. Tachycardia
d. Tachypnoea
e. Hypotension
e. Hypotension
https://www.safetyandquality.gov.au/sites/default/files/2022-06/sepsis_clinical_care_standard_2022.pdf
21.2, 22.2, 23.2 A 25-year-old woman is administered two doses of aprepitant for postoperative nausea and vomiting after a sleeve gastrectomy. She normally takes the oral contraceptive pill. You should advise her to use alternative contraception for the next
a) 3 days
b) 7 days
c) 14 days
d) 28 days
28 days
Aprepitant PI:
“Alternative or “back-up” measures of contraception should be used during treatment with this medicine and for one month following the last dose of this medicine.”
Pharmacokinetics:
- aprepitant is a CYP3A4 inhibitor
- caution is also advised with warfarin and phenytoin use
21.1 The modified Aldrete scoring system uses all of the following EXCEPT
a) BP
b) Pain score
c) Resp rate
d) sedation level
pain score
21.1 The management of a patient who has experienced a cardiac arrest within 10 days of cardiac surgery
should NOT routinely include
a. Atropine 3mg
b. adrenaline 1mg boluses
c. 3 stacked shocks
d. amiodarone 300
e. 1L fluid
b. adrenaline 1mg boluses
The risk of administering adrenaline in conventional doses is with profound hypertension, bleeding, or tearing of vessel anastomoses on return of spontaneous circulation (ROSC), which can precipitate catastrophic harm or further cardiac arrest.
Adrenaline remains a useful drug in peri-arrest situations in smaller doses.
bonus neuro radiology questions
A 23 year-old man developed the abrupt onset of an explosive headache followed by nausea and vomiting. On exam, he had a right third nerve palsy.
He undergoes cerebral angiography.
His cerebral aneurysm is in the
a. Anterior Cerebral Artery
b. Anterior communicating artery
c. MCA
d. PCA
e. Basilar artery
d. PCA
https://case.edu/med/neurology/NR/SubarachnoidHemorrhag3%20Pcom3.htm
22.2 The sensor on a NIM (Nerve Integrity Monitor) endotracheal tube used for thyroid surgery directly records
a. Electromyography of internal laryngeal muscles
b. Recurrent laryngeal nerve action potential
c. Movement of the vocal cords on the endotracheal tube
d. Pressure of the vocal cords on the endotracheal tube
e. Recurrent laryngeal nerve action potential
a. Electromyography of internal laryngeal muscles
23.1 A patient is undergoing a posterior spinal fusion with somatosensory evokedpotential (SSEP) monitoring. Ischaemia is suggested by
a. Increased amplitude, increased latency
b. Increased amplitude, decreased latency
c. Decreased amplitude, increased latency
d. Decreased amplitude, decreased latency
c. Decreased amplitude, increased latency
22.1 The most common cause of maternal mortality in women with preeclampsia is
a. Renal failure
b. Hepatic failure
c. Intracranial haemorrhage
Intracranial haemorrhage
AHA
https://www.ahajournals.org/doi/epub/10.1161/HYPERTENSIONAHA.118.11513
22.1 Red man syndrome as a consequence of vancomycin administration is caused by
a. Type II hypersensitivity reaction
b. IgE sensitivity
c. Vasodilation of vessels
d. Mast cell degranulation
Mast cell degranulation - anaphylactoid reaction
20.1 You want to position a internal jugular CVL with a CXR at the caval-atrial junction. Where is this?
a) 2 vertebral bodies superior to carina
b) 1 vertebral body superior to carina
c) At the carina
d) 1 vertebral body inferior to carina
e) 2 vertebral bodies inferior to carina
e) 2 vertebral bodies inferior to carina
21.1 A neonate born by emergency caesarean section is limp, pale, has a weak grimace and weak cry, and a heart rate of 60 beats per minute. The Apgar Score is
A. 3
B. 4
C. 5
D. 6
E. 7
3
20.2 During spinal surgery, the anaesthetic agent that is least likely to decrease motor evoked potentials is
A. Non-depolarising muscle relaxants
B. Nitrous oxide
C. Opioids
D. Propofol
E. Volatiles
C. Opioids
A. Non-depolarising muscle relaxants - false - NMBDs abolish MEPs
B. Nitrous oxide - false - N2O can completely abolish MEPs
D. Propofol - false - PPF has less of an effect than volatiles, but still affects MEPs
E. Volatiles - false - volatiles are the most likely
NMBDs > volatiles > N2O > PPF > opioids
https://www.uptodate.com/contents/anesthesia-for-elective-spine-surgery-in-adults
While neurologic injury can cause changes in recorded potentials, other factors can interfere with interpretation. Confounding factors that can occur during surgery include inhalational anesthetics, hypothermia, hypotension, hypoxia, anemia, and preexisting neurologic lesions. Inhaled anesthetics such as isoflurane, sevoflurane, and nitrous oxide can reduce the amplitude and prolong the latency of SSEP and can completely abolish MEP. Neuromuscular blocking agents (NMBAs) also abolish motor evoked potentials and cannot be used when monitoring. Intravenous anesthetics such as propofol, barbiturates, and opioids have less of an effect on monitoring, though very deep anesthesia, even with propofol, can affect waveforms.
https://www.uptodate.com/contents/neuromonitoring-in-surgery-and-anesthesia
Evoked potentials — Evoked potential monitoring is used to assess the integrity of the tested neural pathway. Somatosensory, visual, and brainstem auditory evoked potentials monitor neurologic structures between peripheral sites where specific stimulations are applied, and responses are recorded from central locations. Motor evoked potentials monitor such structures by stimulating the motor cortex and recording from the epidural space (D-wave) or, more commonly, from distal muscles. Changes in evoked responses can result from technical, positional, pharmacologic, physiologic, or surgical causes.
For spine surgery, both MEPs and SSEPs are used to monitor spinal cord function to increase sensitivity. Motor and sensory tracts are anatomically distinct and have different vascular supply in areas of the cortex, brainstem, and spinal cord.
Motor evoked potentials (MEPs) – MEP responses are affected by even very low concentrations of volatile anesthetic agents. In general, total intravenous anesthesia (TIVA) facilitates MEP monitoring. However, inhalation agents at 0.5 MAC or less can be used in many patients, especially during intracranial surgery
Opioids – IV opioids cause small, dose-dependent depression of SSEP and MEP responses, though even at very high doses of opioids, evoked potentials can be recorded [76-78]. Infusions of remifentanil, fentanyl, or sufentanil are commonly used as part of TIVA during neuromonitoring. Opioids tend to produce high-amplitude slow waves in the EEG.
Balanced anesthetic approach — When SSEPs and MEPs are monitored, a balanced anesthetic using both a low-dose inhalation anesthetic (up to 0.5-MAC isoflurane, sevoflurane, or desflurane) and low- to medium-dose propofol (eg, propofol, 40 to 75 mcg/kg/min IV) with a relatively high-dose opioid (eg, remifentanil 0.1 to 0.4 mcg/kg/min) offers several advantages:
●Movement with motor stimulation is reduced, which is particularly important during intracranial aneurysm surgery.
●The addition of a 0.3 to 0.5 MAC inhalation agent may reduce the chance of awareness under anesthesia.
●Compared with TIVA, the addition of a 0.5 MAC inhalation agent allows reduction of the dose of propofol infusion, facilitating more rapid wakeup and earlier neurologic examination.
●Compared with TIVA, the chance of accidental interruption of the anesthetic for mechanical reasons (ie, kinked or infiltrated IV catheter or tubing such that IV agents no longer infuse) is reduced.
21.2 A patient has numbness and weakness in her hand postoperatively. You are trying to distinguish between an ulnar nerve lesion and a C8-T1 radiculopathy. You can diagnose a C8-T1 radiculopathy if she has weakness
A. Parasthesia in little finger
B. Parasthesia in the distribution of the interscalene nerve
C. Weakness in adductor digiti minimi
D. Weakness in abductor pollicis brevis
E. Weakness in lateral interosseus
D. Weakness in abductor pollicis brevis
Muscles weak in C8-T1 radiculopathy but intact in ulnar neuropathy
- flexor pollicis brevis
- abductor pollicis brevis
- opponens pollicis
- lateral lumbricals
AbOF the Law may be useful—the abductor (Ab) and flexor (F) pollicis brevis, opponens pollicis (O), and lateral lumbricals (Law) are “above the law” that intrinsic hand muscles are ulnar-innervated
22.1 A 57-year-old female smoker presents for a laparotomy with the following pulmonary function tests
(normal FEV1 FVC, low RV and FRC only, normal DLCO)
They are consistent with a diagnosis of
a. Obesity
b. PE
c. Pulmonary fibrosis
d. COPD
a. Obesity
Obesity and pulmonary function testing
https://www.jacionline.org/article/S0091-6749(05)00164-8/fulltext
- Full pulmonary function tests are often necessary to better characterize the spirometric abnormalities seen in the obese patient
- The most sensitive indicator of obesity is a low expiratory reserve volume (ERV) and functional residual capacity
- Restriction is seen in more severe obesity, with reductions in TLC and FVC.
- However, residual volume is often preserved because of the relative high closing volume in relation to ERV.
20.1 What is the arrow pointing to?
a. Ilioinguinal
b. Psoas
c. Iliacus
d. Lateral cutaneous nerve of thigh
e. Obturator
b. Psoas
20.2 You are seeing a 48 year-old woman in your pre-operative clinic for assessment for laparoscopic sleeve gastrectomy. Her co-morbidities include obesity (BMI is 65 kg/m2), hypertension, type 2 diabetes mellitus and polycystic ovary syndrome. Her neck circumference is 38 cm. Her husband states that she snores loudly, but he has never observed her having any apnoeic episodes and she reports no excessive tiredness during the day. Her score using the STOP-BANG questionnaire is
a. 3
b. 4
c. 5
d. 6
e. 7
a. 3 (snoring, BMI, Htn)
Snoring loudly
Tiredness during day time
Observed Apnoea
Pressure: Htn
BMI > 35
Age > 50
Neck circumference >40cm (43cms male)
Gender: Male
23.1 The following is a chest X-ray from a patient complaining of dyspnoea after thoracic surgery. The diagnosis is
(not the image from the exam)
A. Dextracardia
B. Cardiac hernation
C. LLL collapse
D. Tension Pneumohorax
B. Cardiac hernation
https://www.ahajournals.org/doi/10.1161/CIRCULATIONAHA.109.896829
21.1 During trauma resuscitation in adults, contraindications to blind nasogastric tube insertion include all of
the following EXCEPT
a) High C-spine injury
b) Recent nasal surgery
c) Oesophageal fracture
d) Base of skull fracture
e) oesophageal varices
High C-Spine injury
22.2 All of the following conditions are associated with acromegaly EXCEPT
a) cardiac arrhythmias
b) cardiac failure
c) OSA
d) aortic dilation
d) aortic dilation
Osteoarthritis
nerve compression syndrome due to bony overgrowth, and carpal tunnel syndrome
Hypertension
Diabetes mellitus
Cardiomyopathy/HF
Colorectal cancer
Sleep Apnea
Thyroid nodules and thyroid cancer
Hypogonadism
Compression of the optic chiasm
Source: BJA
21.1 A woman is having a potentially curative primary breast cancer resection. Compared with a sevoflurane and opioid technique, using a regional anaesthesia-analgesia technique with paravertebral block and a propofol infusion will result in
a. Decreased cancer recurrence
b. Decreased chronic pain and recurrence
c. Decreased incision pain at 6 months
d. Decreased CPSP pain at 6 months
e. Decreased CPSP pain at 12 months
Fuck this question
e. Decreased CPSP pain at 12 months
or it could be updated with an option that says makes no difference
most likely they will just remove the question and this is a big waste of time
https://pubs.asahq.org/anesthesiology/article/135/6/1091/117748/Preoperative-Paravertebral-Block-and-Chronic-Pain
—>This says it makes no difference in 2021
https://www.cochranelibrary.com/cdsr/doi/10.1002/14651858.CD007105.pub4/full
—-> this says weak evidence but it helps prevent persistent post surgical pain at 3-12months in 2018
—-> ANZCA pain book references this article
ANZCA pain book
https://www.bjaed.org/article/S2058-5349(18)30101-X/fulltext
A recent review showed that, whilst there was little effect on intra- and postoperative opioid consumption and PONV, patients receiving either both single-shot injections or placement of paravertebral catheters had less acute pain in the first 72 h after surgery.
There is also a suggestion that the use of TPVB for acute postsurgical pain may protect against the development of chronic postsurgical pain after breast surgery at 6 months.
For breast cancer surgery any form of regional anaesthesia (18 RCTs, n=1,297) reduces CPSP 3 to 12 months after surgery compared with systemic analgesia (OR 0.43; 95%CI 0.28 to 0.68) (NNT 7); specifically paravertebral block (PVB) (6 RCTs, n=419) is effective (OR 0.61; 95%CI 0.39 to 0.97) (NNT 11).
In our study population, regional anaesthesia-analgesia (paravertebral block and propofol) did not reduce breast cancer recurrence after potentially curative surgery compared with volatile anaesthesia (sevoflurane) and opioids. The frequency and severity of persistent incisional breast pain was unaffected by anaesthetic technique. Clinicians can use regional or general anaesthesia with respect to breast cancer recurrence and persistent incisional pain.
https://www.thelancet.com/pdfs/journals/lancet/PIIS0140-6736(19)32313-X.
22.1 The recommended filter grade of a needle to be effective in excluding microorganisms is
0.20 um
20.1, 22.2 Your patient underwent a stellate ganglion block 2 hours ago. Prior to discharge you are asked to review the patient in recovery because of a droopy upper eyelid. The patient would also be expected to have ipsilateral
a) Pupillary constriction and reaction to light
b) Pupillary constriction and no response to light
c) Pupillary dilation and response to light
d) Pupillary dilation and no response to light
a) Pupillary constriction and reaction to light
Stellate ganglion block causes ipsilateral Horner’s Syndrome:
Ptosis (eyelid droop)
Miosis (constricted pupils)
Anhydrosis (loss of sweating)
Enophthalmos (sinking of eyeball into the bony cavity that protects the eye)
*Pupillary constriction in response to light is controlled by the Edinger-Westphal nucleus of CN3, which will remain intact.
20.2 You are anaesthetising a 35 year old woman undergoing a laparoscopic appendicectomy. She uses a levonorgestrel-secreting intrauterine device (MirenaTM) for contraception and you have used sugammadex for reversal of neuromuscular blockade at the end of the procedure. Your post-operative
advice to her regarding contraception should state that
a. Barrier protection for a week
b. Barrier protection until the next period.
c. The mirena is sufficient
d. OCP for a week
e. OCP until next period
a. Barrier protection for a week
In the case of non-oral hormonal contraceptives, the patient must use an additional non hormonal contraceptive method for the next 7 days
23.1 A healthy woman is admitted to the obstetric unit with threatened preterm labour at 29 weeks gestation. Her blood pressure is 140/80 mmHg. A magnesium sulfate infusion is indicated for the purpose of
A. Maternal seizure prevention
B. Fetal lung development
C. Foetal neuroprotection
C. Foetal neuroprotection
https://www.kemh.health.wa.gov.au/~/media/HSPs/NMHS/Hospitals/WNHS/Documents/Clinical-guidelines/Obs-Gyn-Guidelines/Preterm-Labour-Magnesium-Sulphate-for-Neuroprotection-of-the-Fetus.pdf?thn=0#:~:text=MgSO4%20is%20only%20given%20to,4%20hours%20prior%20to%20delivery.
22.2 For a 70-year-old patient on rivaroxaban with normal renal function a major guideline recommends proceeding with hip fracture surgery after two half-lives of the drug. This equates to
a. 12 hours
b. 24 hours
c. 48 hours
d. 72 hours
e.
b. 24 hours
ASA guidelines
-If creatinine clearance >/=30 ml.min-1 (Cockcroft-Gault), proceed with surgery after two half lives (24 h) since the last dose, under general anaesthesia (or spinal anaesthesia if indicated)
- If creatinine clearance < 30 ml.min-1, proceed with surgery after four half lives (48 h) since the last dose, under general anaesthesia (or spinal anaesthesia if indicated)
21.1 A normal 75 kg term parturient may be expected to have a total blood volume of
a. 5250
b. 6000
c. 6750
d. 7500
d. 7500
100ml/kg blood volume in term parturient
7.5L (Average increase around 48%)
BJAed
22.1 A normal sized six-year-old girl has a haemoglobin of 70 g/L following surgery. The volume of packed red blood cells that you would plan to infuse to raise her haemoglobin to 80 g/L is
a. 80ml
b. 100ml
c. 120ml
d. 180ml
e. 200ml
b. 100ml
Paediatric weight estimation:
Luscombe: Weight (kg) = (age x 3) + 7
RCH: Weight (kg) = (age + 4) x 2
Formula for calculating transfusion volume (mL)
Children <20 kg:
PRBC (mL) = wt (kg) x Hb (g/L) rise (desired Hb – actual Hb) x 0.5 (transfusion factor)
Children >20 kg: 1 unit PRBC
Example:
6 + 4 x 2 = 20kg
20kg x 10g/l x 0.5 = 100ml
21.1 A patient has numbness and weakness in her hand postoperatively. You are trying to distinguish between an ulnar nerve lesion and a C8-T1 radiculopathy.
You can diagnose a C8-T1 radiculopathy if she has weakness
a) Thumb adduction
b) Thumb abduction
c) Fingers adduction
d) Fingers Abduction
e) Little finger flexion
b) Thumb abduction
(flexor pollicis brevis)
D. Paraesthesia/sensory loss over medial forearm
(medial antebrachial cutaneous)
Severing Ulnar nerve alone results in numbness of the 4th (ring) and 5th (little) fingers alone
C8 and T1 supply the medial antebrachial cutaneous nerve
Muscles weak in C8-T1 radiculopathy but intact in ulnar neuropathy
- flexor pollicis brevis
- abductor pollicis brevis
- opponens pollicis
- lateral lumbricals
AbOF the Law may be useful—the abductor (Ab) and flexor (F) pollicis brevis, opponens pollicis (O), and lateral lumbricals (Law) are “AbOF the Law” that intrinsic hand muscles are ulnar-innervated
21.2 A respiratory effect of high flow nasal oxygen therapy is
a) Increased deadspace
b) Reduced minute volume
c) Increased work of breathing
d) Reduced RR
d) Reduced RR
BJA: HFNP oxygen therapy
https://www.bjanaesthesia.org/article/S0007-0912(17)53999-9/fulltext
- reduced RR
- increased minute volume
- reduced WOB, reduced Vd, reduced AWR
- provides CPAP 3-7 cmH20 (mouth closed)
23.1 According to Australian and New Zealand Anaesthetic Allergy Group (ANZAAG) anaphylaxis guidelines for adults, cardiopulmonary resuscitation should commence at a systolic blood pressure of less than
a. 70
b. 60
c. 50
d. 40
c) 50mmHg
The initial management for a seizure during an awake craniotomy is
a. Cold saline irrigation
b. Midazolam
c. Propofol
a) Cold Saline Irrigation
20.2 A 56 year old patient presents with exertional syncope. The most likely diagnosis is
a) HOCM
b) Long QT
c) CCF
d) Myocardial ischaemia
HOCM if these remembered options are correct
Alternative is Aortic Stenosis which is more common than HOCM in this age group
22.1 The gauge pressure on a gas cylinder does NOT necessarily represent the contents remaining if the cylinder is filled with
Nitrous oxide
Nitrous oxide boiling point -88.6C, critical temperature +36C -> so is below critical temp at room temp, therefore exists as a vapour in equilibrium with its liquid phase and is dependent upon pressure applied to it. Pressure gauge not informative – will always read ~52 bar (the pressure at which N2O liquefies at 20C). As vapour is drawn off, N2O moves from liquid to vapour phase, maintaining the equilibrium and same vapour pressure within the cylinder.
To determine contents: cylinder must be weighed and weight of empty cylinder subtracted, then number of moles of N2O in cylinder calculated using Avogadro’s number.
21.1 Of the following classes of medication for diabetes mellitus, the most likely to cause hypoglycaemia in the fasted patient are the
A. Biguanides (metformin)
B. Sulphonylureas (gliclazide)
C. Acarbose
D. SGLT2 inhibitors (empaglaflozin)
E. DPP4 inhibitors (sitagliptin)
Absolute most = Insulin, but probably not an option.
Sulphonylureas most likely
20.1 Perioperative overheating is most likely to cause worsening of symptoms of
A) Duchenne Muscular dystrophy
B) Myasthenia gravis
C) Multiple sclerosis
D) Myotonica dystrophia
E) Eaton Lambert syndrome
Answer: c) MS
CEACCP 2012 Neuromuscular disorders and anaesthesia. Part 2: specific neuromuscular disorders
Multiple sclerosis
This is the most frequently occurring demyelinating neuromuscular disorder. It is a chronic relapsing condition characterized by the formation of plaques within the brain and spinal cord. These plaques cause demyelination around the axons, resulting in weakness and spasticity as well as sensory dysfunction.
Anaesthetic considerations. Local anaesthetics may exacerbate symptoms due to the increased sensitivity of demyelinated axons to local anaesthetic toxicity.
Non-depolarizing neuromuscular blocking agents may be used in normal doses. Caution should be exercised when using depolar- izing neuromuscular blocking agents if the patient is debilitated. Temperature maintenance is important as symptoms can deteriorate with an increase in temperature, as demyelinated axons are also more sensitive to heat.
BJA: Perioperative management of myasthenia gravis (2021 - written after this MCQ):
Several factors, many associated with surgery and anaesthesia, may exacerbate myasthenia or lead to a myasthenic crisis, a life-threatening condition in which severe respiratory muscle insufficiency leads to respiratory failure.
Crises are most commonly precipitated by infection. Other precipitants include surgery, residual neuromuscular block, pain, many drugs, hypo- and hyperthermia, reduction or withdrawal of treatment, pregnancy, stress and sleep deprivation.
21.2 Regarding healthcare research, the PICO framework describes
a) Critical appraisal
b) Meta-analysis
c) Observational study
d) Systematic review
a) Critical appraisal
PICO is a mnemonic used to describe the four elements of a good clinical foreground question:
P = Population/Patient/Problem - How would I describe the problem or a group of patients similar to mine?
I = Intervention - What main intervention, prognostic factor or exposure am I considering?
C = Comparison - Is there an alternative to compare with the intervention?
O = Outcome - What do I hope to accomplish, measure, improve or affect?
23.1 The neurosurgical registrar has telephoned about a patient with a spinal cord tumour who is on the list for tomorrow. The registrar tells you the patient has Brown-Séquard syndrome (hemisection of the spinal cord). On clinical examination, below the level of the lesion, you would expect to find all EXCEPT ipsilateral
A. Hyperreflexia
B. Loss of tactile stimulation
C. Paralysis
D. Loss of pain/temperature
E. Loss of vibration/proprioception
d) loss of pain and temperature
Brown sequard. Loss of pain and temp on contralateral side.
Paralysis and loss of proprioreception on ipsilateral side
22.2 A patient presents with sepsis-induced hypoperfusion or septic shock. The minimum suggested volume of intravenous crystalloid to be administered over the first three hours as outlined in the Surviving Sepsis Guideline is
a) 10ml/kg
b) 20ml/kg
c) 30ml/kg
d) 40ml/kg
e) 50ml/kg
30ml/kg
https://journals.lww.com/ccmjournal/Fulltext/2021/11000/Executive_Summary__Surviving_Sepsis_Campaign_.14.aspx
20.2 A patient presents with a serum sodium of 110mmol/L. A feature NOT consistent with a diagnosis of syndrome of inappropriate antidiuretic hormone (SIADH) is
a. urinary sodium >40
b. Euvolemia
c. Increased cortisol
d. Urine osmolarity <100
e. Serum Na <145
d. Urine osmolarity <100
DIAGNOSTIC CRITERIA
>hypotonic hyponatraemia
>urine osmolality > plasma osmolality (<275mOsm/kg) (i.e. concentrated urine despite hypotonic blood)
>urinary Na+ > 20mmol/L
>normal renal, hepatic, cardiac, pituitary, adrenal and thyroid function
>euvolaemia (absence of hypotension, hypovolaemia, and oedema)
correction by water restriction
CAUSES (MAD CHOP)
Major Surgery
>abdominal
>thoracic
>transsphenoidal pituitary surgery (6-7 days post op)
ADH production by tumours (Ectopic)
>small cell bronchogenic carcinoma
>adenocarcinoma of pancreas/duodenum
>leukaemia
>lymphoma
>thymoma
Drugs
>antidepressants (e.g. SSRI, TCAs, MAOIs)
>psychotropics (e.g. haloperidol, chlorpromazine), carbamazepine, Na+ valproate)
>anaesthetic drugs (barbiturates, inhalational agents, oxytocin, opioids)
>ADH analogues (vasopressin, DDAVP)
>chemotherapy (e.g.Vinca alkaloids, Melphalan, Methotrexate and cyclophosphamide)
>others (e.g. NSAIDs, amiodarone, ciprofloxacin, morphine, MDMA, proton pump inhibitors)
CNS Disorders
>cerebral trauma
>brain tumour (primary or metastases)
>meningitis/encephalitis
>brain abscess
>SAH
>acute intermittent porphyria
>SLE
Hormone deficiency
>hypothyroidism
>adrenal insufficiency
Others
>Guillain-Barre Syndrome
>HIV infection (early symptomatic or AIDS)
>hereditary SIADH
>giant cell arteritis
>idiopathic (occult small cell or olfactory neuroblastoma)
Pulmonary Disorders
>pneumonia (viral, fungal, bacterial)
>TB
>lung abscess
MANAGEMENT
1. see hyponatraemia
2. fluid restrict
3. incremental increase in Na+ if indicated to avoid central pontine myelinolysis
4. medications to decrease ADH secretion
>Demeclocycline
>Tolvaptan / Conivaptan
20.2 The composition of Plasma-Lyte 148 (in mmol/l) includes
a Na 140 Mg 1.0 K 5.0 acetate 27 lactate 0
b Na 140 Mg 1.5 K 5.0 acetate 0 lactate 27
c Na 140 Mg 1.0 K 4.0 acetate 24 lactate 0
d Na 140 Mg 1.0 K 4.0 acetate 0 lactate 24
e Na 140 Mg 1.5 K 5.0 acetate 27 lactate 0
e Na 140 Mg 1.5 K 5.0 acetate 27 lactate 0
22.2 Which is least likely to cause inaccuracies in pulse oximetry
a) Anaemia
b) Vasoconstriction
c) AF
d) Methaemoglobin
e) Carboxyhaemoglobin
a) Anaemia
No effect
- Fetal haemoglobin (HbF)
- SulphHb
- Bilirubin (absorption peaks are 460, 560 and 600 nm)
- dark skin
Falsely low reading
1. Methaemoglobin (MetHb). The presence of MetHb will prevent the oximeter from working accurately and the readings will tend towards 85%, regardless of the true saturation.
2. Methylene blue. When methylene blue is used in surgery (e.g. parathyroidectomy or to treat methaemoglobinaemia), a short-lived reduction in saturation estimations is seen. Readings may fall by 65% at a concentration of 2-5 mg/kg for between 10 and 60 minutes.
3. Indocyanine green. Use of this dye (e.g. in cardiac output studies) may cause a transient reduction in recorded saturations.
4. A reduction in peripheral pulsatile blood flow produced by peripheral vasoconstriction results in an inadequate signal for analysis.
5. Venous congestion, which may be caused by tricuspid regurgitation, high airway pressures and the Valsalva manoeuvre, may produce venous pulsations which can produce low readings.
6. Venous congestion of the limb may affect readings, as can a badly positioned probe.
7. External fluorescent light in the operating theatre may cause the oximeter to be inaccurate, and the signal may be interrupted by surgical diathermy. Shivering may cause difficulties in picking up an adequate signal.
8. Nail varnish may cause falsely low readings.
Falsely high reading
1. Carboxyhaemoglobin (CoHb). CoHb (haemoglobin combined with carbon monoxide) is registered as 90% oxygenated haemoglobin and 10% desaturated haemoglobin - therefore the oximeter will overestimate the saturation.
Calibration
- Oximeters are calibrated during manufacture and automatically check their internal circuits when they are turned on.
- They are accurate in the range of oxygen saturations of 70% to 100% (+/-2%), but are less accurate under 70%. Below the saturation of 70%, readings are extrapolated.
- The data for calibration came from human volunteer studies, hence it was unethical to allow the saturations to fall below 70%. Due to the shape of the oxyhaemoglobin curve, the saturation starts to fall rapidly at 90%.
Limitations
- The oximeter averages its readings every 10-20 seconds. Hence, they cannot detect acute desaturation. The finger probe has a response time of approximately 60 seconds, whereas the ear probe has a response time of 10-15 seconds.
- The site of application should be checked at regular intervals, as pressure sores and burns have been reported.
- The pulse oximeter only provides information about oxygenation. It does not give any indication of the patient’s carbon dioxide elimination.
20.2 Cardiovascular effects of hyperthyroidism include
a) Decreased CO
b) Increased PVR
c) Increased DBP
d) Decreased SVR
Decreased SVR
Hyperthyroidism:
increases HR
increases cardiac contractility.
increases LVEF
increases diastolic relaxation
increases CO
SVR decreases
>T3 induces systemic vasodilation.
22.1 In comparison with fresh frozen plasma, cryoprecipitate contains an increased concentration of factor
a. II
b. VII
c. XI
d. XIII
d. XIII
But Fibrinogen (I) is the most significant factor that
21.2 Suxamethonium causes a sustained contraction of the extraocular muscles for up to
a) 2 minutes
b) 3 minutes
c) 5 minutes
d) 10 minutes
e) 20 minutes
d) 10 minutes
- best answer; one of those shit questions that depends on your source.
Morgan & Mikhail’s (chapter 36: anaesthesia for ophthalmic surgery):
“ Succinylcholine increases IOP by 5-10mmHg for 5-10 minutes”.
- due to prolonged contracture of the EOM
BARASH:
Succinylcholine increases IOP 7 to 10 mmHg reaching a peak pressure 1 to 2 minutes after IV administration and returns to the baseline in 5 to 7 minutes. This increase may be attenuated by pretreatment with anesthetics, although none completely eliminates the increase in IOP. In the presence of a lacerated globe, this increase in IOP may increase the extrusion of intraocular contents although greater increases in IOP may occur during crying and coughing.
Yao & Artusio’s:
- also quotes same information: increases IOP 7 to 10mmHg, returning to baseline in 5 - 7 minutes.
Stoelting’s:
Intraoccular pressure peaks at 2-4 minutes after administration and returns to normal by 6 minutes
22.2 During spinal surgery, the anaesthetic agent that is least likely to decrease motor evoked potentials is
a Ketamine
b Precedex
c Propofol
d Volatiles
e Remifentanil
Ketamine
22.1 A patient has undergone a multilevel cervical spine fusion and upon emergence from anaesthesia reports complete visual loss. Fundoscopic examination shows a pale optic disc with haemorrhages. This supports a diagnosis of
a. CRAO
b. AION
c. PION
Ischaemic optic neuropathy (anterior)
https://www.researchgate.net/figure/Top-Funduscopic-examination-revealed-pale-and-swollen-discs-with-small-hemorrhages-on_fig2_6759964
At the onset of PION, the optic disc often appears normal, pallor is only seen after 6–8 weeks. The absence of optic disc edema is a key feature distinguishing PION from AION.
Perioperative PION patients may have profound bilateral vision loss (70% of cases), and worse visual outcomes. Prexisting factors that have been related to Perioperative PION include: male sex, obesity, obstructive sleep apnea and amiodarone or PDE-5 inhibitor use.[4]
22.2 The piece of airway equipment shown is a
a. bullard laryngoscope
b. CMAC video stylet
c. lightwand
d. flexible bougie
CMAC video stylet
see image for alternative equipment images
22.2 The initial management for a seizure during an awake craniotomy is
a. GA and tube
b. Cold saline irrigation of brain
c. IV keppra
d. IV propofol
e. IV midazolam
b. Cold saline irrigation of brain
Seizures, either focal or generalized, are most likely to occur during cortical mapping. They are treated by irrigating the brain tissue with ice-cold saline. They usually cease with this treatment alone, but occasionally benzodiazepines, anti-epileptic drugs, or re-sedation with airway control are required.
An emergency plan for airway control has to be in place at all times and this can be challenging as the patient’s head is fixed in head pins and often away from the ventilator. The options include the insertion of an LMA which may be easier than oro-tracheal intubation.
Awake craniotomy is generally a well-tolerated procedure with a low rate of conversion to general anaesthesia and a low rate of complications. One of the most frequent complications is patient intolerance of the procedure, often because of the urinary catheter or prolonged positioning and intra-operative seizures.
21.1 The equipment shown in the picture is a (airway device shown)
a) Arndt bronchial blocker
b) Cohen bronchial
blocker
c) Microlaryngoscopy tube
d) Hunsaker tube
e) Parker flex ETT
Hunsaker Mon-jet ventilation tube for microlarnygeal surgery
Description:
-Laser-safe
-fluoroplastic
-self-centring catheter
Uses:
-subglottic ventilation during microlaryngeal surgery
Components:
- proximal end for attaching to jet insufflation system
-proximal end allows passage of stylet to aid insertion
-Side port at proximal end for monitopring airway pressure and ETCO2
-Outer diameter 4.3mm for maintaining good surgical access
-Green basket to keep the centre port at its tip away from tracheal mucosa and avoiding potential damage from jet ventilation
22.1 Propofol infusion syndrome is characterised by all of the following EXCEPT
a. Splenomegaly
b. ST elevation
c. Hepatomegaly
d. Rhabdomyolysis
e. Metabolic acidosis
a. Splenomegaly
Associated with high doses >4mg/kg/hr and prolonged use (>48hrs)
Safe doses of propofol infusion for sedation in ICU are considered to be 1-4mg/kg/hr
-> fatal Cases pf PRIS have been reported after infusion doses as low as 1.9-2.6mg/kg/hr
Risk factors:
i. Young age
ii. Critical illness
iii. High fat and low Carbohydrate intake
iv. Inborn errors of mitochondrial fatty acid oxidation
v. Catecholamine infusion/ High catecholamine and glucocorticoid levels
vi. Steroid therapy
vii. Severe head injuries
Characteristics:
i. Bradycardia
ii. Severe metabolic acidosis
iii. Cardiovascular collapse
iv. Rhabdomyolysis
v. Hyperlipidaemia
vi. Renal failure
vii. Hepatomegaly
Management:
- Routine monitoring of CK and triglycerides should be performed for the at risk population
○ Daily CK and triglyceridees after 48hrs of propofol infusion
○ Increasing CK in the absence of other pathology triggers suspiscion of PRIS
- Propofol immediately stopped and alternative (midazolam and alfentanil) are used
- PRIS is difficult to treat once it occurs
- CVS support provided as needed
- Renal replacement therapy may be required to treat lactic acidosis, clear propofol and its metabolites from the patient rapidly
- Catecholamine resistant shock has been reported
- Pacing has been used with limited success
ECMO has been reported and successfully used in the CVS support of PRIS
20.2 According to National Audit Project (NAP) 5, the incidence of awareness during general anaesthesia for cardiac surgery is
a) 1:400
b) 1:800
c) 1:8000
d) 1: 12000
e) 1:20000
c) 1:8000
Awareness rates
GA with no muscle relaxant = 1:136,000
GA with muscle relaxation = 1/8,000
CTS 1/8,600
E-LSCS = 1/670
Overall 1:19,000
22.2 The nerve labelled by the arrow marked H in the diagram is the
- Ulnar Nerve
- Axillary Nerve
- Median Nerve
- Medial Cutaneous nerve of the forearm
- Long Thoracic Nerve
- Dorsal Scapular Nerve
- Radial Nerve
- Suprascapular nerve
- Musculocutaneous Nerve
- Median Nerve
22.1 A patient is undergoing treatment for a malignant hyperthermia crisis. Active cooling should be ceased when the patient’s core temperature has dropped to
a. 35
b. 36
c. 37
d. 38
38
23.1 A patient has an acute attack of shingles (herpes zoster). The development of post-
herpetic neuralgia can best be reduced by the administration of
A. Ibuprofen
B. Gabapentin
C. Aciclovir
D. Amitriptyline
E. Oxycodone
D. Amitriptyline
Amitriptyline (used in low doses for 90 days from onset of the herpes zoster rash) reduces the incidence of postherpetic neuralgia
N.B
Antiviral agents started within 72 hours of onset of the herpes zoster rash accelerate the resolution of acute pain (U) (Level I) but do not reduce the incidence, severity and duration of postherpetic neuralgia
UTD
Both Gabapentinoids and TCAs are effective at TREATING postherpetic neuralgia. The former have lower risk of discontinuation due to adverse side effects.
For moderate or severe pain, use gabapentinoids.
22.2 According to the ANZICS Statement on Death and Organ Donation 2021, circulatory determination of death in the context of organ donation requires the absence of evidence of circulation for at least
a. 2min
b. 3min
c. 5 min
d. 10 min
c. 5 min
Circulatory determination of death in the context of organ donation
12 Circulatory determination of death in the context of organ donation requires the absence of spontaneous movement, breathing and circulation. Absence of circulation is evidenced by absent arterial pulsatility for 5 minutes, using intra-arterial pressure monitoring and confirmed by clinical examination (absent heart sounds and/or absent central pulse). In cases without an arterial line, electrical asystole should be observed for 5 minutes on the electrocardiogram and confirmed by clinical examination.
13 For the purposes of organ donation, circulatory determination of death should be documented using a specific form (see Appendix E) to demonstrate explicitly that all criteria set out in this Statement are met. The same criteria should be listed in local hospital forms
21.1 A respiratory effect of high flow nasal oxygen therapy is
A. Reduced RR
B. Reduced MV
C. Increased work of breathing
A. Reduced RR
BJA HFNOT
It has been demonstrated that patients with acute hypoxaemic respiratory failure experience improved comfort and tolerance with HFNOT compared with humidified oxygen via a facemask, and traditional non-invasive ventilation masks. Subjective feelings of dyspnoea AND RESPRIATORY RATES are REDUCED as is airway dryness.
23.1 In order to minimise the risk of cardiac arrhythmia, surgical diathermy has been designed to operate with
A. High frequency
B. High amplitude
C. Low frequency
D. Low amplitude
E. Using EES
A. High frequency
21.1 A patient has numbness and weakness in her hand postoperatively. You are trying to distinguish between an ulnar nerve lesion and a C8-T1 radiculopathy.
You can diagnose a C8-T1 radiculopathy if she has
A. Paraethesia of the 5th digit
B. Paraesthesia over index finger
C. Flexor carpi ulnaris function
D. Paraesthesia/sensory loss over medial forearm
E. Adductor pollicis function
Remembered answers don’t help differentiate.
[A. Paraethesia of the 5th digit - can be ulnar only
B. Paraesthesia over index finger - will be median only
C. Flexor carpi ulnaris function - can be ulnar only
D. Paraesthesia/sensory loss over medial forearm - can be ulnar only
E. Adductor pollicis function - can be ulnar only
C8-T1 radiculopathy
Will cause:
Loss of Thumb and finger abduction
(flexor pollicis brevis - suppled by both ulnar deep branch (C8-T1) and median nerve lateral terminal branch C6-T1)
Severing Ulnar nerve alone results in numbness of the 4th (ring) and 5th (little) fingers alone, and potentially medial forearm sensation (C8 and T1 supply the medial antebrachial cutaneous nerve), although loss of forearm sensation is more common in C6 radiculopathies.
All intrinsic muscles of the hand are innervated by the ulnar nerve, except for 4 muscles supplied by the median nerve. These muscles may be weak in C8-T1 radiculopathy but intact in ulnar neuropathy.
- flexor pollicis brevis
- abductor pollicis brevis, in part.
- opponens pollicis
- lateral lumbricals
AbOF the Law
may be useful—the abductor (Ab) and flexor (F) pollicis brevis, opponens pollicis (O), and lateral lumbricals (Law) are “above the law”
Or
LOAF
22.2 Based on this ECG tracing, the mode in which this pacemaker is operating is
a) VAI with intermittent failure to capture
b) AAI with intermittent failure to sense
c) DDD
d) VVI with intermittent failure to capture
e) VVI with intermittent failure to sense
e) VVI with intermittent failure to sense
20.2 Repeated unreasonable behaviour directed towards a person or group that creates a risk to health and safety is best defined as
a. Bullying
b. Harassment
c. Percipience
d. Discrimination
e. Antagonism
a. Bullying
Bullying is unreasonable behaviour that creates a risk to health and safety. It is behaviour that is repeated over time or occurs as part of a pattern of behaviour. “Unreasonable behaviour” is behaviour that a reasonable person, having regard to all the circumstances, would expect to victimise, humiliate, undermine or threaten the person to whom the behaviour is directed.
- Direct bullying – behaviour that is overt and usually involves conduct directed at a person to belittle or demean them. Examples include:
> Aggressive and intimidating behaviour.
> Belittling, degrading or humiliating comments.
> Spreading misinformation or malicious rumours.
> Interfering with a person’s property or work equipment.
> Displaying offensive material (for example pornography). - Indirect bullying – behaviour that excludes or removes benefits from a person. Examples include:
> Assigning meaningless tasks unrelated to the job.
> Setting tasks that are unreasonably below or beyond a person’s skill level.
> Deliberately changing work rosters to inconvenience particular employees.
> Deliberately withholding information that is vital for effective work performance.
Harassment is any type of unwanted behaviour that offends, humiliates or intimidates a person, and targets them on the basis of a characteristic covered by anti-discrimination law, for example gender, race, ethnicity or disability, etc. In general, harassment is any behaviour that is:
Unwelcome, not asked for and not returned.
Likely to humiliate (put someone down), seriously embarrass, offend or intimidate (threaten or scare) someone.
Based on a personal characteristic (or family or friend’s characteristic) protected by law.
Discrimination means treating a person with an identified attribute or personal characteristic as set out in legislation less favourably than a person who does not have the attribute or personal characteristic.
> Gender.
Transgender, gender history and trans-sexual status.
Pregnancy and potential pregnancy.
Childbirth or breastfeeding.
Marital status.
Sexual orientation.
Lawful sexual activity.
Disability or impairment.
Race (including colour, nationality, descent and origin).
Physical features.
Age.
Carer status and family responsibilities.
Religious belief or activity.
Political belief or activity.
Trade union membership and industrial activity.
Associated with a person who is identified by reference to any of these attributes
Sexual Harassment
Sexual harassment is against the law. Sexual harassment is unwelcome sexual behaviour, which could be expected to make a person feel offended, humiliated or intimidated. It can be physical, verbal or written.
Victimisation
Victimisation is unlawful. Victimisation occurs when a person is treated unfairly due to that person having made a complaint of sexual harassment. Victimisation is behaviour that makes a person suffer a detriment including feeling uncomfortable, isolated, insecure or
intimidated.
https://www.anzca.edu.au/resources/corporate-documents/anzca-policy-on-bullying-discrimination-and-harass.pdf
20.1According to the ANZCA PS 50 “Recommendation on Practice Re-entry for a Specialist Anaesthetist” it is recommended that after an absence of more than 12 month from practicing clinical anaesthesia a re-entry program should be offered. The duration of the program for every year of absence would usually be at least
A) 2 weeks per year off
B) 3 weeks per year off
C) 4 weeks per year off
D) 6 weeks per year off
E) 8 weeks per year off
c) 4 weeks
ANZCA PS
21.1 You are performing a regional block for analgesia following knee surgery. You have an ultrasound probe scanning the anterior mid-thigh. The muscle indicated by the arrow in the ultrasound image below is the
A: Sartorius
B: Vastus Medialis
C: Adductor Longus
D: Gracilis
E: Rectus femoris
A: Sartorius
20.1 Prolonged block post mivacurium
A)Sugammadex 4mg/kg
B)Neostigmine 100microg/kg
C)FFP 20ml/kg
D)Pralidoxime
E)Wait for it to wear off
E)Wait for it to wear off
> Neostigmine inhibits plasma cholinesterases (that could slow mivacurium metabolism), but effects are less than the inhibition of acetylcholinesterases, resulting in a “net” reversal of nondepolarizing block. Dose in stem inappropriate though.
> Administration of whole blood or FFP is not recommended unless there is another primary indication for the transfusion.
> In patients with homozygous pseudocholinesterase deficiency, will result in prolonged NMB; monitor and await.
20.2 You are performing a regional block for analgesia following knee surgery. You have an ultrasound probe scanning the anterior mid-thigh. The muscle indicated by the arrow in the ultrasound image below is the
A. biceps femoris
B. Sartorius
C. Gracillis
D. Adductor longus
E. Adductor magnus
Sartorius
repeat
22.2 The Pin Index System positions on a C size cylinder of medical oxygen are
a) 1,5
b) 2,5
c) 3,5
d) 1,6
e) these options are made up
b) 2,5
Air: 1, 5
Oxygen: 2, 5
N2O: 3,5
CO2: 2, 6
He: 2, 4
Cyclopropane 3, 6
Entonox 7
22.1 The most clinically useful indicator of effective ventilation during neonatal resuscitation is an improvement in
a. HR increases
b. Grimace
c. Resp rate
a. HR increases
22.2 The diabetic medication that, as part of its therapeutic effect, significantly prolongs gastric emptying is
a) dulaglutide
b) sitagliptin
c) metformin
d) gliclazide
e) acarbose
a) dulaglutide
The primary mechanism of action of dulaglutide, as an incretin mimetic hormone or an analogue of human glucagon-like peptide-1, is to increase insulin secretion when glucose levels are elevated, decrease glucagon secretion, and delay gastric emptying in an effort to lower postprandial glucose level.
Acarbose:
Acarbose is a complex oligosaccharide that acts as a competitive, reversible inhibitor of pancreatic alpha-amylase and membrane-bound intestinal alpha-glucoside hydrolase.
Pancreatic alpha-amylase hydrolyzes complex carbohydrates to oligosaccharides in the small intestine
By delaying the digestion of carbohydrates, acarbose slows glucose absorption, resulting in a reduction of postprandial glucose blood concentrations.
-> causes delayed gastric emptying but is not necessarily a part of its therapeutic effect
20.1 You are asked to review a previously well 48-year-old woman two hours after hysteroscopic myomectomy and endometrial ablation under general anaesthesia. Her observations are: Heart rate 70 /minute, blood pressure 130/80 mmHg, SpO2 98% on 2 litres per minute of oxygen via nasal prongs. She is drowsy but rousable, oriented to person but not to time and place. Her electrolytes show: (List of electrolytes given) The most appropriate treatment is
Na 118, K 3.0, Cr 56, Ur normal.
What is your management?
A. 500ml 0.9% NaCl
B. 3% NaCl 100ml
C. 10mmol KCl
D. Fluid restriction
a) 3% saline 100ml
100ml bolus of 3% saline (should raise serum Na by 2-3
meq/L). If no improvement in neurological symptoms, can
repeat bolus 1-2 more times at 10 minute intervals.
Frusemide only recommended if APO
22.1 A 74-year-old man presents for a femoral popliteal artery bypass procedure for peripheral limb ischaemia. Regarding its role in modifying his perioperative cardiovascular risk, clonidine
a. Increased stroke
b. No change in complications
c. Increased death
d. Increased non fatal MI
e. Increased risk of non fatal cardiac arrest
e. Increased risk of non fatal cardiac arrest
POISE II
* clonidine 200mcg per day - did not reduce the rate of composite outcome of death or nonfatal MI - but it increased the risk of clinically important hypotension and nonfatal cardiac arrest
* aspirin initiation or continuation – no significant effect on rate of composite of death or non fatal MI but increased risk of major bleeding
20.1 The Brain Trauma Foundation guideline for management of severe head trauma recommend the treatment of intracranial pressures greater than
a. 5mmHg
b. 10
c. 15
d. 22
e. 25
22
20.1 Compared to a normothermic patient, a patient with mild intraoperative hypothermia (35.0o C) will have
a. increased bleeding and normal aptt and inr
b. Increased bleeding and decreased inr
c. Increased bleeding and decreased aptt
d. Decreased bleeding
a. increased bleeding and normal aptt and inr
Bleeding because cold = we know this
Haemtology analyzer in labs warms blood to 37.2 degrees (fixes hypothermia on sample)
22.2 The medication most strongly associated with an acute primary hypotensive reaction following transfusion of blood products is
a. aspirin
b. celecoxib
c. hydralazine
d. metoprolol
e. labetalol
f. perindopril
f. perindopril
Hypotensive transfusion reactions, which account for almost 3% of all transfusion reactions, are associated with patients treated with angiotensin-converting enzyme inhibitors. The current hypothesis suggests that they are caused by bradykinin-induced vasodilation in the absence of allergic, hemolytic, or septic mechanisms. The hypotension observed frequently is unresponsive to conventional therapy with catecholamines. The suggested intraoperative management includes cessation of transfusion and washing red blood cells before blood replacement.
Hypotensive reactions to transfusion may not always be recognized. To prevent these reactions, clinicians have several options: they may discontinue the ACE inhibitor (elective transfusion), not use a leukoreduction filter (if the patient has no absolute requirement for leukoreduced blood components), use washed cellular components, or use components that have undergone leukoreduction at the collection facility or the hospital blood bank before transfusion (since bradykinin is degraded during storage).
22.1 The abnormality shown in this image (image of shoulder shown) is LEAST likely to be caused by
an injury to the
a. Accessory nerve N
b. Long thoracic N
c. Dorsal scapular N
d. Suprascapular N
Picture in Q shows medial winging of right shoulder
Answer = b
Can be caused by injury to long thoracic nerve (serratus ant’) or serratus itself
a. Accessory nerve (Trapezius paralysis, causing lateral winging)
b. Long thoracic N- (Serratus anterior paralysis, causing medial winging)
c. Dorsal scapular N (Rhomboids paralysis, causing lateral winging)
d. Suprascapular nerve (Infra and supraspinatus – doesn’t affect scapula)
In addition, here is an example of lateral winging
20.1 Cardiovascular effects of hyperthyroidism include
a) decreased diastolic relaxation
b) decreased SVR
c) decreased PVR
d) increased diastolic BP
Decreased SVR
- increased CO, increased SBP and decreased DBP with widened PP
Up to Date
Cardiovascular - Patients with hyperthyroidism have an increase in cardiac output, due both to increased peripheral oxygen needs and increased cardiac contractility. Heart rate is increased, pulse pressure is widened, and peripheral vascular resistance is decreased
22.2 A 45-year-old male received a heart transplant one month ago. He develops a new supraventricular tachyarrhythmia without hypotension during a gastroscopy. The most appropriate therapy is
a) Adenosine
b) Amiodarone
c) Digoxin
d) Esmolol
e) Verapamil
d) Esmolol
Management of Arrhythmias After Heart Transplant
https://www.ahajournals.org/doi/10.1161/CIRCEP.120.007954
In asymptomatic patients, additional cardiac monitoring such as 24-Holter or an event monitor can be useful to assess the SVT burden, and a trial of atrioventricular nodal blockers (β-blockers preferably) can be attempted with caution in view of potential risk of bradycardia. Calcium channel blockers such as diltiazem and verapamil are contraindicated in patients taking immunosuppression such as tacrolimus and cyclosporine as it can impair the metabolism CYP3A, which increases the levels of these drugs potentially causing renal toxicity.
The use of adenosine in the management of SVT has remained a subject of controversy for over a quarter century. In the past, adenosine was contraindicated in patients post-OHT due to its supersensitivity and presumed risk of prolonged atrioventricular block.
Thus, based on the aforementioned data, in patients with OHT, adenosine is feasible and safe at reduced doses (starting at 1.5 mg for patients ≥60 kg) as long as patients are closely monitored, with dose escalation as needed. Furthermore, the 2010 American Heart Association guidelines on advanced cardiovascular life support also recommended lowering the initial dose of adenosine to 3 mg for the acute management of SVT in patients with OHT.
21.1 The advantage of the Mapleson E circuit in paediatric anaesthesia is due to its
A. Can use low gas flows
B. Feel compliance
C. Assess tidal volume
D. Can rapidly change levels of CPAP
E. Low resistance
low resistance
MAPLESON E
- Derived from the Ayre T-piece used in Mapleson D circuit and functions on the same principle as Mapleson D
- The primary difference is in the length of the tubing that is increased to be greater than the patient’s tidal volume
- For spontaneous ventilation, the expiratory limb is open to the atmosphere
- It has no valves so there is no resistance to airflow nor points for possible mechanical failure
- Rebreathing is dependent on the fresh gas flow, patients minute volume and capacity of the expiratory limb
- Its main use is in paediatric patients
21.2 Analysis of variance (ANOVA) is a statistical test to determine
a) comparisons of means between two groups in normally distributed data
b) comparisons of means between two groups in non-normally distributed data
c) comparisons of means between three groups (unpaired) in normally distributed data
d) comparisons of means between three groups (unpaired) in non-normally distributed data
c) comparisons of means between three groups in normally distributed data
ANOVA (analysis of variance): comparisons of means between more than two groups or between several measurements in the same group is called analysis of variance and is frequently cited by the acronym ANOVA
22.1 A 74-year old man complains of chest pain. An electrocardiograph is performed and displayed here. The occluded coronary artery could be the
a) RCA or LCx
b) RCA
c) LAD
RCA or LCx
https://litfl.com/mi-localization-ecg-library/
21.2 The Vortex Approach to airway management does all of the following EXCEPT
a) At least 1 attempt by the most experienced clinician
b) Maximum 3 attempts at each lifeline (unless gamechanger)
c) CICO status escalates with unsuccessful best effort at any lifeline
d) Trigger for initiating CICO Rescue is SpO2 <90%
d) Trigger for initiating CICO Rescue is SpO2 <90%
- According to the Vortex Approach the trigger for initiating CICO Rescue is the inability to confirm adequate alveolar oxygen delivery following best efforts at all three upper airway lifelines.
Trigger for Initiating CICO Rescue VORTEX APPROACH
The trigger for initiating CICO Rescue is the inability to confirm adequate alveolar oxygen delivery following best efforts at all three upper airway lifelines.
Note that this trigger is independent of the oxygen saturations since, even in the unusual situation where the oxygen saturations remain high following best efforts at all three lifelines, the inability to confirm alveolar oxygen delivery means that eventual desaturation is inevitable.
Rather than being a deterrent to its performance, recognition of the need for CICO Rescue while the oxygen saturations remain high should be viewed as advantageous – providing increased time to perform this confronting procedure in a more controlled manner, thereby increasing the chance of success.
Conversely, a critically low oxygen saturation is not in itself a trigger to initiate CICO Rescue if best efforts at all three lifelines have not yet been completed.
While legitimate opportunities to enter the Green Zone in a timely fashion via the familiar upper airway lifelines remain, these should be given priority, as they are more likely to be successful than resorting to an unfamiliar and more traumatic technique.
Oxygen saturations are therefore not a relevant consideration in deciding the trigger for CICO Rescue – this is always “the inability to confirm adequate alveolar oxygen delivery following best efforts at all three upper airway lifelines”.
They are, however, a relevant consideration in making the context dependent decision of what constitutes a best effort at each lifeline in a particular situation.
This is because the oxygen saturations impact on how much time it is reasonable to invest in optimising each of the upper airway lifelines before declaring a best effort.
When the oxygen saturations are critically low it might be reasonable to have only one attempt at each lifeline before declaring a best effort, even though this means leaving some potential optimisation interventions untried.
This is because the incremental benefit of repeated attempts to optimise a lifeline that has already failed is typically low relative to untried alternative lifelines.
Thus the time expended on such low yield interventions cannot be justified when the patient is already critically hypoxaemic and alternatives (including CICO Rescue) with a substantially higher likelihood of success remain.
22.1 A test for a condition which has a prevalence of 1 in 1000 has a sensitivity of 100% and a specificity of 90%.
The probability of a patient who receives a positive result actually having the condition is
a. 1%
b. 10%
c. 50%
d. 100%
a. 1%
i.e. what is the positive predictive value (PPV) for this test
PPV= TP/ TP +FP
Negative Predictive Value = TN / TN + FN
Prevalence of 1/1000
Sensitivity of 100%
Specificity of 90%
Of patients that are disease positive in population of 1000
TP = 1
FN = 0
-> 100% sensitivity
Of patients that are disease negative in population of 1000
FP = 99
TN = 900
-> 90% Specificity
PPV= 1/ 1 + 99
= 1/100
=1%
NPV= 900/ 900 + 0
= 1/1
= 100%
23.1 The glucagon-like peptide-1 receptor (GLP-1) agonist semaglutide is associated with
A. delayed gastric emptying
B. hypoglycaemia
C. hyperlactataemia
a) delayed gastric emptying
22.1 A risk factor for the development of torsade de pointes is
a. hyperkalaemia
b. hypermagnasaemia
c. tachycardia
d. Female
d. Female
23.1 A 24-year-old man has been brought into the emergency department with a traumatic fracture of the femur. His observations are: heart rate 90 beats per minute; blood
pressure 120/80 mmHg; respiratory rate 25 breaths per minute. A peripheral VENOUS blood gas sample shows a pH of 7.29. The arterial blood pH can be estimated to be
A. 7.29
B. 7.32
C. 7.35
D. 7.4
B. 7.32
https://emj.bmj.com/content/18/5/340
The values of pH on arterial and venous samples were highly correlated (r=0.92) with an average difference between the samples of −0.4 units. There was also a high level of agreement between the methods with the 95% limits of agreement being −0.11 to +0.04 units.
https://litfl.com/vbg-versus-abg/
pH
- Good correlation
- pooled mean difference: +0.035 pH units
20.1 This type of tracheal tube is best described as a (picture of airway device shown)
a) Mini tracheostomy tube
b) South facing RAE
c) Laser tube
d) Laryngectomy tube
e) Fenestrated tracheostomy tube
laryngectomy tube
Rusch Larygoflex Reinforced Laryngectomy tube -
22.2 A patient is anaesthetised from the awake state to a state of surgical anaesthesia with propofol or a volatile anaesthetic. As the depth of anaesthesia increases, the patient’s electroencephalogram (EEG) will show oscillations that are of
a. low frequency low amplitude
b. low frequency high amplitude
c. high frequency low amplitude
d. high frequency high amplitude
b. low frequency high amplitude
Changes in the electroencephalogram during anaesthesia and their physiological basis
https://academic.oup.com/bja/article/115/suppl_1/i27/234261
Figure 1 shows raw EEG waveforms during isoflurane anaesthesia.
During light anaesthesia:
-amplitude is shallow and frequency is high.
When a higher concentration is administered:
-amplitude deepens and EEG frequency slows.
During deep anaesthesia:
- a ‘burst and suppression’ pattern becomes apparent, characterized by extreme activity, represented by high-frequency, large-amplitude waves (bursts), alternating with flat traces (suppression).
- This pattern, excluding brain ischaemia or other factors, indicates that anaesthesia is too deep. Beyond this, flat traces become dominant and, eventually waveforms are no longer apparent.
During isoflurane, sevoflurane or propofol anaesthesia, this sequence of changes in pattern is almost identical.
The major difference in EEG between the volatile agents (isoflurane or sevoflurane) and propofol is apparent in power in the theta range.
During propofol anaesthesia, theta power remains low regardless of concentration, but during isoflurane or sevoflurane anaesthesia, it increases at surgical concentrations of anaesthesia.
22.1 A drug which does NOT increase the defibrillation threshold in a patient with an implanted cardioverter defibrillator is
a. Amiodarone
b. Atropine
c. B-blocker
d. Flecainide
e. Sotalol
e. Sotalol
Drugs that INCREASE defibrillation threshold:
+ Amiodarone (Chronic)
+ Atropine
+ lignocaine
+ Diltiazem
+ Flecainide
+ Verapamil
+ Venlafaxine
+ Anaesthetic agents.
Drugs that DECREASE defibrillation threshold:
- Sotalol
- Amiodarone (acute)
- Nifekalant
Drugs with No Change in DFT
= B- blocker
= Disopyramide
= Procainamide
= Propafenone
https://www.ncbi.nlm.nih.gov/pmc/articles/PMC6304797/
22.1 Venous air embolism during frontal craniotomy is most likely to arise from the
a. Transverse sinus
b. Sigmoid sinus
c. Superior sagittal
d. Straight
c. Superior sagittal
Risk factors for venous air embolism include sitting craniotomy, posterior fossa surgery and procedures near the superior sagittal sinus. In these situations, the surgical site is often above the level of the right atrium and hence venous air entrainment is facilitated, or there is a large risk of venous exposure through which air may be entrained. Depending on the volume of air entrained, reduced end-tidal carbon dioxide, arrhythmias or right heart failure and cardiovascular collapse are all possible. However, changes in clinical parameters often occur late and are nonspecific for small volumes of entrainment. Specific monitoring for detection of venous air embolism includes non-invasive means such as end-tidal nitrogen, precordial Doppler or stethoscope and transcranial Doppler. Invasive methods include transoesophageal echocardiography, oesophageal stethoscope, pulmonary artery catheter and central venous pressure monitoring.
https://resources.wfsahq.org/atotw/anaesthesia-for-craniotomy-and-brain-tumour-resection/
22.2 The nerve labelled by the arrow marked B in the diagram is the
- Ulnar Nerve
- Axillary Nerve
- Median Nerve
- Medial Cutaneous nerve of the forearm
- Long Thoracic Nerve
- Dorsal Scapular Nerve
- Radial Nerve
- Suprascapular nerve
- Musculocutaneous Nerve
- Suprascapular nerve
21.1, 21.2 In maternal cardiac arrest the most common arrhythmia is
a) PEA
b) VT
c) VF
d) Asystole
e) SVT
a) PEA
I couldn’t find a great article on this anywhere. BJAED hasn’t got much either
21.1, 22.2 Regarding healthcare research, the SQUIRE guidelines describe
a) Standards for RCTs
b) Standards for meta-analysis
c) Standards for observational studies
d) Standards for systematic reviews
e) Standards of quality improvement
e) Standards of quality improvement
Quality Improvement
(Standards for QUality Imporvement and Reporting Excellence)
CONSORT: randomised trials
PRISMA: systematic reviews and meta-analysis (Preferred Reporting Items for Systematic reviews and meta-analysis).
STROBE: observational studies
21.1 Sensory innervation of the cornea is by the
A. Ophthalmic division of the Trigeminal nerve
B. Nasocilliary Nerve
C. Frontal Nerve
D. Oculomotor
B. Nasocilliary Nerve
a branch of Ophthalmic division of trigeminal
23.1 During standard diagnostic nocturnal polysomnography for investigation of obstructive sleep apnoea, apnoea is defined as cessation of airflow for
A. 10 sec
B. 20 sec
C. 30 sec
D. 10 sec with 3% desat
E. 20 sec with 3 % desat
A
Apnea is defined as the cessation of airflow for ten or more seconds.
Hypopnea is defined as a recognizable, transient reduction, but not a complete cessation of, breathing for ten or more seconds.
Hypopnea requires a 4% fall in SpO2
https://www.ncbi.nlm.nih.gov/books/NBK441909/#:~:text=Obstructive%20Sleep%20Apnea%20(OSA)%2C,for%20ten%20or%20more%20seconds.
20.1 You are planning to perform an adductor canal block for a patient prior to a total knee arthroplasty. The principal advantage of this approach compared to a conventional femoral nerve block below the inguinal ligament is :
a) better block of infrapatellar nerve
b) better analgesia
c) lower dose of LA needed for same analgesia
d) less motor block to quads
d) less motor block to quads
APMSE 5th edition:
Other regional and local analgesic techniques
“Adductor canal block results in similar postoperative pain outcomes following total knee arthroplasty versus femoral nerve block with less quadriceps weakness, earlier mobilisation and better functional recovery
23.1 Findings associated with massive pericardial tamponade include
a. Electrical alternans
b. Exaggerated collapsible IVC on ECHO during respiratory cycle
c. Pulses alternans
d. Kussmaul breathing
a) electrical alternans
Physical findings in Tamponade:
- A number of findings may be present on physical examination, depending upon the type and severity of cardiac tamponade
- None of the findings alone are highly sensitive or specific for the diagnosis.
Beck’s triad
1. Low arterial blood pressure
2. Dilated neck veins
3. Muffled heart sounds
- Are present in only a minority of cases of acute cardiac tamponade.
Diagnosis:
Clinical diagnosis is usually suspected based on the history and physical examination findings, which may include:
●Chest pain
●Syncope or presyncope
●Dyspnea and tachypnea
●Hypotension
●Tachycardia
●Peripheral edema
●Elevated jugular venous pressure
●Pulsus paradoxus
22.1 The mechanism of action of tranexamic acid is to inhibit the formation of
a. Plasminogen
b. Plasmin
c. Fibrin
d. fibrinogen
b. Plasmin
Plasminogen activation results in increased conversion of plasminogen to plasmin, the latter an enzyme that breakdowns the fibrinogen in blood clots.
Tranexamic acid is a synthetic derivative of lysine that exerts antifibrinolytic effects by blocking lysine binding sites on plasminogen molecules, inhibiting the interaction of plasminogen with formed plasmin and fibrin.
As a result, inhibition of plasminogen activation results in stabilization of the preformed fibrin meshwork produced by secondary hemostasis.
A 35-year-old male, three days post laparoscopic sleeve gastrectomy has ongoing nausea and vomiting. His arterial blood gas measurement is as follows: (ABG shown) The best initial therapeutic option would be
Blood gas given:
hypokalaemia
hypochloraemia
alkalosis
normal lactate
a Laparoscopy
b IV fluids and KCL
c 4% albumin
d HCl infusion
e Acetazolamide
b IV fluids and KCL
UTD Stricture post Lap Sleeve Gastrectomy management
Although sleeve strictures have been reported in 0.26 to 4 percent of LSG operations, <1 percent result in symptoms that require endoscopic or surgical intervention
A stricture can manifest acutely, early after surgery, or more chronically.
Although strictures can occur anywhere along the long staple line, they are most often located at the level of the incisura angularis for anatomic reasons.
The etiologies of post-LSG strictures are either mechanical or functional. Mechanical strictures usually derive from the use of small bougies, stapling too close to the bougie (especially at the incisura angularis), twisting of the staple line creating a “spiral” sleeve, or aggressive imbrication of the staple line.
Functional stenoses derive from edema or hematomas at the staple line. As a result, functional stenoses are transient, which present immediately following LSG and resolve spontaneously with expectant treatment.
Patients who present with obstructive symptoms during the early postoperative period should be resuscitated with hydration and antiemetic medications and studied with an upper gastrointestinal (UGI) series.
Stable patients with a stricture can be observed to allow postsurgical mucosal edema to resolve, typically in 24 to 48 hours. Patients who cannot handle their own secretions require nasogastric tube decompression, preferably placed under fluoroscopic guidance.
Patients with an acute stricture who do not respond to conservative management require early surgical reintervention. Laparoscopy could demonstrate kinking of the gastric tube, a tight suture, or a compressing hematoma.
●Endoscopy is a good initial treatment for short-segment strictures, most of which can be dilated with balloons. Multiple treatments in four- to six-week intervals are sometimes needed to treat the stricture and improve patient symptoms. Stents have also been tried but are not effective for post-LSG strictures.
●Laparoscopic seromyotomy is a treatment option for long-segment strictures . In a small retrospective study, patients treated with laparoscopic seromyotomy had good symptomatic relief.
●Conversion to an RYGB is the last option for patients with a refractory stricture who have failed all other treatments.
A 65-year-old woman has presented with a grade 2 subarachnoid haemorrhage equally suitable for treatment with surgical clipping or endovascular coiling. The factor shown to most effectively reduce mortality in early subarachnoid haemorrhage treatment is
a) Nimodipine
b) Tranexamic acid
c) Early repair
d) Atorvastatin
e) EVD placement
c) Early repair
Coil within 24 hours
Early repair - the outcome in terms of survival free of disability at 1 year is significantly better with endovascular coiling. Preferably within 24 hours
23.1 In subarachnoid block for caesarean section, hyperbaric local anaesthetic compared to regular local anaesthetic has been shown to reduce the
a. Risk of total spinal
b. Analgesic properties
c. Onset of anaesthetic
d. Offset of anaesthetic
e. Chance of inadequate anaesthetic
reduce onset time
c) faster onset of anaesthetic
https://pubmed.ncbi.nlm.nih.gov/28708665/ agrees with faster onset but for non obstetric surgery
UTD
hyperbaric bupivacaine because of its rapid onset and the option to modify the spinal level by changing the position of the operating table. Plain bupivacaine (ie, slightly hypobaric, prepared in saline) may also be used for spinal anesthesia for CD. The literature comparing safety and efficacy of hyperbaric with isobaric bupivacaine for CD is inconclusive
20.1 A patient with multiple co-morbidities has severe symptomatic aortic stenosis and is considered for an aortic valve replacement. Compared to an open surgical approach, a transcatheter aortic valve implantation (TAVI) has
a) Reduced vascular injury
b) Reduced mean valve gradient
c) Reduced paravalvular leak
d) Reduced complete heart block
e) Reduced reintervention
b) Reduced mean valve gradient
TAVI decreased:
- AKI
- AF
- Transfusion
- Mean prosthetic valve gradient
TAVI increased:
- Major vascular complications
- Permanent pacemaker implantation
- Paravalvular regurgitation
- Need for re-intervention
20.1 IgE-related penicillin anaphylaxis crossover rate with cephazolin
a. 0.1%
b. 1%
c. 5%
d. 10%
1%
BJA ED
23.1 According to the Australian and New Zealand Anaesthetic Allergy Group (ANZAAG) guidelines for the investigation of a suspected anaphylactic reaction, serum tryptase should be measured at
a. 0, 4, 12
b. 0, 2, 4, 24
c. 0, 1, 4, 24
d. 0, 4 , 6, 24
e. 1, 6, 24
c) 0, 1, 4, 24
Serum tryptase levels are recommended to be collected as soon as possible after the onset of symptoms and then at 1 hour, 4 hours and after 24 hours.
https://www.anzca.edu.au/resources/professional-documents/endorsed-guidelines/anaphylaxis-guideline-2022.pdf
22.2 For a skewed distribution of data the best measure of dispersion of data is the
a) range
b) mode
c) standard deviation
d) variance
e) Interquartile Range
f) median
e) Interquartile Range
https://statisticsbyjim.com/basics/skewed-distribution/
https://statisticsbyjim.com/basics/variability-range-interquartile-variance-standard-deviation/
A measure of variability is a summary statistic that represents the amount of dispersion in a dataset. How spread out are the values? While a measure of central tendency describes the typical value, measures of variability define how far away the data points tend to fall from the center.
In statistics, variability, dispersion, and spread are synonyms that denote the width of the distribution. Just as there are multiple measures of central tendency, there are several measures of variability.
When a distribution has lower variability, the values in a dataset are more consistent. However, when the variability is higher, the data points are more dissimilar and extreme values become more likely. Consequently, understanding variability helps you grasp the likelihood of unusual events.
> Range is easy to understand, it is based on only the two most extreme values in the dataset, which makes it very susceptible to outliers. If one of those numbers is unusually high or low, it affects the entire range even if it is atypical.
> The interquartile range is the middle half of the data. To visualize it, think about the median value that splits the dataset in half. The interquartile range is the middle half of the data that is in between the upper and lower quartiles. In other words, the interquartile range includes the 50% of data points that fall between Q1 and Q3
> The interquartile range is a robust measure of variability in a similar manner that the median is a robust measure of central tendency. Neither measure is influenced dramatically by outliers because they don’t depend on every value. Additionally, the interquartile range is excellent for skewed distributions, just like the median.
> when you have a normal distribution, the standard deviation tells you the percentage of observations that fall specific distances from the mean. However, this doesn’t work for skewed distributions, and the IQR is a great alternative.
> Variance is the average squared difference of the values from the mean. Unlike the previous measures of variability, the variance includes all values in the calculation by comparing each value to the mean. To calculate this statistic, you calculate a set of squared differences between the data points and the mean, sum them, and then divide by the number of observations. Hence, it’s the average squared difference.
> While higher values of the variance indicate greater variability, there is no intuitive interpretation for specific values. Despite this limitation, various statistical tests use the variance in their calculations. For an example, read my post about the F-test and ANOVA. While it is difficult to interpret the variance itself, the standard deviation resolves this problem!
> The standard deviation is the standard or typical difference between each data point and the mean. When the values in a dataset are grouped closer together, you have a smaller standard deviation. On the other hand, when the values are spread out more, the standard deviation is larger because the standard distance is greater
> The standard deviation is just the square root of the variance. Recall that the variance is in squared units. Hence, the square root returns the value to the natural units. The symbol for the standard deviation as a population parameter is σ while s represents it as a sample estimate. To calculate the standard deviation, calculate the variance as shown above, and then take the square root of it. Voila! You have the standard deviation!
> People often confuse the standard deviation with the standard error of the mean. Both measures assess variability, but they have extremely different purposes.
> When you have normally distributed data, or approximately so, the standard deviation becomes particularly valuable. You can use it to determine the proportion of the values that fall within a specified number of standard deviations from the mean. For example, in a normal distribution, 68% of the values will fall within +/- 1 standard deviation from the mean. This property is part of the Empirical Rule. This rule describes the percentage of the data that fall within specific numbers of standard deviations from the mean for bell-shaped curves.
20.1 A 64-year-old man presenting for elective surgery is on thyroxine 100 mcg daily. His thyroid function tests are: (Thyroid function tests shown). These results are most consistent with
TFTs thryoxine TSH < .05 T4 and T3 completely normal
a) Hypophysectomy
b) Subclinical Hyperthyoirdism
c) Sick euthyroid
d) Toxic Multinodular goitre
b) Subclinical Hyperthyoirdism
Subclinical hyperthyroidism: low TSH, normal T3 + T4
Clinical hyperthyroidism: low TSH, high T3, high/normal T4
Subclinical hypothyroidism: high TSH, normal T3 + T4
Clinical hypothyroidism: high TSH, low/normal T3, i T4
Amiodarone: high/normal TSH, low T3 (2o to inhibition of pituitary T4 to T3 conversion)
Sick euthyroid: low TSH, low T3
Hypophysectomy (central hypothyroidism): low/normal TSH/T3/T4
Compliant on thyroxine: normal TSH, high/normal T3, low T4
Non-compliant w thyroxine (pt taking several tabs prior to Dr’s appointment): high TSH, normal T4
20.1 What is the arrow pointing to?
a. Ilioinguinal
b. Iliohypogastric
c. Genitofemoral
d. Accessory Obturator
e. Obturator
e. Obturator
21.1 A 25-year-old ASA I patient develops ongoing seizures five minutes after receiving a brachial plexus block with ropivacaine. Of the following, the most suitable initial intravenous treatment is
a) Midazolam
b) Intralipid
c) Propofol
d) Levetiracetam
e) Phenytoin
Control seizures first
a) Midazolam if an option
or
c) propofol
or
treat seizures 1st followedLAST
- ABCD
- Intralipid 1.5mL/kg
23.1 A 60-year-old woman presents for thrombectomy with left lower leg ischaemia. She has not received any medications since presentation and takes none at home. The sole abnormality on laboratory testing is an activated partial thromboplastin time (APTT) of 52 seconds. The most likely cause of the raised APTT is
a. Cold agglutinins
b. Erroneous reading
c. Lupus anticoagulant
d. Factor VII deficiency
e. Haemophilia A
c. Lupus anticoagulant
(normal PT, raised APTT)
Lupus anticoagulant (more likely to be associated with thrombosis than bleeding)
https://www.uptodate.com/contents/image?imageKey=HEME%2F79969
21.1 The ANZCA Choosing Wisely recommendations advise avoiding all of the following EXCEPT
a) Doing an epidural on a patient who is labouring normally with a normal pregnancy and no comorbidities
b) Giving blood transfusion on a healthy 20yo male with Hb > 70g/L, except when severe and symptomatic
c) Giving an anaesthetic to a high risk patient with severe comorbidities without risk stratifying them and taking an anaesthetic history and assessment
d) Routinely performing preoperative blood investigations, chest X-ray or spirometry prior to surgery, but instead, ordering in response to patient factors, symptoms and signs, disease, or planned surgery.
e) Ordering cardiac stress testing for asymptomatic patients prior to undergoing low to intermediate risk non-cardiac surgery.
doing an epidural on a patient who is labouring with normal pregnancy and no comorbidities
- Avoid routinely performing preoperative blood investigations, chest X-ray or spirometry prior to surgery, but instead order in response to patient factors, symptoms and signs, disease, or planned surgery.
- Avoid ordering cardiac stress testing for asymptomatic patients prior to undergoing low to intermediate risk non-cardiac surgery.
- Avoid administering packed red blood cells (blood transfusion) to a young healthy patient with a haemoglobin of ≥70g/L who does not have on-going blood loss, unless the patient is symptomatic or haemodynamically unstable.
- Avoid initiating anaesthesia for patients with limited life expectancy, at high risk of death or severely impaired functional recovery, without discussing expected outcomes and goals of care.
- Avoid initiating anaesthesia for patients with significant co-morbidities without adequate, timely preoperative assessment and postoperative facilities to meet their needs.
- Avoid routine prescription of slow-release opioids in the management of acute pain unless there is a demonstrated need, close monitoring is available and a cessation plan is in place
20.2, 22.2 The modified Aldrete scoring system is used for determining the
a) Predicts difficulty of bag mask ventilation
b) Safety of day surgery
c) Discharge from recovery
d) Modification of recovery criteria
e) Discharge from hospital
c) Discharge from recovery
Aldrete score, which includes five elements (activity, respiration, circulation, consciousness, oxygen saturation) [16].
The original scoring system was developed before the invention of pulse oximetry and used the patient’s colouration as a surrogate marker of their oxygenation status. A modified Aldrete scoring system was described in 1995 which replaces the assessment of skin colouration with the use of pulse oximetry to measure SpO2.
The Modified Aldrete system includes five additional elements that are particularly useful during the Phase II recovery period prior to discharge to home (dressing, pain, ambulation, feeding, urine output)
20.2 Idarucizumab reverses the anticoagulant effect of
a) Clopidogrel
b) Rivaroxaban
c) Dabigatran
d) Apixaban
e) Rivaroxaban
c) Dabigatran
Idarucizumab (Praxbind) is a monoclonal antibody to dabigatran
Dabigatran bleeding may be treated with:
- idarucizumab
- haemodialysis
- TXA will decrease fibrinolysis and has some effect
- FFP also has some effect
Humanized monoclonal antibody fragment (Fab) indicated in patients treated with dabigatran (Pradaxa) when reversal of the anticoagulant effects are needed for emergency surgery or urgent procedures, or in the event of life-threatening or uncontrolled bleeding
- 5 g IV, provided as 2 separate vials each containing 2.5 g/50 mL (see Administration)
- Limited data support administration of an additional 5 g
Dosage Modifications
Renal impairment: Renal impairment did not impact the reversal effect of idarucizumab; no dosage adjustment required
Hepatic impairment: Not studied
Dosing Considerations
This indication is approved under accelerated approval based on a reduction in unbound dabigatran and normalization of coagulation parameters in healthy volunteers; continued approval for this indication may be contingent upon the results of an ongoing cohort case series study
22.2 When used for prolonged analgesia in a healthy adult, the recommended maximum dose of ropivacaine via continuous infusion or bolus dosing in a 24-hour period is
a) 450mg
b) 600mg
c) 770mg
d) 1200mg
c) 770mg
Product info: Fresenius-Kabi
When prolonged epidural blocks are used, either by continuous infusion or repeated bolus administration, the risks of reaching a toxic plasma concentration or inducing local neural injury must be considered. Cumulative doses of up to 800 mg ropivacaine for surgery and postoperative analgesiaadministered over 24 hours were well tolerated in adults, as were postoperative continuous epidural infusions at rates up to 28 mg/hour for 72 hours.
product info: pfizer
When prolonged blocks are used, either through continuous infusion or through repeated bolus administration, the risks of reaching a toxic plasma concentration or inducing local neural injury must be considered. Experience to date indicates that a cumulative dose of up to 770 mg ropivacaine hydrochloride administered over 24 hours is well tolerated in adults when used for postoperative pain management: i.e., 2016 mg. Caution should be exercised when administering ropivacaine for prolonged periods of time, e.g., > 70 hours in debilitated patients
20.2 The most common cause of post operative visual loss after spinal surgery is
a) Corneal abrasion
b) Retinal artery occlusion
c) Central retinal vein occlusion
d) Ischaemic optic neuropathy
e) Occipital infarct
a) Ischaemic optic neuropathy
Postoperative visual loss (POVL) occurs in 1/60 000–1/125 000 operations. Spinal surgery has the highest incidence of POVL.
American Society of Anesthesiologists (ASA) Post Operative Visual Loss Registry, spinal surgery accounted for 93/131 (70%) of all cases of visual loss after non-ophthalmic surgery.
Of these:
> 83 were attributable to ischaemic optic atrophy (ION)
> 10 were caused by central retinal artery occlusion (CRAO).
CRAO
- caused by direct pressure on the globe causing raised intraocular pressure and compromising retinal perfusion.
- visual loss is usually unilateral and associated with other signs of pressure (e.g. ophthalmoplegia, ptosis, or altered sensation in the territory of the supraorbital nerve).
- Initial careful positioning of the head and regular checks throughout the procedure in case of movement minimizes the risk
- documentation of eye checks should occur every 30mins and horseshoe shaped head rests should be avoided in prone patients
ION
> associated with:
- male gender
- obesity
- increasing blood loss
- operative procedures >6 hrs in length.
- The use of the Wilson frame has also been implicated.
> final common pathway is thought to be hypoperfusion of the optic nerve, there is no clear association with either intraoperative systemic hypotension or with the presence of peripheral vascular disease or diabetes.
> recently updated ASA practice advisory for POVL associated with spinal surgery recommends regular intraoperative testing of haemoglobin concentration. However, it was unable to suggest a transfusion threshold that would prevent POVL.
Other possible causes of POVL:
1. Cortical ischaemia
2. Haemorrhage into a cerebral tumour.
In high-risk cases, assessment of vision should be performed as soon as possible in PACU and an early ophthalmic opinion sought if there is a suggestion of visual compromise.
Initial management
1. optimization of arterial pressure
2. oxygenation
3. correction of anaemia.
Treatment with agents such as acetazolamide has not been beneficial and there is rarely any useful improvement in vision with either injury, so attention should be focused on preventative measures:
1. Careful positioning with the head at the same level as the heart
2. Meticulous haemostasis,
3. Possibly staging prolonged procedures should be considered.
Because of the devastating nature of this complication, patients should be informed of an increased incidence of visual loss after spinal operations that are expected to be of prolonged duration and associated with significant blood loss.
22.1 When using an endotracheal tube in an adult, the highest recommended cuff pressure to avoid mucosal ischaemia is
a. 10cmH2O
b. 20
c. 30
d. 40
e. 50
c. 30cmH2O
21.2 Of the following drugs, the least likely to cause pulmonary vasodilation when used at low
doses in patients with chronic pulmonary hypertension is
a) Dopamine
b) Dobutamine
c) Vasopressin
d) Milrinone
dopamine
- least likely to cause pulmonary vasodilation (all the others do to my knowledge)
- From UP TO DATE:
> At low doses of 1 to 3 mcg/kg per min, dopamine acts primarily on dopamine-1 receptors to dilate the renal and mesenteric artery beds
> At 3 to 10 mcg/kg per min (and perhaps also at lower doses), dopamine also stimulates beta-1 adrenergic receptors and increases cardiac output, predominantly by increasing stroke volume with variable effects on heart rate.
> At medium-to-high doses, dopamine also stimulates alpha-adrenergic receptors, although a small study suggested that renal arterial vasodilation and improvement in cardiac output may persist as the dopamine dose is titrated up to 10 mcg/kg per min
*clinically, the haemodynamic effects of dopamine demonstrate individual variability
Dobutamine (inodilator):
- selective β1-agonist that increases cardiac contractility and reduces pulmonary vascular and systemic vascular resistances
Vasopressin:
- vasopressin may have pulmonary vasodilatory effects in addition to a systemic vasoconstrictive effect
Milrinone (inodilator):
- the phosphodiesterase-3 inhibitors, milrinone and enxoimone, have positive inotropic effects combined with the capacity to reduce RV afterload (‘inodilators’) without significant chronotropic effect, but they can be associated with significant systemic hypotension
22.2 According to ANZCA PS54(A), an anaesthetic machine requiring electrical power must, in the event of mains power failure, be able to operate under battery backup power for a minimum of
a) 30 min
b) 60 min
c) 120 min
d) 240 min
a) 30 min
If the anaesthesia machine requires electrical power for normal operation, a backup power supply must be a part of the machine and permit normal operation for at least 30 minutes after a mains power supply failure. An alarm must be activated at the time of the mains failure and the state of the reserve power supply must be indicated while it is in use.
https://www.anzca.edu.au/getattachment/f05e02ec-2023-4c50-b57f-9549ea0c4183/PS54(A)-Position-statement-on-the-minimum-safety-requirements-for-anaesthesia-machines-and-workstations-for-clinical-practice-2021#page=
23.1 In patients with primary adrenal insufficiency, a markedly elevated renin is most likely due to
A Insufficient corticosteroid replacement
B Insufficient fludrocortisone replacement
C Excessive corticosteroid replacement
D Excessive fludrocortisone replacement
b. Insufficient fludrocortisone replacement
In Primary Adrenal Insufficency, cortisol deficiency results in decreased feedback to the HPA axis, leading to increased secretion of ACTH to stimulate the adrenal cortex. Simultaneously, MCs deficiency causes increased release of renin by the juxtaglomerular apparatus of the kidneys.
22.1 A 30-year-old parturient presents in labour. She has a history of Addison’s disease from autoimmune adrenalitis and has been taking prednisolone 6 mg daily for ten years. On presentation the patient is given hydrocortisone 100 mg intravenously. The most appropriate steroid replacement regimen the patient should receive during labour is
a. 25mg TDS hydrocortisone
b. 8mg/hr hydrocortisone
c. 6mg PO prednisone
8mg/hr
Guidelines for mx of glucocorticoids during the perioperative period for patients with adrenal insufficiency
https://associationofanaesthetists-publications.onlinelibrary.wiley.com/doi/10.1111/anae.14963
23.1 The next patient on your endoscopy list is a 50-year-old woman who has been scheduled for gastroscopy and colonoscopy under sedation, after unsatisfactory
proceduralist-supervised midazolam and fentanyl sedation in the past. She states that she has egg anaphylaxis and carries an adrenaline (epinephrine) auto-injector.
The most appropriate agent to use for her sedation is
A. Propofol
B. Ketamine
C. Remifentanil
D. Sevofluarane
A
The situation in adults is straightforward: there is convincing evidence that propofol is safe in patients who are allergic to peanut and/or soy and/or egg.
BJA Ed
https://academic.oup.com/bja/article/116/1/11/2566111
21.2 The oral morphine equivalent of tapentadol 50 mg (immediate release) is
a) 5mg
b) 10mg
c) 15mg
d) 20mg
e) 25mg
c) 15mg
Oral Tapentadol 25mg = 8mg Oral Morphine
Oral Oxycodone 5mg = 8mg Oral Morphine
Oral Tramadol 25mg = Oral Morphine 5mg
Oral Hydromorphone 4mg = Oral Morphine 20mg
S/L Buprenorphine 200mcg = 8mg Oral Morphine
IV Oxycodone 5mg = Oral Morphine 15mg
IV Morphine 5mg = Oral Morphine 15mg
IV Hydromorphone 1mg = Oral Morphine 15mg
20.1 The substance that should be avoided in a patient with history of anaphylaxis to MMR vaccine is
a) Protamine
b) Penicillin
c) Sulphonamides
d) Gelofusine
gelofusin
Anaphylaxis after vaccination is probably due to anaphylactic sensitivity to gelatin or neomycin, not an egg allergy
21.1 The implemention of comprehensive multidisciplinary geriatric assessments in the peri-operative period has been shown to
a) Reduce mortality
b) Reduce AKI
c) Reduce periop risk of MACE
d) Reduce length of stay
e) Increase cancellation for surgery
d) Reduce length of stay
less time in aged care and reduced mortality
Blue book 2019:
“A referral to a geriatrician for further assessment and management may also be warranted in the preoperative period.
Indeed, a meta-analysis of perioperative interventions to reduce delirium found that a geriatrics consultation before surgery was one of only two perioperative interventions that were associated with a reduction in delirium.”
Association of anaesthetists: The impact of pre-operative comprehensive geriatric assessment on postoperative outcomes in older patients undergoing scheduled surgery: a systematic review
- reduced medical complications
> reduced postop delirium,
> reduced pneumonia
> reduced pressure sores - fewer cancellations
- reduced length of stay
22.2 A patient presents for endoscopic retrograde cholangiopancreatography (ERCP) with a history of previous post-ERCP pancreatitis. The management most likely to reduce the likelihood of pancreatitis is
a) Gentamicin
b) PR indomethacin
c) Creon post op
d) Preop smoking cessation
b) PR indomethacin
APMSE 5th edition 8.6.1.3: Only rectal NSAIDs are effective for reducing post ERCP pancreatitis, particularly indomethacin. Epidural > PCA for severe acute pancreatitis
A Randomized Trial of Rectal Indomethacin to Prevent Post-ERCP Pancreatitis
https://www.nejm.org/doi/full/10.1056/NEJMoa1111103
Nonsteroidal antiinflammatory drugs (NSAIDs) are potent inhibitors of phospholipase A2, cyclooxygenase, and neutrophil–endothelial interactions, all believed to play an important role in the pathogenesis of acute pancreatitis. NSAIDs are inexpensive and easily administered and have a favorable risk profile when given as a single dose, making them an attractive option in the prevention of post-ERCP pancreatitis. Preliminary studies evaluating the protective effects of single-dose rectal indomethacin or diclofenac in post-ERCP pancreatitis have been conducted, and a meta-analysis suggests benefit.
Results
A total of 602 patients were enrolled and completed follow-up. The majority of patients (82%) had a clinical suspicion of sphincter of Oddi dysfunction. Post-ERCP pancreatitis developed in 27 of 295 patients (9.2%) in the indomethacin group and in 52 of 307 patients (16.9%) in the placebo group (P=0.005). Moderate-to-severe pancreatitis developed in 13 patients (4.4%) in the indomethacin group and in 27 patients (8.8%) in the placebo group (P=0.03).
Conclusions
Among patients at high risk for post-ERCP pancreatitis, rectal indomethacin significantly reduced the incidence of the condition.
The amount of intravenous potassium chloride required to raise the plasma potassium level from 2.8 mmol/L to 3.8 mmol/L in a normal adult is approximately
a. 10mmol
b. 20mmol
c. 30mmol
d. 100mmol
e. 200mmol
e. 200mmol
K+ < 3.0 mmol/L: 200-400 mmol of potassium are required to raise it by 1 mmol/L
K+ > 3.0 mmol/L: 100-200 mmol of potassium are required to raise it by 1 mmol/L
Hypokalaemia P. GLOVER
https://www.cicm.org.au/CICM_Media/CICMSite/CICM-Website/Resources/Publications/CCR Journal/Previous Editions/September 1999/05-Sept_1999_Hypokalaemia.pdf
If the serum potassium level is greater than 3 mmol/L, 100-200 mmol of potassium are required to raise it by 1 mmol/L; 200 - 400 mmol are required to raise the serum potassium level by 1 mmol/L when the potassium concentration is less than 3mmol/L, assuming a normal distribution between cells and the intracellular space, and a linear relationship between plasma potassium and body deficit (which has been described, i.e. 0.27 mmol/L/100 mmol deficit/70 kg), exists. The rate of administration of potassium will be influenced by the presence and seriousness of the pathophysiological changes caused by hypokalaemia. The underlying disorder should also be treated simultaneously.
22.2 A 72-year-old patient is undergoing resection of an anterior skull based tumour using a combined endoscopic and frontal craniotomy approach. Seven hours into the procedure she has a large diuresis of pale urine and you suspect she may have developed diabetes insipidus. The most appropriate test result to confirm your diagnosis in this setting is a
a. Low serum ADH levels
b. Sequentially increasing Na levels
c. Serum osmolality <260
d. Urine Na >40
e. Urine specific gravity > something
b. Sequentially increasing Na levels
20.2 You are called to assist in the resuscitation of a 75-year-old female patient in the emergency department who is hypotensive and hypoxaemic in extremis. The image shown is of a focused transthoracic echocardiogram, parasternal short axis view. The most likely diagnosis is
a) Pulmonary embolism
b) Anterior MI
c) Cardiac tamponade
d) Pneumothorax
a) Pulmonary embolism
A bit about the RV in PE:
The right ventricle drapes around the LV. In response to an acute Pulmonary Embolus (PE) it first dilates. The RV can’t generate much force without training, sowhen the Pulmonary Vascular Resistance (PVR) first rises with a PE, thepulmonary arterypressures don’t actually rise substantially because the RV can’t generate largepressures.
Looking at the ventricle in short axis, the septum maybow towardstheLV which will form aD shape indiastole,producing a“volumeoverloaded right ventricle” appearance.
Only later whenthe RV has beentrainedwill it be able togenerate higher pressures. If the LV is D shaped insystole, this is a “pressureoverloaded right ventricle”.
Acute cor pulmonale with bothpressureANDvolumeoverload (D shape insystoleANDdiastole)is often absent.
22.2 The nerve labelled by the arrow marked A in the diagram is the
- Ulnar Nerve
- Axillary Nerve
- Median Nerve
- Medial Cutaneous nerve of the forearm
- Long Thoracic Nerve
- Dorsal Scapular Nerve
- Radial Nerve
- Suprascapular nerve
- Musculocutaneous Nerve
- Dorsal Scapular Nerve
21.1 A five-year-old child weighing 25 kg is to be strictly nil by mouth overnight following a laparotomy. The most appropriate fluid prescription is
a. 65ml/hr N Saline
b. 45ml/hr N saline
c. 45ml/hr N Saline w 5% dex
d. 65ml/hr .45% saline w 2.5% dex
e. 65ml/hr .45% saline w 5% dex
b. 45ml/hr N saline w 5% dextrose
Nsaline + 5% dextrose is fluid of choice
A guide to paediatric anaesthesia fluid management
-421 rule overestimates fluid resus
-due to stress response from ADH release
-post-op fluid maintenance is 2/3rds calculated due to increased ADH
-never use hypotonic fluids
https://www.rch.org.au/clinicalguide/guideline_index/Intravenous_fluids/
23.1 A level two check of the inhalational anaesthesia delivery device does NOT include checking the
A. Accurate delivery of volatile concentration from vaporiser
B. Connection of vaporiser and seating
C. Secure vaporiser cap
D. Adequate filling of vaporizers
E. Power to vaporiser
a) Accurate delivery of volatile concentration from vaporiser
PS31
Level two check should be performed at the start of each anaesthetic list.
4.2.3.2 Inhalational anaesthesia delivery devices (vapouriser)
4.2.3.2.1 Ensure electricity is connected to vapourisers that require it.
4.2.3.2.2 Check the anaesthetic liquid level is within marked limits.
4.2.3.2.3 Ensure all filling ports are sealed.
4.2.3.2.4 Check correct seating, locking and interlocking of detachable vapourisers or casettes.
4.2.3.2.5 Test for circuit leaks with a cassette installed or for each vapouriser in the “on” and “off” state.
4.2.3.3 Check for machine leaks upstream from the common gas outlet or breathing system, using a protocol appropriate for the anaesthesia delivery system.
22.2 A 48-year-old man is day two post-laparoscopic high anterior resection. He has used 42 mg of intravenous morphine in the past 24 hours. You wish to start him on oral tapentadol immediate release. The most appropriate equianalgesic dosage would be
a) 50mg six times a day
b) 100mg six times a day
c) 200mg six times a day
d) 300 mg six times a day
a) 50mg six times a day
42mg IV Morphine = 126mg Oral Morphine
126/8= 15.75
15.75 x 25 = 393.75 (*400mg/day Tapentadol)
Option 50mg 6 times a day = 300mg
As direct OME to tapentadol conversion is 400mg, a 300mg dose represents a 25% dose reduction, which is line with a 25-50% dose reduction due to incomplete cross-tolerance during opioid rotation.
Oral Tapentadol 25mg = 8mg Oral Morphine
Oral Oxycodone 5mg = 8mg Oral Morphine
Oral Tramadol 25mg = Oral Morphine 5mg
Oral Hydromorphone 4mg = Oral Morphine 20mg
S/L Buprenorphine 200mcg = 8mg Oral Morphine
IV Oxycodone 5mg = Oral Morphine 15mg
IV Morphine 5mg = Oral Morphine 15mg
IV Hydromorphone 1mg = Oral Morphine 15mg
20.1 You are inserting a pulmonary artery catheter in an intubated patient prior to cardiac surgery, and a significant amount of blood appears in the endotracheal tube. The most appropriate specific initial management is to:
A. Remove PAC and insert DLT
B. Wedge PAC and insert DLT
C. Wedge PAC and insert bronchial blocker
D. Withdraw PAC 2 cm and insert DLT
E. Withdraw PAC and insert bronchial blocker
F. Inflate balloon
D. Withdraw PAC 2 cm and insert DLT
LITFL: Pulmonary haemorrhage after PAOP measurement
a life threatening time-critical emergency pulmonary artery rupture caused by inflation of the pulmonary artery catheter (PAC) balloon during ‘wedging’ (measurement of the pulmonary artery occlusion pressure) some experts advise against measuring PAWP because of the risk of pulmonary artery rupture 0.2% risk, 30% mortality
RISK FACTORS
pulmonary hypertension mitral valve disease anticoagulants age >60 years
MANAGEMENT
Goals
prevent further pulmonary haemorrhage stop bleeding resuscitate
Call for help
ICU consultant anaesthetist/ OT cardiothoracic surgeon interventional radiology
Resuscitation
A may have to be emergently intubated if not already B FiO2 1.0 controlled ventilation if able to recognize which lung is haemorrhaging may be able to perform lung isolation (insert single lumen tube into unaffected side, exchange for a double lumen tube or use bronchial blocker with bronchoscopic assistance) apply PEEP to tamponade wound C large bore IV cannulae, fluids, blood products, inotropes
Specific therapy
Lay the patient ruptured side down withdraw pulmonary catheter 2-3 cm with balloon down then refloat PAC with balloon inflated to occlude pulmonary artery (to try to tamponade bleeding) stop antiplatelet agents and anticoagulants give reversal agents: — protamine for heparin — platelets for anti-platelet agents give blood products as indicated by FBC, coags and clinical state interventions — angiogram or bronchoscopy to isolate pulmonary vessel involved — if bleeding doesn’t settle will require lobectomy
21.2, 22.2 A patient has blunt chest trauma. A thoracotomy is indicated if the immediate blood drainage after closed thoracostomy is greater than
a) 500mL
b) 750mL
c) 1L
d) 1.2L
e) 1.5L
1,500 mL immediately
OR
200 mL/hr in the first 2-4 hours
21.2 A 30 year old athlete undergoing a knee arthroscopy under general anaesthesia becomes tachycardic intraoperatively. A 12-lead electrocardiogram (ECG) is obtained. The most likely diagnosis is
a) Atrial fibrillation
b) Atrial flutter
c) Sinus tachycardia
d) WPW
d) WPW
Type B pattern
LITFL:
ECG features of WPW in sinus rhythm
-> PR interval < 120ms
-> Delta wave: slurring slow rise of initial portion of the QRS
-> QRS prolongation > 110ms
-> Discordant ST-segment and T-wave changes (i.e. in the opposite direction to the major component of the QRS complex)
-> Pseudo-infarction pattern in up to 70% of patients — due to negatively deflected delta waves in inferior/anterior leads (“pseudo-Q waves”), or prominent R waves in V1-3 (mimicking posterior infarction
Can be left-sided (Type A) or right-sided (Type B), and ECG features will vary depending on this:
Left-sided AP:
produces a positive delta wave in all precordial leads, with R/S > 1 in V1.
(Dominant R Wave in V1)
Sometimes referred to as a type A WPW pattern
Right-sided AP:
produces a negative delta wave in leads V1 and V2.
Sometimes referred to as a type B WPW pattern
Tachyarrhythmias in WPW
There are only two main forms of tachyarrhythmias that occur in patients with WPW
- Atrial fibrillation or flutter.
-> Due to direct conduction from atria to ventricles via an AP, bypassing the AV node - Atrioventricular re-entry tachycardia (AVRT).
-> Due to formation of a re-entry circuit involving the AP
Breakdown of Type A example:
- Sinus rhythm with a very short PR interval (< 120 ms)
- Broad QRS complexes with a slurred upstroke to the QRS complex — the delta wave
- Dominant R wave in V1 suggests a left-sided AP, and is sometimes referred to as “Type A” WPW
- Tall R waves and inverted T waves in V1-3 mimicking right ventricular hypertrophy (RVH) — these changes are due to WPW and do not indicate underlying RVH
- Negative delta wave in aVL simulating the Q waves of lateral infarction — this is referred to as the “pseudo-infarction” pattern
21.1 A 45-year-old man has the following results on his blood biochemistry testing (Liver function tests shown). The most likely diagnosis is
a. Cholecystitis
b. Metastatic liver disease
c. Hepatitis C
d. Chronic liver disease
e. Paracetamol toxicity
a. Cholecystitis
Example and explanation taken from RACGP:
The raised AlP relative to Alt suggests cholestasis and the high GGt confirms liver origin. The mild hyperbilirubinaemia confirms the clinical impression of jaundice. Biliary disease is highly likely with gallstones the most likely differential diagnosis. however, this clinical picture may also occur in drug reactions or infiltrative conditions. After a careful history, abdominal ultrasound is the most appropriate next investigation.
23.1 The bioavailability of an oral dose of ketamine is approximately
A. 10%
B. 20%
C. 40%
D. 70%
E. 80%
B. 20%
25% (a few studies have higher ranges but typically around 20-25%)
https://doi.org/10.1192/bjp.bp.115.165498
bonus neuro radiology questions
A 24 year-old man developed an explosive headache followed by nausea and vomiting.
He undergoes cerebral angiography.
His cerebral aneurysm is in the
a. Anterior Cerebral Artery
b. Anterior communicating artery
c. MCA
d. PCA
e. Basilar artery
b. Anterior communicating artery
https://case.edu/med/neurology/NR/SubarachnoidHemorrhageAComm.htm
The ANZCA guidelines regarding pre-operative oral intake for infants under 6 months of age having an elective procedure under anaesthesia are
a) Breast milk 2 hours before, clear fluids 1 hour before 3mls/kg
b) Breast milk 2 hours before, clear fluids 1 hour before 5mls/kg
c) Breast milk 3 hours before, clear fluids 1 hour before 3mls/kg
d) Breast milk 3 hours before, clear fluids 1 hour before 5mls/kg
e) Breast milk 4 hours before, clear fluids 1 hour before 3mls/kg
c) Breast milk 3 hours before, clear fluids 1 hour before 3mls/kg
PS07 - patient preparation and preanaesthetic consultation appendix 1 2023
Children up to 16 years:
Clear fluids of 3ml/kg up to 1 hour before.
<6 months
Formula 4 hours
Breast 3 hours
Clear 1 hour
> 6 months:
solids and formula wait 6 hours. Breast milk 4 hours.
Clear 1 hour
22.1 Complex regional pain syndrome is NOT characterised by
a. Vasomotor
b. Sudomotor
c. Pain distal to primary injury
d. Hypoaesthesia
e. Edema
Hypoasthesia - Veldman criteria
Pain distal to primary injury – Not mentioned specifically in Budapest criteria, but in Veldman
Hypoaesthesia = reduced sensation to pain
20.1 The anti-emetic action of aprepitant is via receptors for
A. Serotonin
B. Neurokinin-A
C. Dopamine
D. Substance P
E. Glycine
D. Substance P
Development of aprepitant, the first neurokinin-1 receptor antagonist for the prevention of chemotherapy-induced nausea and vomiting (2011)
https://www.ncbi.nlm.nih.gov/pubmed/21434941
Aprepitant acts centrally at NK-1 receptors in vomiting centres within the central nervous system to block their activation by substance P released as an unwanted consequence of chemotherapy.
21.2, 22.2 You are involved in the care of a two-year-old child who ingested a button battery within the last 4 hours. You should consider giving
a. milk,
b. sodium bicarbonate
c. Pantoprazole
d. sucralfate
Honey (or sucralfate) - 10 mL every 10 minutes (maximum 6 times) while awaiting surgical retrieval
Source QCH guidelines
21.2 The image below shows results from non inferiority trials. The trial labelled ‘B’ is best
described as
a) Non-inferiority is not demonstrated
b) Non-inferiority is demonstrated
c) Superiority is demonstrated
d) Inferiority is demonstrated
a) Non-inferiority is not demonstrated
Possible outcomes in a non-inferiority trial.
In A (blue), non-inferiority is demonstrated.
In B (green), non-inferiority is not demonstrated, and the trial is inconclusive.
In C (red), the new treatment is inferior.
23.1 Of the following drugs, the LEAST suitable for managing atrial arrhythmias in a patient with a left ventricular assist device is
A. Metoprolol
B. Amiodarone
C. Digoxin
D. Diltiazem
d) diltiazem
Nondihydropyridine calcium channel blockers should be used cautiously in patients with HFrEF because of their negative inotropic effects, and the role of these agents in LVAD recipients remains unclear
https://www.ahajournals.org/doi/10.1161/CIR.0000000000000673
Should also avoid sotolol
20.2 A 55 year old man with no past history of ischaemic heart disease is 3 days post total hip replacement surgery. He has an episode of chest pain that sounds ischaemic, began at rest and lasts thirty minutes before resolving fully. There are no ECG changes nor troponin rise. The diagnosis is
a. No diagnosis made
b. Unstable angina
c. STEMI
d. NSTEMI
e. MINS
b. Unstable angina
Not a Repeat, no Tropnin rise in this question making the answer unstable angina as opposed to NSTEMI
UTD:
Unstable angina (UA) and acute non-ST elevation myocardial infarction (NSTEMI) differ primarily in whether the ischemia is severe enough to cause sufficient myocardial damage to release detectable quantities of a marker of myocardial injury (troponins):
●UA is considered to be present in patients with ischemic symptoms suggestive of an ACS and no elevation in troponins, with or without electrocardiogram changes indicative of ischemia (eg, ST segment depression or transient elevation or new T wave inversion).
●NSTEMI is considered to be present in patients having the same manifestations as those in UA, but in whom an elevation in troponins is present.
MINS: Myocardial injury after non-cardiac surgery (up to 30 days post-op):
1. Elevated postop troponin
2. Resulting from myocardial ischaemia (i.e. no evidence of a non-ischaemic aetiology), not requiring an ischaemic feature (i.e. no chest pain, no ECG change)
VISION studies (Vascular Events in Noncardiac Surgery Patients Cohort Evaluation) demonstrated that severity of MINS strongly associated with 30-day mortality after NCS.
hs-cTnT
<20ng/L ~ 0.5% 30 day mortality
20-64ng/L ~3% 30 day mortality
65-999 ng/L ~9% 30 day mortality
>1000ng/L ~30% 30 day mortality
Whilst VISION trial identified MINS in at risk patients, the question now becomes what interventions are available to prevent this complication?
20.1 Best resolution US probe for median nerve visualisation:
d) 5-10mHz
e) 6-13mHz
High frequency probe at 90 degrees to the skin
- to best visualise superficial structures have the probe at 90 degrees to the skin with a high frequency transducer
it is best to use high-frequency transducers (up to 10–15 MHz range) to image superficial structures (such as for stellate ganglion blocks) and low-frequency transducers (typically 2–5 MHz) for imaging the lumbar neuraxial structures that are deep in most adults.
22.2 A test for a condition which has a prevalence of 1 in 1,000 has a sensitivity of 100% and a specificity of 90%. The probability of a patient who receives a positive result actually having the condition is
–50% was not an option
a. 1%
b. 10%
c. 90%
d. 100%
a. 1%
i.e. what is the positive predictive value (PPV) for this test
PPV= TP/ TP +FP
Negative Predictive Value = TN / TN + FN
Prevalence of 1/1000
Sensitivity of 100%
Specificity of 90%
Of patients that are disease positive in population of 1000
TP = 1
FN = 0
-> 100% sensitivity
Of patients that are disease negative in population of 1000
FP = 99
TN = 900
-> 90% Specificity
PPV= TP/ TP + FP
= 1/ 1 + 99
= 1/100
=1%
NPV= TN/ TN + FN
=900/ 900 + 0
= 1/1
= 100%
22.1 A patient is anaesthetised from the awake state to a state of surgical anaesthesia with propofol or a volatile anaesthetic. As the depth of anaesthesia increases, the patient’s electroencephalogram (EEG) will show oscillations that are of
Dominant EEG frequency decreases, and amplitude increases with increasing concentrations of anaesthetic. End result is burst suppression
https://academic.oup.com/bja/article/115/suppl_ 1/i27/234261
Figure 1 shows raw EEG waveforms during isoflurane anaesthesia.
During light anaesthesia:
-amplitude is shallow and frequency is high.
When a higher concentration is administered:
-amplitude deepens and EEG frequency slows.
During deep anaesthesia:
- a ‘burst and suppression’ pattern becomes apparent, characterized by extreme activity, represented by high-frequency, large-amplitude waves (bursts), alternating with flat traces (suppression).
- This pattern, excluding brain ischaemia or other factors, indicates that anaesthesia is too deep. Beyond this, flat traces become dominant and, eventually waveforms are no longer apparent.
During isoflurane, sevoflurane or propofol anaesthesia, this sequence of changes in pattern is almost identical.
The major difference in EEG between the volatile agents (isoflurane or sevoflurane) and propofol is apparent in power in the theta range.
During propofol anaesthesia, theta power remains low regardless of concentration, but during isoflurane or sevoflurane anaesthesia, it increases at surgical concentrations of anaesthesia.
23.1 A five-year-old child weighing 25 kg is to be strictly nil by mouth overnight following a laparotomy. The most appropriate fluid prescription is
a) 45ml/hr 0.9% NS 2.5% dextrose
b) 65ml/hr 0.9% NS 5% dextrose
c) 45ml/hr 0.45% saline with 2.5% dextrose
d) 65ml/hr 0.45% saline with 5% dextrose
e) 45ml/hr 0.9% NS 5% dextrose
e. 45ml/hr 0.9% NS 5% dextrose
REPEAT
2/3rd standard full maintenance as unwell
23.1 The nerve labelled with the arrow in the diagram is the (diagram of the brachial
plexus shown)
a. Musculocutaneous
b. Median
c. Radial
d. Ulnar
e. Axillary
a) muscolocutaneous
21.1 The 12 lead ECG shown (ECG with ST depression V1-V5, perhaps 1mm ste in lead 3) is most consistent with acute total occlusion of the
A. Posterior descending
B. RCA
C. LAD
D. OM
C. LAD
20.1 Dental damage risk to be determined in your department. 100 cases reviewed, zero cases of dental damage. What is the 95% confidence interval?
a) 0/100
b) 1/100
c) 3/100
d) 5/100
e) 9/100
Answer: 3/100
In statistical analysis, the rule of three states that if a certain event did not occur in a sample with n subjects, the interval from 0 to 3/n is a 95% confidence interval for the rate of occurrences in the population. When n is greater than 30, this is a good approximation of results from more sensitive tests. For example, a pain-relief drug is tested on 1500 human subjects, and no adverse event is recorded. From the rule of three, it can be concluded with 95% confidence that fewer than 1 person in 500 (or 3/1500) will experience an adverse event. By symmetry, for only successes, the 95% confidence interval is [1−3/n,1].
The rule is useful in the interpretation of clinical trials generally, particularly in phase II and phase III where often there are limitations in duration or statistical power. The rule of three applies well beyond medical research, to any trial done n times. If 300 parachutes are randomly tested and all open successfully, then it is concluded with 95% confidence that fewer than 1 in 100 parachutes with the same characteristics (3/300) will fail.
21.1 The main advantage of using norepinephrine (noradrenaline) over phenylephrine for the prevention of
hypotension as a result of spinal anaesthesia for elective caesarean section is
A. Better APGAR
B. Better foetal acid/base
C. Less nausea/vomiting
D. Less maternal bradycardia
less maternal bradycardia
21.1 The muscle or muscle group with the greatest sensitivity to the action of non-depolarising neuromuscular blocking agents is/are the
a. Abdominal muscles
b. Adductor pollicis
c. Pharyngeal muscles
d. Diaphragm
c. Pharyngeal muscles
Millers Anaesthesia:
Reference artyicle from Millers: https://pubs.asahq.org/anesthesiology/article/92/4/977/710/The-Incidence-and-Mechanisms-of-Pharyngeal-and
An adductor pollicis TOF ratio of 0.90 or less was associated with impaired pharyngeal function and airway protection, resulting in a four- to fivefold increase in the incidence of pharyngeal dysfunction causing misdirected swallowing. Moreover, pharyngeal function and airway protection may be impaired, even if the adductor pollicis muscle has recovered to a TOF ratio of more than 0.90.
22.1 Complications of severe anorexia nervosa (body weight < 40% ideal) include all of the following EXCEPT
a. HypoK
b. Cl abnormality
c. Delayed gastric emptying
d. Hypercalcaemia
e. Cardiomyopathy
Hypercalcaemia
21.2 The nerve labelled by the arrow in the diagram below is the
a) Obturator
b) Accessory obturator
c) Genitofemoral
d) Ilioinguinal
e) Iliohypogastric
c) Genitofemoral
22.1 A 45-year-old man presents on the day of surgery for an elective inguinal hernia repair. He is well
but is noted to be mildly jaundiced. He takes simvastatin for hyperlipidaemia and has no other medical history. He consumes about three standard drinks of alcohol per day and does not smoke. He briefly experimented with illicit drugs more than ten years ago. His laboratory results show: (supplied) The most likely diagnosis is
Normal electrolytes
ALP 85 N
ALT 31 N
AST 31 N
GGT 15 N
Urea 10 [4-9]
Creatinine 103 N
Total protein 74 N
Albumin 40 N
BSL 4.2 N
Bilirubin 29 [0-20]
Conjugated 5
Unconjugated 24
A. Fatty liver
B. Hepatitis
C. Cholestasis
D. Gilbert syndrome
E. Drug induced
Gilberts
Gilbert’s syndrome is a benign genetic condition that commonly presents as incidental
hyperbilirubinaemia or painless jaundice.
It is relatively common with a population frequency of approximately 2–10%.
Gilbert’s syndrome is caused by defective bilirubin clearance by the hepatic conjugating enzyme UDP-glucuronosyltransferase
https://www.rcpa.edu.au/getattachment/8b9a8acf-f7f5-4088-951c-3f65f0c2f8fe/Interpreting-liver-function-tests.aspx
20.2 A 46-year old man collapses unexpectedly and fractures his femur. He is booked for acute theatre. A pre-operative electrocardiogram is performed. Of the following, the most appropriate peri-operative medical management is (ECG shown)
ECG = WPW
a) Flecainide
b) Aspirin
c) Digoxin
d) Magnesium
e) Verapamil
a) Flecanide
WPW ECG = short PR, wide QRS, delta wave at start of QRS
If WPW, need to prolong refractor period of accessory pathway with agents such as procainamide/flecainide/amiodarone/sotalol.
Avoid verapamil (increases ventricular rate).
Avoid beta blockers (don’t affect accessory pathway).
https://litfl.com/wolff-parkinson-white-syndrome-ccc/
22.2 Predictors of difficult sedation (agitation or inability to complete the procedure) of patients undergoing gastroscopy do NOT include
Unknown options but…
Factors associated WITH difficulty during Gastroscopy were younger age, procedure indication, male sex, presence of a trainee, psychiatric history and benzodiazepine and opioid use. Factors associated with difficulty during COLONOSCOPY were younger age, female sex, BMI <25, procedure indication, tobacco, benzodiazepine, opioid and other psychoactive medication use
21.2 The condition in which volatile anaesthesia is least appropriate is
a) Multiple sclerosis
b) Myasthenia gravis
c) Lambert-Eaton syndrome
d) Guillain-Barre syndrome
e) Muscular dystrophy
e) Muscular dystrophy
- rhabdomyolysis risk if given to patients with Duchenne or Becker’s muscular dystrophy
- volatiles safe in all above, and also safe in patient’s with myotonic dystrophy
Malignant hyperthermia
- high mortality uncoupling regulation of RyR1 to SR
Duschenne muscular dystrophy
- fatal rhabdo (hyperkalaemia)
23.1 A patient with severe abdominal trauma develops acute respiratory distress syndrome. A diagnosis of abdominal compartment syndrome is confirmed if the patient also has a sustained intraabdominal pressure greater than
A. 10mmHg
B. 16mmHg
C. 20mmHg
D. 24mmHg
c) 20mmHg
Intra-abdominal hypertension is defined as a sustained intra-abdominal pressure of >12 mm Hg, and abdominal compartment syndrome occurs at a pressure >20 mm Hg in association with new organ dysfunction.
Intra-abdominal hypertension is graded as follows: Grade 1=12–15 mm Hg; Grade 2=16–20 mm Hg; Grade 3=21–25 mm Hg; and Grade 4 >25 mm Hg.
https://academic.oup.com/bjaed/article/12/3/110/258792
21.2 An electrocardiogram (ECG) abnormality which is NOT usually associated with severe anorexia nervosa is
a) Sinus tachycardia
b) Wandering atrial pacemakers
c) ST depression
d) T wave inversion
e) Prolonged QT
a) Sinus tachycardia
BJA: Anorexia nervosa: perioperative implications
https://academic.oup.com/bjaed/article/9/2/61/299563
Cardiovascular
Typically anorexic patients are hypotensive and bradycardic. These physiological markers may be used as indications for hospitalization.
Bradycardia reflects the decrease in basal metabolic rate that arises as an adaptive response to starvation. Although patients are usually in sinus rhythm, electrocardiographic abnormalities are common and may be found in >80% of strict dieters. These include: atrioventricular block, ST depression, T wave inversion, and QT prolongation.
QT prolongation may be caused by hypocalcaemia, hypomagnesaemia, drugs, or directly by starvation itself. Electrolyte disturbances have a significant causal role in ECG abnormalities.
Other factors, for example, atypical antipsychotics, may also contribute.
Associated arrhythmias include: sinus arrest, wandering atrial pacemakers, nodal escape beats, supraventricular tachycardia, and ventricular tachycardia.
The reported incidence of arrhythmias under anaesthesia is 16–62%.
With respect to myocardial contractility, left ventricular function has been demonstrated to be impaired in a proportion of patients. Echocardiographic studies have also demonstrated a higher incidence of mitral valve prolapse in anorexic patients. The reasons for this are not entirely clear. It is postulated that the loss of left ventricular volume and mass leads to abnormal mitral valve motion.
In addition to starvation-induced myocardial impairment, the myocardium may be specifically damaged by pharmacological agents.
For example, emetogenic ipecac syrup is directly cardiomyotoxic and produces inflammatory changes and myocardial fibre degeneration when used long term.
Rarely, antipsychotic drugs, for example, olanzapine, may cause cardiomyopathy.
Myocardial impairment can be caused by hypophosphataemia which also reduces the threshold for arrhythmias.
Compromised myocardial function requires judicious use of fluids perioperatively as there is an increased risk of congestive cardiac failure. Echocardiography along with invasive perioperative monitoring (central venous catheter) should be considered to prevent fluid overload.
23.1 Features of hypocalcaemia include all of the following EXCEPT
a. Polydipsia
b. Circumoral tingling
c. Long QTc
d. Laryngospasm
e. Hallucinations
a) polydipsia
Hypocalcemia varies from a mild asymptomatic biochemical abnormality to a life-threatening disorder. Acute hypocalcemia can lead to paresthesia, tetany, and seizures (characteristic physical signs may be observed, including Chvostek sign, which is poorly sensitive and specific of hypocalcemia, and Trousseau sign).
https://bestpractice.bmj.com/topics/en-us/160
20.1Of the following, the LEAST appropriate treatment in the management of severe acute respiratory distress syndrome (ARDS) is
a) High PEEP
b) Recruitment maneuvers
c) Neuromuscular blockade
d) Prone
e) Negative fluid balance
Muscle relaxation or Recruitment maneuvers
A, D and E are all appropriate for ARDS.
Muscle relaxation and lung recruitment are controversial.
Best answer is probably A muscle relaxation (not recommended unless there is dyssynchrony).
Recruitment and higher PEEP are conditional.
UP TO DATE: RE: Muscule relaxation: “ Until a clear benefit is demonstrated, we suggest not routinely administering NMBs to patients with moderate to severe ARDS, unless other indications are present (eg, severe ventilator dyssynchrony, particularly if it leads to double triggering, or unwanted motor movement refractory to ventilator adjustment and sedation). “
Recruitment manoeuvres – no positive influence on survival.
https://derangedphysiology.com/main/required-reading/respiratory-medicine-and-ventilation/Chapter%20512/ventilation-strategies-ards
Does this strategy improve survival? Probably not, according to this Cochrane review. In fact, in the ART trial (Cavalcanti et al, 2017) they probably caused harm. But, they can improve oxygenation temporarily.
One can see the benefit of recruitment manoeuvres in patients who have accidentally become disconnected from the ventilator.
The 2017 ATS guidelines were published in May of 20117, whereas the ART trial came out in September that year, and so the ATS guidelines still recommend recruitment manoeuvres whereas the rest of the world probably does not.
In fact, in their answer to Question 8 from the first paper of 2018 the college remarked that if any trainee who confesses to the routine use of recruitment manouvres, “they were should be marked down” by the examiners.
22.1 A man has symptomatic carbon monoxide poisoning. His pulse oximetry (SpO2) and arterial blood gas (PaO2) would be expected to show
a. Normal SpO2, Normal PaO2
b. Normal SpO2, reduced PaO2
c. Reduced SpO2, normal PaO2
d. Reduced SpO2, reduced PaO2
a. Normal SpO2, Normal PaO2
ABG
HbCO (elevated levels are significant, but low levels do not rule out exposure) lactate (tissue hypoxia) PaO2 should be normal, SpO2 only accurate if measured (not calculated from PaO2) MetHb (exclude)
https://litfl.com/carbon-monoxide-poisoning/
21.2 A 69-year-old woman has a recent onset of dyspnoea and undergoes a right heart catheterisation, with results displayed below. Her pulmonary capillary wedge pressure is 10 mmHg. The most likely diagnosis is
a) Mitral stenosis
b) Mitral regurgitation
c) Aortic stenosis
d) Pulmonary embolism
e) COPD
d) Pulmonary embolism
- fits with history of acute dyspnoea
- PCWP normal, therefore precapillary PH
- thus left heart disease unlikely to be the cause of elevated RVSP (clinical group 2)
- COPD possible if cor pulmonale, but this is an unlikely cause of acute dyspnoea given history
Normal PCWP excludes left heart disease as cause of pulmonary HTN (so not MR, MS or AS). The causes of pre-capillary pulm HTN are pulmonary arterial hypertension, pulmHTN secondary to lung disease, chronic thromboembolic pulmonaryHTN, pulmHTN with unclear/multifactorial mechanisms.
Normal pulmonary capillary wedge pressure = 8- 12mmHg
Normal PASP: 15-25mmHg
Normal PADP: 8-15mmHg
Pulmonary HTN is mPAP ≥25mmHg at rest. mPAP = PADP + (PASP-PADP/3)
mPAP in this image is 43 mmHg Transpulmonary gradient = mPAP – PAWP
20B
20B
23.1 A patient with an acute subarachnoid haemorrhage arrives in the emergency department. Her Glasgow Coma Scale score is 10 and she has no motor deficit. A CT brain shows diffuse subarachnoid haemorrhage with no localised areas of blood greater than 1mm thick, and no intracerebral or intraventricular blood. Her World Federation of Neurosurgical Societies (WFNS) grade of subarachnoid haemorrhage is
A. 1
B. 2
C. 3
D. 4
E. 5
D. 4
- alternatively her Fisher score is: grade 2 (diffuse thin (<1 mm) SAH, no clots; which estimates an incidence of symptomatic vasospasm of 25%)
Note the new modified Fischer scale.
G0 No SAH or IVH (0%)
G1 Focal or diffuse thin SAH but no IVH (6-24%)
G2 Focal or diffuse thin SAH with IVH (15-33%)
G3 Thick SAH no IVH (33-35%)
G4 Thicc SAH with IVH (34-40%)
The main differences between the Fisher scale and modified Fisher scale are:
1) Fisher scale, no SAH is grade 1, but 0 in modified Fisher scale
2) Fisher scale, thin SAH & no IVH is grade 2, but 1 in modified Fisher scale
3) Fisher scale, thick SAH with no IVH is grade 3 and the same 3 in modified Fisher scale
4) Fisher scale, any IVH is grade 4, irrespective of the presence of SAH but in modified Fisher scale it is either 2 if thin or no SAH, or grade 4 if thick SAH
Modified Fisher Scale:
grade 0
- no subarachnoid hemorrhage (SAH)
- no intraventricular hemorrhage (IVH)
- incidence of symptomatic vasospasm: 0% 3
grade 1
- focal or diffuse, thin SAH
- no IVH
- the incidence of symptomatic vasospasm: 24%
grade 2
- focal or diffuse, thin SAH
- IVH present
- the incidence of symptomatic vasospasm: 33%
grade 3
- thick SAH
- no IVH
- the incidence of symptomatic vasospasm: 33%
grade 4
- thick SAH
- IVH present
- the incidence of symptomatic vasospasm: 40%
Note: the original study did not include a specified measurement or criteria to define thick vs thin hemorrhage.
REPEAT
20.1 The radial artery pressure trace shown below is from a patient who has an intra-aortic balloon pump in situ. The device has been switched to 1:2 augmentation to assess the timing. The trace shows an augmented beat followed by an un-augmented beat. With respect to the augmentation, the trace shows
a. Correct timing
b. Early inflation
c. Late inflation
d. Early deflation
e. Late deflation
Bonus question
b. Early inflation
Waveform features:
> Diastolic augmentation (peak B) encroaches on the peak corresponding to unassisted systole (peak A) – the two peaks have merged and are barely distinguishable.
> There is no ‘sharp V’ or dicrotic notch between peaks A and B.
Early IAB inflation may result in:
> Premature closure of the aortic valve and possible aortic regurgitation, thus impairing left ventricular emptying. There may be an increase in LVEDV, LVEDP and PCWP.
> Increased left ventricular wall stress (afterload) and increased myocardial oxygen consumption will occur.
how to correct:
Delay the onset of IAB inflation, so that it inflates at the dicrotic notch resulting in a ‘sharp V’ (see the normal pressure waveform).
21.1 A 30-year-old woman, gravida 2, parity 1, undergoes an elective lower segment caesarean section for breech presentation. The international consensus statement on the use of uterotonic agents recommends that the first line uterotonic management is
a) 1unit
b) 1 unit followed by infusion 2.5-7.5 Units/hr
c) 3 units
d) 3 units followed by infusion
Bolus 1 IU oxytocin; start oxytocin infusion at 2.5–7.5IU.h (0.04–0.125 IU.min)
EmLSCS; 3 IU oxytocinover≥30 s; start oxytocininfusion at 7.5–15 IU.h (0.125–0.25 IU.min).
https://associationofanaesthetists-publications.onlinelibrary.wiley.com/doi/10.1111/anae.14757
23.1 A risk factor for the development of chronic postsurgical pain is having
a. Age >65
b. Male
c. Pain at site 1 month prior to surgery
d. Higher SES
c. Pain at site 1 month prior to surgery
Pain itself is a risk factor: the strongest predictors of CPSP are chronic preoperative pain and the severity of acute postoperative pain
https://www.ncbi.nlm.nih.gov/pmc/articles/PMC5741327/#:~:text=Pain%20its
22.1 The first-line drug recommended by both the Australian Resuscitation Council and the New Zealand Resuscitation Council to treat severe cyanide poisoning is
a. Methylene blue
b. Hydroxycobalamine
c. Sodium thiosulfate
hydroxycobalamin
During a routine preoperative examination of a patient’s heart, you note exaggerated splitting of the second heart sound with inspiration. This is characteristically heard in
A. Aortic Reguritation
B. HOCM
C. Left bundle branch block
D. Mitral Stenosis
E. Pulmonary Stenosis
E. Pulmonary Stenosis
DERANGED PHYSIOLOGY:
Splitting of the first heart sound
Right bundle branch block can produce a split first heart sound - because the contraction of the right ventricle is delayed- the conduction occurs via the left ventricle rather than the bundle of His- and thefore the closure of the tricuspid valve occurs after a substantial delay.
Atrial septal defect can result in a fixed split of the first heart sound
Splitting of the second heart sound
It is normal for this sound to be split. The high pressure in the systemic circulation slams the aortic valve shut rather abruptly, almost angrily. In contrast, low pressure of the pulmonary circulation tends to close the pulmonary valve gently, and therefore the pulmonary component of the second heart sound (P2) is usually delayed by about 20-30 milliseconds.
It is also normal for increased right ventricular filling to cause a widening of the split. The more blood in the RV, the longer it takes to eject, and therefore the greater the delay until pulmonary valve closure.
n the spontaneously breathing patient, the delay is greatest during inspiration. Naturally, in the patient ventilated with positive pressure the delay is greatest during expiration (positive pressure being a barrier to diastolic filling).
Increased normal splitting of S2
Anything that delays the end of right ventricular systole can cause this sort of picture.
Right bundle branch block - the delay in conduction via the left ventricle causes a delay in right ventricular contraction, and therefore a delay in pulmonary valve closure. The S1 will also be split.
Ventricular septal defect - because the right ventricle receives a large volume load directly from the left ventricle, and therefore takes longer to complete its systolic contraction.
Pulmonary valve stenosis - because the right ventricle takes longer to empty though a narrowed valve
Mitral regurgitation- not because right ventricular contraction is delayed, but because left ventricular contraction is shortened (as the LV empties in both the aortic and the atrial directuion, systole is over very quickly).
Fixed splitting of S2
Atrial septal defect - the atria, joined by a gaping hole in their seput, act as one atrium. The result is a reasonably equal distribution in volume betweent the right and left atrium. This way, both sides of the circulation share the same diastolic filling pressure. Dragging more volume into the right atrium with respiratory activity will not cause an inequality of ventricular filling (between the right and left ventricles) because the venous return will be “shared”.
Reversed splitting of S2
In this situation, P2 occurs before A2, and splitting widens during expiration (or inspiration in the mechanically ventilated patient). This only happens if the conduction to the left ventricle is delayed, or if the left ventricle is massively volume overload (and the right ventricle is not).
Left bundle branch block - the left ventricle depolarises after the right ventricle, and A2 is delayed
Aortic stenosis - the left ventricle empties slowly though a narrow valve
Large patent ductus arteriosus - the left ventricle receives a backflow of blood from the aorta, which causes it to become volume-overloaded
21.1 A patient with C6 tetraplegia is undergoing removal of bladder stones under general anaesthesia. The blood pressure rises to 166/88 mmHg. The appropriate response is to
a. Clonidine
b. Hydralazine
c. Decompress the bladder
d. Fentanyl
e. Deepen your anaesthetic
decompress the bladder
Autonomic Dysreflexia:
- medical emergency characterised by severe hypertension,
- brought on by stimulation below the level of the lesion
Factors affecting the development of ADR:
1. Level of spinal injury
2. Duration of injury
3. Whether injury is complete or incomplete
Pathology:
Stimuli arise from caudal roots below the level of the lesion leading to uncontrolled sympathetic activation below the level of the lesion
○ 80% being due to bladder distension
○ Other triggers include
§ bowel distension
§ acute abdo pathology
§ activation of pain fibres
§ sexual activity
§ uterine contractions
21.1 A 100 kg 32-year-old male presents two hours after suffering a 30% total body surface area electrical burn. He has had no resuscitation fluids. The infusion rate of isotonic crystalloid resuscitation fluid required for this man for the next six hours is
a. 500 ml/hr
b. 750 ml/hr
c. 1000 ml/hr
d. 1200 ml/hr
c. 1000 ml/hr
30% x 100kg x 4ml= 12000ml
50% in first 8hrs = 6000ml
pt presents 2hrs late = 6000ml/6hrs = 1000ml/hr
or
30% x 100kg x 3ml= 9000ml
50% in 1st 8 hrs= 4500ml
pt presents 2hrs late= 4500ml/6hrs= 750ml/hr
750mls/hr
EMSB recommends 3ml/kg, BJA recommends 4ml/kg
-> go with 4ml/kg because electrical burn and more likely to require increased fluids anyway
23.1 According to the categorisation system used in Australia and New Zealand for prescribing medicines safely in pregnancy, category X denotes drugs which are
a. Drugs that absolutely must not be used for pregnancy. (absolute contraindication)
b. Untested drugs in pregnancy
c. Drugs safe in pregnancy
a. Drugs that absolutely must not be used for pregnancy. (absolute contraindication)
https://www.tga.gov.au/australian-categorisation-system-prescribing-medicines-pregnancy
21.1 Considering emergency front-of-neck airway access, the major blood vessel that is most likely to lie anterior to the trachea above the sternal notch is the
a) Brachiocephalic artery
b) Brachiocephalic Vein
c) Superior thyroid artery
d) Inferior thyroid artery
e) Carotid artery
a) Brachiocephalic artery
Major vessels, most commonly the brachiocephalic artery, traverse the anterior tracheal wall in up to 53% patients at the suprasternal notch
23.1 Self-report of pain in children is usually possible by the age of
a. 2 yo
b. 4 yo
c. 6 yo
d. 8 yo
b) 4yo
4 yo = wong baker faces score 3-18.
8 yo = Visual analogue scale.
https://www.rch.org.au/rchcpg/hospital_clinical_guideline_index/Pain_assessment_and_measur ement/
APMSE 5 also
22.1 Preperitoneal pelvic packing is a surgical treatment of haemorrhage from a/an
a) pelvic fracture
Haemodynamically unstable pelvic fracture
22.1 Following the initial subarachnoid haemorrhage from a ruptured aneurysm, the patient is at greatest risk of rebleeding during the following
a. 1-3 days
b. 3-5 days
c. 5-7 days
d. 7-10 days
a. 1-3 days
22.2 A child with well controlled dysrhythmias has an ASA (American Society of Anesthesiologists) Physical Status classification of at least
a) I
b) II
c) III
d) IV
e) V
B II
ASA II Paediatric examples: Asymptomatic congenital cardiac disease, well controlled dysrhythmias, asthma without exacerbation, well controlled epilepsy, non-insulin dependent diabetes mellitus, abnormal BMI percentile for age, mild/moderate OSA, oncologic state in remission, autism with mild limitations
20.1 Interventions that reduce the risk of agitation following electroconvulsive therapy include all of the following EXCEPT
a Low dose of propofol following the seizure
b Low dose of midazolam following the seizure
c Premedication with olanzapine
d Premedication with dexmedetomidine
e Induction with remifentanil
e) Remifentanil bolus
Induction agents:
Propofol:
-0.75-2.5mg/kg
- shortest seizure duration
- improved CVS stability, less PONV, quicker emergence
- pain on injection
Etomidate
- 0.15-0.3mg/kg
- Prolonged seizure activity, may reduce seizure threshold
- Useful in resistant seizures
- Hyperdynamic response more pronounced compared with propofol, increased PONV, longer emergence time
Methohexital
- 0.5-1.5 mg/kg
- “gold standard” for ECT seizure quality
- long history of use
- reduced availability; lack of familiarity with sue
Thiopental
- 2-5mg/kg
- Seizure duration reduced but better than propofol
- need to reconstitute, has increased dysrhythmias
Ketamine
- 0.7-2.8mg/kg
- unclear effect on seizures: reduced and prolongesd in different studies
- usefull in resistant seizures
- emergence phenomena, reduced CVS stability and increases ICP
Sevoflurane
-6-8% inspired concentration; MAC1-2
- reduced seizure duration compared to methohexital
- useful if difficult IV access, reduces uterine contractions in pregnancy
- extra equipment needed; more time consuming
Induction agents in the descending order of CMRO2 reducing ability:
Propofol > sevoflurane > thiopental and methohexital > etomidate > ketamine.
Induction agents in the descending order of CBF and ICP reducing ability:
Propofol > thiopental and methohexital > etomidate > ketamine.
Induction agents in the descending order of emergence time:
Ketamine > etomidate > barbiturates > propofol > sevoflurane.
Emergence time is the time from drug administration for general anaesthesia till eye opening or following commands. The differences in emergence time among induction agents suitable for ECT are small, and these small variations in emergence should not govern drug choice.
Induction agents in descending order of seizure threshold reducing property are:
Etomidate > ketamine > methohexital > thiopental > propofol.
Opioid:
- Alfentanil (10-20mcg/kg) or remifentanil (1mcg/kg) can be used along with the induction agent to increase the seizure duration and reduce haemodynamic response.
- It is unclear if the effect on seizure duration is an inherent effect of the opioid or as a result of its dose sparing effect.
NMB:
-Neuromuscular blocking agents reduce muscular convulsions and decrease the risk of serious injury.
- Sux at 0.5mg/kg most commonly used, larger doses upto 1.5mg/kg nay be required
Adjuncts:
- used to reduce dose of induction agent, or mitigate cardiovascular response to ECT in high risk patients
- To treat adverse PNS effects
Glycopyrolate: - superior anti-sialogogue effect
- no adverse CNS effects
- less post ECT tachycardia
Atropine - routine atropine pre-medicattion is not recommended due to adverse effects of increased myocardial work and O2 demand
To treat Adverse SNS effects:
- B-blockers: atenolol (pre-ECT) or labetalol and esmolol (intra-ECT), this may reduce seizure duration
-CCB: sublingual nifedapine and IV nicardipine for Htn but may reduce seizure duration
- a-2 agonists: Dexmedetomidine blunts the hyperdynamic rsponse as does GTN and should be considered in patients at high risk of ischaemia
- Dexmedetomidine reduces the incidence of post-ECT adverse effects such as headache, agitation, postictal delirium, or pain associated with propofol injection
-IV lignocaine is not effective
Emergence agitation:
- Small doses of midazolam may be useful if simple measures such as a secluded, calm recovery environment do not help
-However, we avoid administration of any benzodiazepine such as midazolam before performing an ECT procedure, due to known anticonvulsant properties that would make seizure induction more difficult
- In patients with a history of severe postictal agitation, intravenous (IV) benzodiazepines or propofol may be administered at the end of the seizure Dexmedetomidine may be useful in the treatment of refractory cases
22.2 A 45-year-old man is ventilated in the intensive care unit and is in a critical state. His pulmonary artery wedge pressure is 26 mmHg, cardiac index is 1.7 L/minute/m2 and his PaO2/FiO2 ratio is 200 mmHg. A decision is made to place him on extracorporeal membrane oxygenation. The most appropriate mode is
a) VV ECMO
b) VA ECMO
c) Atrio-aorto ECMO
d) Ventriculo-atrial ECMO
b) VA ECMO
PaO2/FiO2 ratio
Mild: 200-300 = mortality 27%
Moderate = 100-200 mortality 32%
Severe < 100 = Mortality 45%
Cardiac Index
Normal: 2.5-4.2l/min
PAWP:
Normal 4-12mmHg
CI is low, PaO2/FiO2 ratio is mild, PAWP is high
22.1 The sensory innervation to the larynx above the vocal cords is provided by the
a) External SLN
b) Internal SLN
c) RLN
b) Internal SLN
23.1 A third heart sound at the apex may be heard in
a) pulmonary stenosis
b) pulmonary hypertension
c) pericarditis
d) pregnancy
d. pregnancy
22.2 A five-month-old child is to undergo routine elective morning surgery. Current ANZCA guidelines advise minimum fasting intervals prior to anaesthesia of
A. 4 hours for breast milk, 2 hours clear fluids
B. 4 hours for formula, 1 hour clear fluids
C. 3 hours for breast milk, 1 hour for clear fluids
D. 6 hours for formula, 2 hours clear fluids
E. 8 hours for solids, 4 hours for all fluids
C 3 hours for breast milk, 1 hour for clear fluids
also
B. 4 hours for formula, 1 hour clear fluids
0-6mo 4/3/1
Children > 6mo 6/4/1
Clear fluids 3mL/kg max
22.2 The most likely side effect observed in the post anaesthetic care unit after the use of dexmedetomidine is
a. Bradycardia
b. hypotension
c. shivering
d. cough
e. sedation
b. hypotension
The use of dexmedetomidine did not increase the duration of PACU LOS but was associated with reduced emergence agitation, cough, pain, postoperative nausea and vomiting, and shivering in PACU. There was an increased incidence of hypotension but not residual sedation or bradycardia in PACU.
https://pubmed.ncbi.nlm.nih.gov/35085107/#:~:text=Conclusions%3A%20The%20use%20of%20dexmedetomidine,sedation%20or%20bradycardia%20in%20PACU
22.2 The nerve labelled by the arrow marked F in the diagram is the
- Ulnar Nerve
- Axillary Nerve
- Median Nerve
- Medial Cutaneous nerve of the forearm
- Long Thoracic Nerve
- Dorsal Scapular Nerve
- Radial Nerve
- Suprascapular nerve
- Musculocutaneous Nerve
- Axillary Nerve
20.1 Perioperative hypothermia down to 35degrees - effect on bleeding:
a) More bleeding with normal INR and APTT
b) More bleeding with normal INR and raised APTT
c) More bleeding with raised INR and normal APTT
d) Unchanged bleeding and normal INR and APTT
e) Unchanged bleeding and elevated INR and APTT
More bleeding with normal INR and APPT
https://academic.oup.com/bja/article/117/suppl_3/iii18/2664400
Bleeding observed at reduced temperatures (33 – 37 °C) often occurs because of defects in platelet adhesion, while at temperatures below 33 °C, both reduced platelet function and coagulation enzyme activity contribute
Also lab INR and APTT are not temperature corrected
23.1 The success rate of stopping smoking before surgery is NOT improved by
a) Bupropion
b) Clonidine
c) Nortroptyline
d) Varencicline
e) SSRI
E - SSRIs
ANZCA PG12 Background Paper
20.1 What is the level below which we need to replace fibrinogen in a pregnant patient with a PPH
A. <1 g/L
B. <1.5 g/L
C. <2 g/L
D. <2.5 g/L
E. <3 g/L
<2g/L
22.2 The nerve labelled by the arrow marked E in the diagram is the
- Ulnar Nerve
- Axillary Nerve
- Median Nerve
- Medial Cutaneous nerve of the forearm
- Long Thoracic Nerve
- Dorsal Scapular Nerve
- Radial Nerve
- Suprascapular nerve
- Musculocutaneous Nerve
- Musculocutaneous Nerve
21.2 A patient presents for a trans-urethral resection of the prostate (TURP). He had a single drug-eluting coronary stent for angina pectoris inserted six months ago and is taking clopidogrel and aspirin. The most appropriate preoperative management of his medications is to
a) Cease aspirin, continue clopidogrel
b) Cease aspirin for 10 days, cease clopidogrel for 5 days
c) Cease clopidogrel for 5 days, continue aspirin
d) Cease clopidogrel for 10 days, continue aspirin
e) Continue both aspirin and clopidogrel
c) Cease clopidogrel for 5 days, continue aspirin
- prostatic surgery, the risk of major bleeding may be greater than the risk of stent thrombosis
- For clopidogrel, we stop five days before surgery
- Clopidogrel, if stopped, should be restarted with a loading dose of 300 mg as soon as possible after surgery, perhaps later in the day if postoperative bleeding has stopped. Some experts recommend a higher loading dose of 600 mg to decrease time to effectiveness in the higher-risk postoperative setting
- suggest that surgery be performed in centers with 24-hour interventional cardiology coverage
UP TO DATE: Noncardiac surgery after PCI
Nonemergency noncardiac surgery — For patients who have undergone previous stenting with either BMS or DES and who will need cessation of one or both antiplatelet agents, we prefer to defer planned nonemergency, nonurgent noncardiac surgery until at least six months after stent implantation. The risks of noncardiac surgery before six months are increased after both BMS and DES.
For patients whose surgery requires cessation of one or both antiplatelet agents and cannot wait six months, and where the risks of delaying surgery outweigh the benefits, our recommended minimal duration of DAPT is four to six weeks, depending on the urgency of surgery and risk of thrombotic complication. This is based in part on evidence suggesting that the increased risk of MI and cardiac death is highest within the first month after stent placement and no clear difference in risk between BMS and DES. Although we prefer to wait at least six weeks when possible, in patients for whom earlier surgery is in their best interest after weighing risks and benefits, we sometimes refer patients as early as four weeks after stent placement.
The proinflammatory and prothrombotic risks of surgery may increase the baseline risk of stent thrombosis even in the presence of DAPT and regardless of stent type during this early period after stenting. We believe this risk to be higher prior to the minimum duration of DAPT recommended above, but the final decision to continue or discontinue antiplatelet therapy in the perioperative period should be made only after an informed discussion among the surgeon, managing cardiologist (and other health care providers), and patient has taken place. In many cases, DAPT can be continued in the perioperative period, although for some surgeries, such as neurosurgery, posterior eye surgery, or prostatic surgery, the risk of major bleeding may be greater than the risk of stent thrombosis.
In these patients who undergo noncardiac surgery before the recommended minimum duration of DAPT, a platelet P2Y12 receptor blocker should be discontinued for as brief a period as possible. Aspirin should be continued through the perioperative period, since the risk of stent thrombosis is further increased with the cessation of both aspirin and clopidogrel and surgery can usually be safely performed on aspirin. The rationale to continue aspirin comes in part from the POISE-2 trial (PCI subgroup analysis), which is discussed separately. However, as many neurosurgical patients, for whom bleeding might be life threatening or lead to severe adverse outcomes, were not enrolled in POISE-2, the optimal strategy is not known.
●Minor surgical and dental procedures usually do not require cessation of antiplatelet therapy.
●With regard to stopping P2Y12 inhibitor prior to noncardiac surgery, we generally follow recommendations found in the manufacturer’s package insert for each drug.
- For clopidogrel, we stop five days before surgery; that is, the last dose is taken on the sixth day before surgery.
- For prasugrel, we stop seven days before surgery.
- For ticagrelor, we stop three to five days before surgery.
- Some experts are willing to recommend shorter discontinuation periods for procedures less likely to be associated with major bleeding.
●Clopidogrel, if stopped, should be restarted with a loading dose of 300 mg as soon as possible after surgery, perhaps later in the day if postoperative bleeding has stopped. Some experts recommend a higher loading dose of 600 mg to decrease time to effectiveness in the higher-risk postoperative setting.
●We suggest that surgery be performed in centers with 24-hour interventional cardiology coverage
23.1 A patient presents for a transurethral resection of the prostate (TURP). He had a single drug-eluting coronary stent for angina pectoris inserted six months ago and is taking clopidogrel and aspirin. The most appropriate preoperative management of his medications is to
a) Cease aspirin, continue clopidogrel
b) Cease aspirin for 10 days, cease clopidogrel for 5 days
c) Cease clopidogrel for 5 days, continue aspirin
d) Cease clopidogrel for 10 days, continue aspirin
e) Continue both aspirin and clopidogrel
C) Cease clopidogrel for 5 days, continue aspirin
WFSA update document
https://resources.wfsahq.org/wp-content/uploads/uia29-Perioperative-management-of-patients-with-coronary-stents-for-non-cardiac-surgery.pdf
Dual antiplatelet therapy should be continued in all patients with coronary stents presenting for surgery.
However, if there is a high risk of surgical bleeding then clopidogrel should be stopped 5-7 days before surgery and monotherapy with aspirin should be continued.
Clopidogrel should be restarted as soon as possible post surgery. Cessation of aspirin therapy may be considered during intracranial surgery and transuretheral resection of prostrate as these procedures are associated with an increased risk of bleeding, but only after contemplating the risk-benefit ratio.
2014 AHA/ACC guidelines on perioperative medicine don’t give a firm answer except: > 180 days since insertion = proceed (Level II b evidence)
22.1 The manufacturer’s instructions for use of the i-gel supraglottic airway device recommend a minimum patient weight of
a. 1 kg
b. 2 kg
c. 3 kg
d. 5 kg
e. 10 kg
b. 2 kg
22.2 A 50-year-old man has the following pulmonary function test result: (provided). The most consistent diagnosis is
FEV1 68%, FVC 68%, DLCO 91%
a. Pulmonary hypertension
b. pulmonary fibrosis
c. myasthenia gravis
d. sarcoidosis
c. myasthenia gravis
21.2 A 25-year-old male has continued post operative bleeding after an extraction of an impacted third molar tooth under a general anaesthetic. The patient mentions that his father bruises quite easily. His coagulation screen reveals: (Coagulation tests provided). The most likely diagnosis is
His coagulation screen reveals: Prolonged APTT, Normal PT.
a) Factor V Leiden
b) Haemophilia A
c) Haemophilia B
d) Von willebrand disease
d) Von willebrand disease
- autosomal dominant inheritance
- may have normal or prolonged APTT, PT is normal
*Haem A: X-linked recessive disorder; would expect prolonged aPTT, and normal PT
*Haem B: X-linked recessive disorder; would expect normal aPTT and normal PT
Up to date:
Inheritance patterns — Most cases of VWD are transmitted as an autosomal dominant trait; this includes types 1 and 2B, and most types 2A and 2M.
Baseline hemostasis assessment —
Most patients will have a complete blood count (CBC) with platelet count and coagulation studies during the initial evaluation for excessive bleeding or bruising.
●Individuals with VWD generally have a normal CBC and a normal platelet count, with the exception of those with type 2B VWD, most of whom will have mild thrombocytopenia (eg, platelet count 100,000 to 140,000/microL).
●Individuals with VWD may have a normal or prolonged activated partial thromboplastin time (aPTT), depending on the degree of reduction of the factor VIII level. The prothrombin time (PT) is normal in VWD.
Up to date:
●Hemophilia A – Inherited deficiency of factor VIII (factor 8 [F8]); an X-linked recessive disorder.
●Hemophilia B – Inherited deficiency of factor IX (factor 9 [F9]); also called Christmas disease; an X-linked recessive disorder.
Laboratory findings —
Hemophilia is characterized by a prolonged activated partial thromboplastin time (aPTT).
However, the aPTT may be normal in individuals with milder factor deficiencies (eg, factor activity level >15 percent), especially in hemophilia B (factor IX deficiency), where even individuals with moderate disease may have a normal aPTT.
In some individuals with hemophilia A, factor VIII levels may increase with stress, leading to a normalization of the aPTT or mis-categorization of factor levels and disease severity.
In patients with hemophilia, the aPTT corrects in mixing studies, unless an inhibitor is present, which only applies to individuals who have received factor infusions or who have an autoantibody such as a lupus anticoagulant or an acquired factor inhibitor.
Mixing studies that do not show correction of a prolonged aPTT suggest an alternative diagnosis such as an acquired factor inhibitor.
The platelet count and prothrombin time (PT) are normal in hemophilia.
Thrombocytopenia and/or prolonged PT suggest another diagnosis instead of (or in addition to) hemophilia.
Measurement of the factor activity level (factor VIII in hemophilia A; factor IX in hemophilia B) shows a reduced level compared with controls (generally <40 percent).
One exception is an individual with mild hemophilia A who undergoes testing when stressed or pregnant and has a falsely elevated factor level. If this is suspected, factor activity testing should be repeated under conditions of low stress.
The plasma von Willebrand factor antigen (VWF:Ag) is normal in hemophilia.
If VWF:Ag is reduced, this suggests the possibility of von Willebrand disease (VWD) rather than (or in addition to) hemophilia.
Urinalysis is not done routinely, but if performed it may sometimes (but not always) show microscopic or macroscopic hematuria.
22.1 An anaesthetised patient is ventilated and has standard monitoring plus a central venous line. As surgery commences, the line isolation monitor alarms, indicating a potential leakage current of greater than 5 mA from one of the power circuits in use. The most appropriate action is to
a) Ignore it
b) Disconnect non-essential
equipment one by one to identify fault
Line isolation monitor alarms when single fault in system. If the alarm is going off, the last piece of equipment plugged in is usually suspect and should be unplugged.
21.1 The atmospheric lifetime of nitrous oxide (in years) is approximately
A. 1yr
B. 10 yr
C. 50 yrs
D. 100years
100 years
Desflurane: 10yrs
Sevoflurane 1yr
ISO quoted about 3 to 5
23.1 Of the following, the drug that is LEAST likely to provide effective analgesia following paediatric tonsillectomy is
A. Inhalational anesthesia
B. Remifentanil at end of case
C. Dexamethasone
D. Intranasal ketamine
or
a. Ketamine
b. Clonidine
c. NSAIDs
d. Paracetamol
e. Dexamethasone
A. Inhalational anesthesia
or
b. Clonidine
Prospect: two studies focused on tonsillectomy, and those did not show any additional analgesic effect of clonidine when used on top of adequate baseline medication after tonsillectomy.
PROSPECT
https://associationofanaesthetists-publications.onlinelibrary.wiley.com/doi/10.1111/anae.15299#:~:text=The%20basic%20analgesic%20regimen%20should,analgesic%20and%20anti%2Demetic%20effects.
23.1 In a patient with glucose-6-phosphate dehydrogenase deficiency (G6PD), the
intravenous agent that should be avoided is
a. Methylene blue
b. Indocyanine green (ICG)
c. Iodine
d. Dextrose
a) methylene blue
Drugs to avoid:
Antibiotics
Sulphonamides (check with your doctor)
Co-trimoxazole (Bactrim, Septrin)
Dapsone
Chloramphenicol
Nitrofurantoin
Nalidixic acid
Antimalarials
Chloroquine
Hydroxychloroquine
Primaquine
Quinine
Mepacrine
Chemicals
Moth balls (naphthalene)
Methylene blue
Foods
Fava beans (also called broad beans)
Other drugs
Sulphasalazine
Methyldopa
Large doses of vitamin C
Hydralazine
Procainamide
Quinidine
Some anti-cancer drugs
21.2 A patient with a history of restless leg syndrome is agitated in the post-anaesthesia care unit.
After excluding other causes, the best treatment of the agitation in this patient is
a) Pethidine
b) Clonidine
c) Droperidol
d) Haloperidol
e) Midazolam
midazolam
- Opioids, benzodiazepines and pregabalin may also be used to alleviate symptoms.
Perioperative treatment of symptoms
If RLS symptoms occur perioperatively, patients should be allowed to walk or move their legs in bed as soon as possible.
If prolonged bed rest is required, the frequency of RLS medications may be increased to three times a day.
If oral intake is feasible, a patient’s usual oral medication may be given.
Levodopa (a dopamine agonist) may be administered by nasogastric tube.
Alternatively, parenteral apomorphine or a rotigotine patch may be used.
Apomorphine (1 milligram) may be injected subcutaneously on an hourly basis.
Nausea is a common side effect so it may need to be given with an antiemetic.
Rotigotine patches may be used every 24 hours.
Opioids, benzodiazepines and pregabalin may also be used to alleviate symptoms.
Patients should be proactively investigated and treated for iron deficiency, targeting ferritin level greater than 300 micrograms/ litre in adults, and 50 micrograms/litre in children.
21.1 Unsupported ventilation in a non-anaesthetised patient with long-standing tetraplegia is improved when
a) Trendelenberg
b) Reverse Trendelenberg
c) Supine
d) Left lateral
e) Right lateral
C) supine
Vital capacity is increased in the supine position as abdominal wall paralysis permits greater displacement of abdominal contents during caudad diaphragmatic excursion. Patients will benefit from being recovered in the supine position.
Effect of the level of the lesion
Lesions above C3: complete dependence on mechanical ventilation because of phrenic nerve denervation causing complete diaphragmatic paralysis.
Lesions between C3 and C5: variable dependence on ventilatory support because of variable effect on diaphragmatic and accessory muscle function.
Lesions between C6 and C8: they may require intermittent non-invasive ventilatory support. Intact diaphragmatic function and accessory neck muscles enable adequate inspiratory effort. However, intercostals and abdominal wall muscles remain paralysed. Exhalation occurs via passive recoil of the chest wall, and cough is impaired. There is an increased risk of pneumonia because of poor mobilization of lung secretions.
Thoracic injuries: little respiratory compromise; the main problems are attributable to an inefficient cough.
20.2 When providing anaesthesia for endovascular treatment of acute ischaemic stroke, the Society of NeuroInterventional Surgery and the Neurocritical Care Society recommend
a) General anaesthesia
b) Hypervolaemia
c) Maintain temp >35
d) Maintain BGL 8-12
e) Maintain SBP 140-180
e) Maintain SBP 140-180
- Tight control of BP, preferentially with IABP
> goal of >140/90 mmHg and <180/105 mmHg. - Oxygen supplementation to maintain SpO2 >92%.
- Maintenance of eucapnia to avoid cerebral vasoconstriction
> (ETCO2 35- 45 mmHg) - Temperature maintained 35-37c
- Euglycaemia (BGL 70-140 mg/dL (4-8 mmol/L)) and hourly monitoring
21.1 Of the following, allergy based on cross reaction to penicillin sensitivity is most likely with
A) Cephazolin
B) ceftriaxone
C) cefapime
D) cefaclor
E) cefoxatin
D) Cefaclor
- Cephalexin? More so than Cephazolin (no B-lactam)
- Cefaclor
Source: UpToDate
21.1 21.2 The breast does NOT receive sensory innervation from the
a. Long thoracic
b. Thoracodorsal
c. Anterior intercostals
d. Posterior intercostals
e. Supraclavicular
b. Thoracodorsal
Thoracodorsal nerve (C6-C8) is a branch of the posterior cord of the brachial plexus. Its primary function is motor innervation of the latissimus dorsi muscle. Its blockade is relevant in more extensive breast reconstruction procedures.
The Pecs I, Pecs II and Serratus Plane blocks are superficial thoracic wall blocks which through blockade of the
1. Pectoral N.
2. Intercostal N.
3. Thoracodorsal N.
3. Long thoracic N.
It can be used to provide analgesia for breast surgery and other procedures/surgery involving the anterior chest wall.
22.2 1 MAC of sevoflurane affects the sensory evoked potential signal for spinal surgery by
a) increased latency, increased conduction speed, increased amplitude
b) increased latency, decreased conduction speed, decreased amplitude
c) decrease latency, increased conduction speed, decreased amplitude
d) increased latency, increased conduction speed, decreased aptitude
Increased latency, decreased conduction speed, decreased amplitude
20.1 The neurosurgical registrar has telephoned about a patient with a spinal cord tumour who is on the list for tomorrow. The registrar tells you the patient has Brown-Séquard syndrome (hemisection of the spinal cord). On clinical examination, below the level of the lesion, you would expect to find all EXCEPT ipsilateral
A. Hyperreflexia
B. Loss of tactile stimulation
C. Paralysis
D. Loss of pain/temperature
E. Loss of vibration/proprioception
D. Loss of pain/temperature
Brown-Sequard syndrome:
- Also known as Lateral hemi-section syndrome
- Causes
○ Common
§ Knife or bullet injuries
§ Demyelination
○ Rare
§ Spinal cord tumours
§ Disc herniation
§ Infarction
§ infection
Ipsilateral:
- Motor weakness
- Loss of vibration sensation
- Loss of proprioception sensation
Contralateral:
- Loss of pain sensation
- Loss of temperature
Segmental Syndrome:
- Pathologies that affect all functions of the spinal cord at one or more levels
- Total cord transection:
○ Cessation of function in all ascending and descending spinal cord pathways
○ Loss of all types of sensation below the level of the lesion
○ Loss of movement below the level of the lesion
- Acute transection:
○ Spinal shock
○ Flaccid paralysis
○ Urinary retention
○ Diminished tendon reflexes
○ This is usually temporary followed by:
§ Increased tone
§ Spasticity
§ Hyperrelfexia
§ supervene days or weeks after the event
- Transverse injuries above C3 involve sensation of respiration and are often fatal if acute
- Lesions above L2 will cause impotence and spastic paralysis of bladder
- Causes:
○ Myelopathies
§ Traumatic injury
§ Spinal cord haemorrhage
○ Epidural or intramedullary abscesses or tumours and transverse myelitis may have a more subacute presentation
Dorsal (posterior) cord syndrome:
- Bilateral involvement of:
○ Dorsal Columns
○ Corticospinal tracts
○ Descending central autonomic tracts to bladder control centres in the sacral cord
- Symptoms/signs:
○ Gait Ataxia (DC)
○ Paraesthesias (DC)
○ Weakness (CST)
§ Acute
□ Muscle flaccidity
□ Hyporeflexia
§ Chronic
□ Muscle hypertonia
□ Hyperreflexia
○ Extensor plantar response
○ Urinary incontinence (Auto)
- Causes:
○ MS
○ Tabes dorsalis
○ Friedrich ataxia
○ Sub-acute combined degeneration
○ Vascular malformations
○ Epidural and intradural extrameduallry tumours
○ Atlantoaxial subluxation
Ventral (anterior) cord syndrome
- Involves cords in the anterior 2/3rds of the spinal cord
○ Corticospinal tract
○ Spinothalamic tract
○ Descending autonomic tracts to the sacral centers for bladder control
- Signs/symptoms
○ Weakness (CST)
○ Reflex changes (CST)
○ B/L temp and pain sensation (Spino)
○ Tactile and vibratory sense are normal
○ Urinary incontinence (Auto)
- Causes:
○ Spinal cord infarction
○ Intervertebral disc herniation
○ Radiation myelopathy
21.1 The risk of a perioperative respiratory adverse event in a child is least likely to be increased by
A. Asthma
B. infection 3 weeks ago,
C. history of eczema,
D. passive smoking
history of eczema
21.2 The number of segments in the lower lobe of the left lung is
a) 3
b) 4
c) 5
d) 10
e) 12
b) 4
Right lung:
RUL: APA
RML: LM
RLL: SMALP
Left lung:
LUL: ASIA (S&I form the lingular lobe)
LLL: ALPS
Subsegments (total of 42)
Left: 10 + 10
Right: 6 + 4 + 12
21.2 Methylene blue may be used in the treatment of all of the following conditions EXCEPT
a) Methemoglobinemia
b) Priapism
c) Hepatopulmonary syndrome
d) G6PD deficiency
e) Sepsis
d) G6PD deficiency
(contraindicated)
Methylene blue PI:
PROVEBLUE® is indicated:
* for the treatment of drug-induced methaemoglobinaemia (e.g. prilocaine)
* for the treatment of idiopathic methaemoglobinaemia (in which structural abnormality of haemoglobin is not present)
* as a bacteriological stain
* as a dye in diagnostic procedures such as fistula detection
* for the delineation of certain body tissues during surgery.
Contraindications:
PROVEBLUE® is contraindicated in the following circumstances:
* known hypersensitivity to the drug or any other thiazide dyes
* patients with severe renal impairment
* patients with glucose-6-phosphate dehydrogenase deficiency
* methaemoglobinaemia due to chlorate poisoning
* methaemoglobinaemia during treatment of cyanide poisoning
Intrathecal and subcutaneous injection of methylene blue are also contraindicated as they can result in neural damage (intrathecal administration) and necrotic abscess (subcutaneous administration).
Precautions:
Methylene blue is a potent monoamine oxidase inhibitor.
Serotonin syndrome.
Dose:
Adults and children: In the treatment of methaemoglobinaemia, methylene blue is administered intravenously as the 0.5 % solution in doses of 1 to 2 mg per kg bodyweight injected over a period of 5 minutes.
A repeat dose may be given after one hour if required.
A maximum dose of 7mg/kg bodyweight is recommended.
The use of methylene blue is not recommended in infants under 4 months of age.
STAT PEARLS :Methylene blue
https://www.ncbi.nlm.nih.gov/books/NBK557593/
“Methylene blue is a safe drug at a therapeutic dose of <2 mg/kg; however, when levels are >7 mg/kg, many of the adverse effects it exhibits will occur. Serotonin syndrome has been found to occur when combining serotonergic agents with methylene blue at a dose of 5 mg/kg.”
Methylene blue: caution serotonin syndrome, G6PD deficiency
Indications: vasoplegic syndrome, plasmodium falciparum, methaemoglobinaemia, diagnostic purposes.
Safe at doses <2mg/kg. (used in vasoplegic syndrome on CPB at 3mg/kg - Up to date)
Serotonin syndrome at >5m/kg
Other adverse effects at >7mg/kg.
21.2 A patient presents with a serum sodium of 110 mmol/L. A feature NOT consistent with a
diagnosis of syndrome of inappropriate antiduretic hormone (SIADH) is
a) Urine osmolality <100mOsm/kg
b) Euvolaemic state
c) Urine Na >40 mmol/L
d) Increased cortisol
a) Urine osmolality <100mOsm/kg
DIAGNOSTIC CRITERIA
hypotonic hyponatraemia
urine osmolality > plasma osmolality (<275mOsm/kg) (i.e. concentrated urine despite hypotonic blood)
urinary Na+ > 20mmol/L
normal renal, hepatic, cardiac, pituitary, adrenal and thyroid function
euvolaemia (absence of hypotension, hypovolaemia, and oedema)
correction by water restriction
Source LITFL
The power of a two sample (two group) randomised controlled trial is NOT affected by (the)
a) Sample size
b) Statistical test
c) P value
d) Standard deviation/ variance in sample
e) Effect size
? same?
The power of a statistical test is not affected by the:
a) sample size
b) population variability
c) alpha error
d) effect size
e) robustness of the statistics
c) p-value
e) robustness of the statistics
The power of a study is determined by:
1. Frequency of outcome being studied
2. magnitude of effect
3. study design
4. sample size.
The greater the error variance (or standard deviation) the less the power
BARASH: Experimental Medicine: Statistical Analysis
POWER:
The error of failing to reject a false null hypothesis (false-negative) is called a type II or β-error.
(The power of a test is 1−β.)
The probability of a type II error depends on four factors.
- Unfortunately, the smaller the α, the greater the chance of a false-negative conclusion; this fact keeps the experimenter from automatically choosing a very small α.
- The more variability there is in the populations being compared, the greater the chance of a type II error. This is analogous to listening to a noisy radio broadcast; the more static there is, the harder it will be to discriminate between words.
- Increasing the number of subjects will lower the probability of a type II error.
- The most important factor is the magnitude of the difference between the two experimental conditions. The probability of a type II error goes from very high, when there is only a small difference, to extremely low, when the two conditions produce large differences in population parameters.
There are four options for decreasing type II error (increasing statistical power):
(1) raise α,
(2) reduce population variability,
(3) make the sample bigger, and
(4) make the difference between the conditions greater.
What is Alpha:
Alpha is also known as the level of significance. This represents the probability of obtaining your results due to chance. The smaller this value is, the more “unusual” the results, indicating that the sample is from a different population than it’s being compared to, for example. Commonly, this value is set to .05 (or 5%), but can take on any value chosen by the research not exceeding .05.
Alpha also represents your chance of making a Type I Error. What’s that? The chance that you reject the null hypothesis when in reality, you should fail to reject the null hypothesis. In other words, your sample data indicates that there is a difference when in reality, there is not. Like a false positive.
23.1 The odds ratio is the measure of choice for a
a. Case control
b. Cohort
c. RCT
d. Epidemiological study
a) case control
https://www.cdc.gov/csels/dsepd/ss1978/lesson3/section5.html
23.1 A patient with long-term severe anorexia nervosa is commenced on a normal diet. Three days later she develops cardiac failure and exhibits a decreased level of consciousness. The most important parameter to assay and normalise is the plasma
a. Phosphate
b. Potassium
c. Magnesium
d. Sodium
e. Calcium
a) Phosphate
hypophosphate: Clinical symptoms range from muscle weakness and paraesthesia to severe cardiac failure, seizures and diaphragmatic paralysis
Refeeding malnourished patients with anorexia nervosa can be associated with hypophosphatemia, cardiac arrhythmia and delirium. Phosphorus repletion should be started early with and serum levels maintained above 3 mg/dL
weakness and fatigue, in the context of a recent history of starting a regular diet while in a state of chronic malnutrition, are concerning for refeeding syndrome, which typically occurs 2 to 5 days after beginning nutritional repletion. Depleted phosphate stores due to prolonged starvation, hypocalcemia, and hypokalemia can lead to impaired muscle contractility and subsequently weakness, myalgia, and tetany.
https://www.ncbi.nlm.nih.gov/pmc/articles/PMC4168120/
21.1 Of the following, the device that delivers the greatest flow when using ‘Level 1® Fast Flow Fluid Warmer’ rapid fluid infuser system is a (list of intravascular catheters)
a. 6.5 Fr sheath
b. 8.5 Fr Multilumen line
c. 8.5 Fr Swan Ganz Sheath
d. Multilumen something 14G cannula (50mm?)
e. Peripheral RICC line, 8.5 Fr
e. Peripheral RICC line, 8.5 Fr
21.1 The risk of major bleeding in patients taking direct oral anticoagulants (DOACs) is NOT significantly increased by commencing administration of
a) Atorvastatin
b) Amiodarone
c) Digoxin
d) Diltiazem
e) Fluconazole
1st a) Atorvastatin
2nd c) Digoxin
source of Atorvastatin > Digoxin
https://www.ncbi.nlm.nih.gov/pmc/articles/PMC5818856/
All of the DOACs are avid substrates for the excretory P-gp system of the gastrointestinal epithelial cells, and drugs that inhibit or induce the P-gp system may affect plasma DOAC levels
Dabigatran and edoxaban are substrates for P-glycoprotein (P-gp)
Apixaban and rivaroxaban are metabolised by cytochrome P450 enzyme CYP3A4 and are substrates for P-gp
There is study evidence that among patients taking DOACs for non-valvular atrial fibrillation, concurrent use of amiodarone, fluconazole, rifampicin, and phenytoin compared with the use of DOACs alone, was associated with increased risk of major bleeding
It is unlikely that clinically significant interactions occur between dabigatran and other drugs that are merely substrates for P-gp-mediated excretion. When dabigatran was coadministered with digoxin neither digoxin nor dabigatran plasma levels were significantly altered
Rivaroxaban and apixaban are metabolised to an extent of 40–50 % in the liver to variable degrees by CYP3A4 and may interact with drugs that inhibit this enzyme.
The metabolism of Apixaban and rivaroxaban can be decreased when combined with Atorvastatin which is also metabolised by CYP3A4
23.1 Rotational thromboelastometry (ROTEM) is performed on a bleeding patient with the
following series of graphs produced. The most appropriate therapy to be
administered is
a. TXA
b. Fibrinogen
c. Cryo
d. FFP
a) TXA
Hyperfibrinolysis
https://derangedphysiology.com/main/required-reading/haematology-and-oncology/Chapter%201.2.0.1/intepretation-abnormal-rotem-data
22.1 This posteroanterior chest X-ray shows enlargement of the
(everyone seems to be unsure of answer, no image supplied)
a. Aorta
b. RA
c. RV
d. LA
e. LV
22.2 Blockade of the superficial cervical plexus includes the
a. C1 dermatome
b. C5
c. phrenic nerve
d. transverse cervical
e. greater occipital
d. transverse cervical
Supraclavicular nerve block. An initial injection of 3 mL local anesthetic is deposited at the midpoint of the sternocleidomastoid muscle, followed by 7 mL injected subcutaneously in a caudad and cephalad direction along the posterior border of the muscle.
complications:
1.Infection
2.Hematoma
3.Phrenic nerve block
4.Local anesthetic toxicity
5.Nerve injury
https://www.nysora.com/techniques/head-and-neck-blocks/cervical/cervical-plexus-block/
22.1 A 26-year-old patient presents with exertional syncope. The most likely diagnosis is
a. HOCM
b. Long QT syndrome
c. CCF
d. IHD
HOCM: pathopneumonic
A person who has syncope during exertion is more likely to have an obstruction to blood flow (aortic stenosis or hypertrophic cardiomyopathy) or ventricular tachycardia as a cause. On the other hand, syncope after completion of exercise is more likely of reflex origin, such as the common faint.
https://www.uptodate.com/contents/syncope-fainting-beyond-the-basics#:~:text=A%20person%20who%20has%20syncope,such%20as%20the%20common%20faint
21.1 Local anaesthetic-induced myotoxicity is most likely to be associated with
A. Biers
B. Interscalene
C. Sciatic
D. Adductor Canal
D. Adductor Canal
unclear phenomonenon
prolonged exposure and high concentrations of local anaesthetic
23.1 You are called to assist in the resuscitation of a 75-year-old patient in the emergency
department who is in extremis with severe hypotension and hypoxaemia. The image
shown is of a focused transthoracic echocardiogram, parasternal short axis view.
The most likely diagnosis is
a. PE
b. Tamponade
a) PE
D-shaped left ventricle
22.2 A thoracic regional technique that will NOT provide analgesia for sternal fractures is a
a. PECS I
b. PECS II
c. Parasternal intercostal nerve block?
d. Transfascial muscle block (can’t remember wording)
e. transverse thoracic plane block
b. PECS I
(PECS II Covers SA and will extend to the sternum)
21.2 With regard to the risk of postoperative surgical-site infection, 8 mg dexamethasone administered intraoperatively has
a) No increased risk of surgical wound infection
b) Increased surgical wound infection in diabetics
c) Increased surgical wound infection in non-diabetics
d) Decreased surgical wound infection
a) No increased risk of surgical wound infection
- Now, the Perioperative Administration of Dexamethasone and Infection Trial (PADDI), led by Professor Tomás Corcoran, Director of Research in the Department of Anaesthesia and Pain Medicine, Royal Perth Hospital has found that administering a low-dose of dexamethasone during anaesthesia for surgical operations does not increase the risk of surgical wound infections.
22.1 The train-of-four (TOF) ratio above which the majority of anaesthetists will NOT be able to visually detect fade on TOF stimulation is
a. 0.2
b. 0.4
c. 0.6
d. 0.7
e. 0.9
b. 0.4
BJA: Monitoring of neuromuscular block
https://academic.oup.com/bjaed/article/6/1/7/347026
When neuromuscular monitoring is used, visual or tactile evaluation of the degree of neuromuscular block is unreliable.
Even experienced anaesthetists are unable to detect fade when the TOF ratio is >0.4.
It is now thought that significant residual curarization is still present if the TOF ratio is <0.97 (not 0.7 as previously suggested8).
22.2 You are inserting a pulmonary artery catheter in an intubated patient prior to cardiac surgery and a significant amount of blood appears in the endotracheal tube. The most appropriate specific initial management is to
a. Remove PAC and insert DLT
b. Wedge PAC and insert BB
c. Wedge PAC and insert DLT
d. Withdraw PAC 2cm and insert DLT
d. Withdraw PAC 2cm and insert DLT
Pulmonary rupture
Miller:
- Position pt with bleeding lung dependent
- Perform endotracheal intubation, oxygenation, airway toilet
- Isolate lung by endobronchial DLT or SLT or bronchial blocker
- Withdraw PAC several centimetres, leaving it in the main PA. Do not inflate the balloon (except with fluoroscopic guidance)
- Position pt with isolated bleeding lung nondependent. Administer PEEP to the bleeding lung if possible
- Transport the patient to medical imaging for diagnosis and embolisation if feasible
21.1, 20.1 The drug which has the LEAST impact on somatosensory evoked potentials (SSEPs) monitored in a 15-year-old patient undergoing scoliosis surgery is
A) propofol
B) fentanyl
C) desflurane
D) Midazolam
E) sevoflurane
B) fentanyl
Drugs which have the least impact on SSEPs
1. Ketamine
2. Opioids
3. Dexmedetomidine
Article in Anaesthesiology
https://pubs.asahq.org/anesthesiology/article/99/3/716/40407/Pharmacologic-and-Physiologic-Influences-Affecting
o SSEPs = small amplitude potentials measured over the sensory cortex or via epidural electrodes from stimuli applied to the posterior tibial nerves. SSEPs are transmitted via the posterior columns of the spinal cord in the territory of the posterior spinal arteries which supply the posterior 1/3 of the cord. As they are low amplitude they are affected by basal muscle tremor and the signal-to-noise ratio is improved by increasing the depth of muscle relaxation. Their use is not significantly affected by therapeutic concentrations of anaesthetic vapours
o MEPs = series of short-duration constant current stimuli of 300-700 V applied to the motor cortex and measured via needle electrodes inserted into tibialis anterior, abductor halluces and vastus medialis muscles along with selected small muscles of the hands for reference. MEPs rely on corticospinal tract integrity which lies in the territory of the anterior spinal artery. MEPs therefore complement SSEPs in their assessment of spinal cord function. MEPs are large amplitude potentials and are incompatible with profound muscle relaxation. Neuromuscular blocking agents are therefore best avoided or given by infusion and dose optomised with discussion with the technicians (or just give remi).
o All anaesthetic vapours reduce MEP amplitude in a dose-dependent manner, and more than 0.5 MAC are not compatible with reliable monitoring. Thus Propofol TIVA is preferred.
- Remifentanil is commonly used due to low context sensitive half life and negligible effect on intraop evoked responses
22.1 Created by the Global Initiative for Chronic Obstructive Lung Disease, the alphabetical GOLD groups A to D are tools for the assessment of chronic obstructive pulmonary disease. These classes are based on
a. Symptoms and exacerbations
b. FEV1
c. FEV1 and exacerbations
d. FEV1/FVC and exacerbations
e. FEV1 and symptoms
Sx and exacerbations
FEV1
GOLD ABE assessment tool
23.1 A newborn baby is pale, limp, grimacing with stimulation, gasping weakly, and has a pulse rate of 90 beats per minute. This corresponds to an Apgar score of
A. 1
B. 2
C. 3
D. 4
E. 5
C. 3
Subject repeat but different stem
21.1 The independent predictors for severe bone cement implantation syndrome (BCIS) in cemented
hemiarthroplasty for hip fracture do NOT include
a. Male
b. GA
c. severe cardiopulmonary disease
d. Diuretic use
e. Age
b. GA
Independent predictors for severe BCIS were:
ASA grade III—IV
chronic obstructive pulmonary disease
medication with diuretics or warfarin
Source: BJA 2014 Article
https://academic.oup.com/bja/article/113/5/800/2920080
21.2 A factor that is NOT used to calculate the Child-Pugh score is
a) Albumin
b) Bilirubin
c) INR
d) Creatinine
e) Ascites
d) Creatinine
- Originally devised to predict outcomes in Cirrhotic patients undergoing portosystemic Surgery
- Assess perioperative risk for patients with liver disease who undergo hepatic or non-hepatic surgery
- Factors include:
o Encepahlopathy
§ None +1
§ Mild to moerate + 2
§ Severe +3
o Ascites
§ None +1
§ Mild to moderate (diuretic responsive) +2
§ Severe (diuretic refractory) +3
o Bilirubin
§ <2 mg/dl +1
§ 2-3mg/dl +2
§ >3 mg/dl +3
o Albumin
§ >3.5g/dl +1
§ 2.8-3.5g/dl +2
§ <2.8g/dl +3
o INR
§ <1.7 +1
§ 1.7-2.3 +2
§ >2.3 +3 - Class A 5-6 points
o 1-5yr survival rate 95% - Class B 7-9 points
o 1-5 year survival rate 75% - Class C 10-15 points
1-5 yr survival rate 50%
Original study Mortality rates in patients who undergo abdominal surgeries:
- Class A 10%
- Class B 82%
- Class C 82%
Newer Study mortality rates after surgery:
- Class A 2%
- Class B 12%
- Class C 12%
Drawbacks:
- Subjective measurement of:
o ascites
o encephalopathy
- Does not consider
o Pre-op infection
o Aetiology of cirrhosis
o Surgery type
21.1 A patient with a purely metabolic acidosis has a serum bicarbonate of 14 mmol/L and a lactate of 3.8 mmol/L. The expected PaCO2 is
a. 24
b. 29
c. 35
d. 40
B. 29
PaCO2= 1.5 x 14 + 8
PaCO2= 21 + 8
PaCO2= 29
Winter’s formula: expected PaCO2 = [1.5 x (serum HCO3)] + [8±2]
if PaCO2 lower, there is a concomitant primary respiratory alkalosis
if PaCO2 higher, there is a concomitant primary respiratory acidosis
A 50-year-old man has the following pulmonary function test result. The most consistent diagnosis is
FEV1 98% predicted
FVC 98% predicted
DLCO 48% predicted
a) Asthma
b) Obesity
c) Sarcoidosis
d) Pulmonary hypertension
d) Pulmonary hypertension
Normal spirometry + low DLCO
Asthma: obstructive pattern and normal DLCO
Obesity: restrictive pattern and normal DLCO
Sarcoid: restrictive pattern and low DLCO
The maximum warm ischaemic time acceptable for procuring the kidney following donation after cardiac death is
A. 30 mins
B. 60 mins
C. 90 mins
D. 120 mins
E. 180 mins
Warm ischaemia time:
- Time from treatment withdrawal to the start of cold perfusion of the donated organs
- Significance is the impact on graft function
- Most important phase of WIT begins when the systolic BP is < 60mmHg
- This includes the waiting period from the absence of circulation to the declaration of death and the time before initiating the flow of cold perfusate through the cannula
Maximum WARM Ischaemia time
- Heart 30 mins
- Liver 30 mins
- Pancreas 30 mins
- Kidney 60 mins
- Lungs 90 mins
Maximum COLD Ischaemia time:
- Heart = 4 hrs
- Lungs = 6-8hrs
- Liver/Pancreas = 12hrs (DBD)/6 hrs (DCD)
- Kidneys = 18hrs (DBD)/ 12 hrs (DCD)
22.2 A 47-year-old man is anaesthetised for an elective laparoscopic cholecystectomy. Three minutes after induction, he is noted to have a heart rate of 130 bpm and systolic blood pressure of 60 mmHg. The most appropriate initial dose of adrenaline is
a) 100mcg IM Adr
b) 200mcg IM Adr
c) 20mcg Adr IV
d) 100mcg Adr IV
e) 50mcg Adr IV
c) 20mcg Adr IV
ANZAAG use Ring and Mesmer scale for anaphylactic reactions as a base for classifying anaphylaxis grade (see image)
From sunny coast QH document
With PAGS ‘Life Threatening Anaphylaxis’ can be distinguished from
‘Moderate Anaphylaxis’ in an adult by the presence of any
one of these signs:
* systolic blood pressure of <60 mmHg
* life-threatening tachy- or bradyarrhythmia
* oxygen saturation <90%
* inspiratory pressures of >40 cmH2
Life-threatening anaphylaxis
21.2 Cardiovascular effects of hyperthyroidism include
a) Increased DBP
b) Narrow pulse pressure
c) Reduced diastolic relaxation
d) Decreased CO
e) Decreased SVR
e) Decreased SVR
- increased CO, increased SBP and decreased DBP with widened PP
UP TO DATE: Cardiovascular effects of hyperthyroidism:
- Thyroid hormone has important effects on cardiac muscle, the peripheral circulation, and the sympathetic nervous system that alter cardiovascular hemodynamics in a predictable way in patients with hyperthyroidism.
- The main changes are :
●Increases in heart rate, cardiac contractility, systolic and mean pulmonary artery pressure, cardiac output, diastolic relaxation, and myocardial oxygen consumption
●Reductions in systemic vascular resistance and diastolic pressure
20.2 When performing an infraclavicular block of the brachial plexus under ultrasound guidance, the structure indicated by the arrow is the (ultrasound image shown
a) Musculocutaneous nerve
b) Lateral cord
c) Medial cord
d) Superior trunk
e) Inferior trunk
c) Medial cord
20.1 Abuse of nitrous oxide may lead to
a. Anaemia due to decreased erythropoietin
b. Anaemia due to glutathione deficiency
c. Neurological damage due to methionine deficit
d. Pulmonary HTN
C
Methionine Synthetase Inhibitor
22.1 A patient in atrial fibrillation with a CHA2DS2-VASc score of 2 has presented for elective hip surgery. Warfarin had been ceased for four days preoperatively and on the day before surgery the international normalized ratio (INR) was 2.1. The best course of action at this point is to
a) Postpone surgery
b) Vitamin K 3mg IV
c) Prothrombinex 25IU/kg
d) Cell saver intraop
e) Proceed with surgery
Give 3mg of Vitamin K and re-check on day of surgery proceed if INR <1.5 on DOS
23.1 A patient is suffering an acute myocardial infarction. Australian and New Zealand
guidelines recommend the threshold for the use of supplemental oxygen is when the
SpO2 falls below
a. 88%
b. 90%
c. 93%
d. 97%
e. 100%
c) 93%
ANZCOR suggests against the routine administration of oxygen in persons with chest pain.13 [2015 COSTR, weak recommendation, very-low certainty evidence]
For persons with heart attack, routine use of oxygen is not recommended if the oxygen saturation is >93% [National Heart Foundation of Australia & Cardiac Society of Australia and New Zealand: practice advice].9
21.2 Cryoprecipitate contains all of the following EXCEPT
a) Factor I
b) Factor VII
c) Factor VIII
d) VWF
e) Fibronectin
b) Factor VII
Redcross:
Cryoprecipitate contains most of the following found in fresh frozen plasma:
1. factor VIII
2. fibrinogen
3. factor XIII
4. von Willebrand factor
5. fibronectin
Prothrombinex-VF® is a lyophilised concentrate of human coagulation factors it contains:
Factors:
II
IX
X
small amount of factor VII.
Also contains:
plasma proteins (human)
Antithrombin III (human)
Heparin sodium (porcine)
Sodium
Phosphate
Citrate
Chloride
20.2 During the 21st century, the dominant ozone-depleting substance emitted as a result of medical usage to date has been
a) Desflurane
b) Nitrous oxide
c) CO2
d) Isoflurane
e) CFCs
Nitrous oxide
Halothane & isoflurane cause catalytic destruction of ozone, but halothane hardly used and isoflurane has short atmospheric lifetime.
Desflurane + sevoflurane don’t cause ozone depletion.
21.2 The size 5 i-gel® supraglottic airway is recommended for patients who weigh over
a) 50kg
b) 60kg
c) 70kg
d) 80kg
e) 90kg
e) 90kg
21.2, 22.2 Of the following, the procedure that is most commonly associated with chronic pain after surgery is
a) Amputation
b) Mastectomy
c) Thoracotomy
d) TKR
e) Hernia repair
a) Amputation
Top 10 Rank order:
1. Amputation 30-85%
2. Thoracotomy 5-67%
3. Mastectomy 11-57%
4. Inguinal hernia repair 0-63%
5. Sternotomy 28-56%
6. Cholecystectomy 3-56%
7. Knee arthroplasty 19-43%
8. Breast Augmentation 13-38%
9. Vasectomy 0-37%
10. Radical prostatectomy 35%
21.1 Local anaesthetic systemic toxicity does NOT manifest as
a) hypoxaemia
b) severe agitation
c) sinus bradycardia
d) VF
e) seizures
a) hypoxaemia
After LA administration, any abnormal cardiovascular or neurological symptoms and signs, including isolated cardiac arrest, should raise suspicion of LAST
Presenting features of LAST vary widely. Cardiovascular collapse may occur without preceding neurological changes.
Clinical features of LAST:
CNS
- 2 stage process of excitatory phase followed by a depressive phase
- early signs:
1. perioral tingling
2. tinnitus
3. slurred speech
4. lightheadedness
5. tremor
6. change in mental state: confusion and agitation
- excitatory phase culminates in generalised convulsions
-Depressive phase:
1. Coma
2. Respiratory depression
CVS
- 3 phases:
- initial phase:
Htn and tachycardia
- intermediate phase:
myocardial depression and hypotension - terminal phase:
peripheral vasodialtion
severe hypotension
arrhythmias:
1. sinus bradycardia
2. conduction blocks
3. VT
4. Asystole
21.2 The risk of postoperative respiratory failure in myasthenia gravis is increased by the
administration of
a) Teicoplanin
b) Flucloxacillin
c) Cephazolin
d) Gentamicin
e) Vancomycin
d) Gentamicin
Drugs in the anaesthetic trolley that may unmask or worsen MG:
- NMBs
- gentamicin
- beta blockers (metoprolol)
- magnesium
Anaesthetic drugs to be cautious with:
- dexamethasone
- antipsychotics
- anticonvulsants
- antibiotics (vancomycin, metronidazole)
20.2 The flow volume loop is most consistent with (Flow-volume loop shown)
a) Variable intra-thoracic obstruction
b) Variable extra-thoracic obstruction
c) Lower airway obstruction
d) Fixed upper Airway obstruction
e) Mixed pattern
d) Fixed upper Airway obstruction
Fixed upper airway obstruction (can be intrathoracic or extrathoracic): flow limitation and flattening are noted in both the inspiratory and expiratory limbs of the flow-volume loop.
22.2 The nerve labelled by the arrow marked G in the diagram is the
- Ulnar Nerve
- Axillary Nerve
- Median Nerve
- Medial Cutaneous nerve of the forearm
- Long Thoracic Nerve
- Dorsal Scapular Nerve
- Radial Nerve
- Suprascapular nerve
- Musculocutaneous Nerve
- Radial Nerve
23.1 A patient requiring an elective major joint replacement has had a recent stroke. The minimum recommended duration between the stroke and surgery is
a) 3 months
b) 6 months
c) 9 months
d) 12 months
c. 9
AHA guidelines
12 Months
But 12 weeks minimum
Although the evidence between surgical timing and stroke risk is limited to only these 2 studies, we suggest that elective noncardiac surgery be deferred at least 6 months after a prior stroke, and possibly as long as 9 months to reduce the risk of perioperative stroke in patients undergoing noncardiac surgery.
Alternatively, patients who stand to gain significant improvements in quality of life with elective surgery may consider waiting only 6 months after a prior stroke
REPEAT
20.1 What is the arrow pointing to?
a. Psoas
b. Femoral
c. Genitofemoral
d. Lumbosacral trunk
e. Obturator
d. Lumbosacral trunk
20.1 A 25-year-old man suffers a 30% total body surface area burn. A physiological change expected within the first 24 hours is
a) Increase PVR
b) Decreased SVR
c) Increased cardiac index
d) Increased stroke volume
a) Increase PVR
UTD:
Low cardiac output “ebb,” phase (resuscitative phase) –
In the first 48 hours after a major burn, cardiac output (CO) is reduced up to 60 percent from baseline due to:
1. hypovolemia from permeability-induced plasma loss
2. reduced myocardial response to catecholamines
3. increased systemic vascular resistance due to elevated vasopressin levels
4. depressed myocardial contractility
5. possible myocardial ischemia due to decreased coronary blood flow
The large volumes that these patients require can sometimes result in over-resuscitation, leading to pulmonary edema and right heart failure.
High cardiac output “flow,” phase (recovery phase)
During the recovery phase 72 to 96 hours postburn, hyperdynamic and hypermetabolic responses result in:
1. increased CO
2. Tachycardia
3. increased myocardial oxygen consumption
4. decreased systemic vascular resistance
Elevation of catecholamines in major burns produces:
1. hyperdynamic circulation
2. augments energy expenditure
3. romotes protein catabolism in skeletal muscle.
Nonselective beta blocker therapy is sometimes used to block catecholamine receptors, treat cardiac dysfunction, and modulate the hyperdynamic response during this phase
23.1 Cryoprecipitate contains coagulation factors
A. 2, 8, 13, von willebrands
B. 1, 7, 13 , von willebrands.
C. 1, 8, 13, von willebrands.
D. 2, 7, 13, von willebrands.
C.
Cryoprecipitate contains Factor VIII, XIII, fibrinogen (factor I), fibronectin, vWF
https://www.lifeblood.com.au/health-professionals/products/blood-components/cryoprecipitate
21.1 A 40-year-old man suffers a hydrofluoric acid burn to 60% of his total body surface area in an industrial accident. An expected electrolyte disturbance is:
a. Hypocalcemia
b. Hyponatremia
c. Hypophosphatemia
d. Hypomagnesemia
hypoCALCEMIA
Hydrofluoric acid: highly corrosive inorganic aacid of element fluorine
- used in glass etching, electronics industry and cleaning solutions for metals, stone and marble
- dilute solutions can rapidly penetrate the skin and exposure of even a small area can be fatal (2%TBSA)
Fluoride ions bind with calcium and magnesium ions in the tissues
-> significant hypocalcaemia and hypomagnesaemia
-> Hyperkalaemia may also be seen
Direct cardiotoxic effects of fluoride ions can lead to cardiac arrhythmias that are difficult to treat
20.1 The radial artery pressure trace shown below is from a patient who has an intra-aortic balloon pump in situ. The device has been switched to 1:2 augmentation to assess the timing. The trace shows an augmented beat followed by an un-augmented beat. With respect to the augmentation, the trace shows
a. Correct timing
b. Early inflation
c. Late inflation
d. Early deflation
e. Late deflation
d. Early deflation
https://litfl.com/intra-aortic-balloon-pump-trouble-shooting/
Waveform features:
> There is a sharp drop in pressure immediately following the peak of diastolic augmentation (peak B).
> Diastolic augmentation may be suboptimal but it is difficult to confirm in the absence of a pressure scale or comparison to an unassisted waveform.
> Assisted aortic end-diastolic BP may be sub-optimally increased (trough C), but it is difficult to say based on the information given. With early deflation a widened U-shaped trough is typically seen.
> Early deflation can lead to an assisted aortic end-diastolic BP that equals or exceeds the unassisted aortic end-diastolic BP (trough F), although this is not the case in this scenario.
> Assisted SBP (peak D) is the same or higher than the unassisted SBP (peak A) – it should be slightly less.
Early IAB deflation may result in:
> inadequate coronary perfusion, with the potential for retrograde coronary blood flow. This may result in angina due to decreased myocardial oxygen supply.
> suboptimal afterload reduction and increased myocardial oxygen demand
How to correct:
Prolong the IAB inflation time, so that it deflates at the end of diastole, just before the onset of isovolumetric systolic contraction.
22.1 When fresh frozen plasma is administered to treat hypofibrinogenaemia in a bleeding patient, the volume required to achieve an increase in plasma fibrinogen concentration of one gram per litre is
A. 5 ml/kg
B. 10 ml/kg
C. 20 ml/kg
D. 30 ml/kg
E. 50 ml/kg
D. 30 ml/kg
Identification and Management of Obstetric Hemorrhage
Anesthesiology Clinics - Obstetric Anesthesia (2017)
https://www.anesthesiology.theclinics.com/article/S1932-2275(16)30074-X/fulltext
Although FFP, cryoprecipitate, and fibrinogen concentrates can all be used to increase fibrinogen levels, the optimal strategy for managing hypofibrinogenemia in obstetric hemorrhage is unclear. The relatively low concentration of fibrinogen in FFP limits its usefulness in the treatment of significant hypofibrinogenemia. To increase fibrinogen plasma level by 1 g/L, 30 mL/kg of FFP is necessary, increasing the risk of pulmonary edema and other hypervolemic complications. Cryoprecipitate, which is a concentrated source of fibrinogen, factor VIII, fibronectin, von Willebrand factor (vWF), and factor XIII, will increase fibrinogen levels by ~0.7 to 1 g/L for every 100 mL given. Although cryoprecipitate is associated with a lower transfusion volume, the standard “dose” (10 U) is typically prepared by pooling concentrates from multiple donors. Given the risk of infectious disease transmission and/or an immunologic reaction from exposure to multiple donors, several countries preferentially use purified, pasteurized fibrinogen concentrate for the treatment of congenital and/ or acquired hypofibrinogenemia. Fibrinogen concentrates are also prepared from large donor pools, but subsequent processing removes or inactivates potentially contaminating viruses, antibodies, and antigens. Studies comparing cryoprecipitate and fibrinogen concentrates utilization in hemorrhage resuscitation suggest fibrinogen concentrates are associated with lower blood loss, decreased RBC transfusion, and greater increases in plasma fibrinogen levels. Although the most appropriate method of fibrinogen replacement is somewhat controversial, the critical role of fibrinogen in reversing the coagulopathy accompanying obstetric hemorrhage is clear. As such, close monitoring and replacement of fibrinogen are crucial in the management of the bleeding parturient.
23.1 In patients without other comorbidities, bariatric weight loss surgery is indicated when
the body mass index (kg/m2) is greater than
a. 35
b. 40
c. 45
d. 50
a. 35
Major updates (2022) to 1991 National Institutes of Health guidelines for bariatric surgery
Metabolic and bariatric surgery (MBS) is recommended for individuals with a body mass index (BMI) 35 kg/m2 , regardless of presence, absence, or severity of co-morbidities.
MBS should be considered for individuals with metabolic disease and BMI of 30-34.9 kg/m2
BMI thresholds should be adjusted in the Asian population such that a BMI 25 kg/m2 suggests clinical obesity, and individuals with BMI 27.5 kg/m2 should be offered MBS.
Long-term results of MBS consistently demonstrate safety and efficacy.
Appropriately selected children and adolescents should be considered for MBS.
https://www.soard.org/article/S1550-7289(22)00641-4/fulltext#:~:text=The%201991%20NIH%20Consensus%20Statement,surgery%20that%20is%20applied%20universally.
23.1 According to the ANZICS Statement on Death and Organ Donation (2021), for the diagnosis of brain death after resuscitation and return of circulation following cardiorespiratory arrest, clinical testing should be delayed for at least
a. 12hr
b. 24hr
c. 36hr
d. 48hr
e. 72hr
b) 24 hrs
20.1 To reduce the risk of ?re-bleed, Neuroradiology society recommend:(uncertain source of this question)
a. Coiling <24hrs
b. Coiling >24hrs
c. Clipping <24hrs
d. Clipping >24hrs
A or D
International subarachnoid aneurysm trial (ISAT) of neurosurgical clipping versus endovascular coiling in 2143 patients with ruptured intracranial aneurysms: a randomised comparison of effects on survival, dependency, seizures, rebleeding, subgroups, and aneurysm occlusion
Findings:
In patients with ruptured intracranial aneurysms suitable for both treatments, endovascular coiling is more likely to result in independent survival at 1 year than neurosurgical clipping; the survival benefit continues for at least 7 years. The risk of late rebleeding is low, but is more common after endovascular coiling than after neurosurgical clipping.
20.2 You are called to assist with a patient in the intensive care unit who has had cardiac surgery three days ago and is now in cardiac arrest. External cardiac massage should aim for a systolic blood pressure of
a. 40
b. 60
c. 80
d. 100
e. 120
b. 60
BJA Article - Management of cardiac arrest following cardiac surgery - BJA Education
In the CICU, the effectiveness of ECC is confirmed by monitoring the arterial pressure trace with a target compression rate and depth to achieve a systolic impulse of > 60 mm Hg to maintain a mean perfusion pressure, preventing ventricular distension, LV wall stress, and ischaemia.
22.2 Normal (0.9%) saline has the physical properties of
a. Na 140, 280 mOsm/L
b. Na 148, 296 mOsm/L
c. Na 150, 300 mOsm/L
d. Na 154, 308 mOsm/L
D Na 154, 308 mOsm/L
21.2 The most common complication of extracorporeal membrane oxygenation (ECMO) in adults is
a) Bleeding
b) Vascular damage
c) Embolism
d) Inadvertently decannulate
a) Bleeding
ECMO complications:
- patient complications: bleeding & coagulopathy most common
- mechanical complications: access insufficiency common
Blue book 2017
20.1 What is the abnormality in this CXR?
a. Pneumonectomy
b. Pleural effusion
c. Pneumonia
d. Unilateral pulmonary oedema
c. Pneumonia
Complete white-out of left lung with air bronchogram sign consistent with total consolidation. Patchy areas of ground glass opacities in right lung. There is silhouetting of the left heart border and left hemi-diaphragm. There is no rib crowding to suggest atelectasis.
Complete white-out of the hemithorax with air bronchograms and trachea in a central position is consistent with consolidation secondary to pneumonia. The patient is on linezolid, micafungin, and piperacillin-tazobactam pending culture and susceptibility studies.
Differential diagnosis of hemithorax white-out with a midline trachea include:
- consolidation
- pulmonary edema/ARDS
- pleural mass
- chest wall mass
22.2 Of the following, the congenital condition LEAST commonly associated with obstructive sleep apnoea in children is
Hypoplastic mandible (micrognathia) – difficult intubation
§ Pierre Robin sequence
§ Treacher Collins
§ Hemifacial microsomia (Goldenhar syndrome)
Midface hypoplasia – difficult bag-mask ventilation
§ Apert syndrome
§ Crouzon syndrome
§ Pfeiffer syndrome
§ Saethre-Chotzen syndrome
Macroglossia – difficult bag-mask ventilation AND difficult intubation
§ Hurler’s/Hunter’s syndrome (mucopolysaccharidoses)
§ Beckwith-Wiedemann syndrome
§ Down’s syndrome
https://www.frca.co.uk/Documents/250%20The%20Difficult%20Paediatric%20Ai
Mucopolysaccharidoses, Down syndrome, muscular dystrophies, and other neurologic disorders have been associated with obstructive sleep apnea
Prevalence of OSAS.
Genetic Disorder Prevalence of OSAS
Neuromuscular diseases 69.2%
Prader–Willi syndrome 94.7%
Arnold–Chiari syndrome 80%
Achondroplasia 100%
Crouzon syndrome 100%
https://www.ncbi.nlm.nih.gov/pmc/articles/PMC8156845/
https://www.frca.co.uk/Documents/250%20The%20Difficult%20Paediatric%20Ai
(
21.1 A 10-year-old boy (weight 30 kg) has a displaced distal forearm fracture that requires manipulation and application of plaster. The volume of 0.5% lidocaine (lignocaine) that should be used for intravenous regional anaesthesia (Bier block) is
a. 12 ml
b. 18ml
c. 30 ml
d. 42 ml
b. 18ml
3mg/kg max dose as per RCH guidelines
3mg x 30kg = 90mg
90mg/5mg/ml = 18ml
or
0.6ml/kg of 0.5% Lignocaine
0.6ml x 30kg = 18ml
https://www.rch.org.au/clinicalguide/guideline_index/Bier_block/
20.1 A 55-year-old lady scheduled for a transphenoidal hypophysectomy undergoes an oral glucose tolerance test with the following results:
GH normal <10
Time 0, BSL 5.5, GH 30, IGF-1 790 (elevated)
Time 30, BSL 7.6, GH 24
Time 60, BSL 7.2, GH 28
Time 90, BSL 6.5, GH 26
Time 120, BSL 5.8, GH 29
These results are most consistent with a diagnosis of
A. Prolactinoma
B. Acromegaly
C. Cushing’s
D. MEN 2
E. Normal
Acromegaly
IGF-2 is consistently elevated
GH should be suppressed by glucose load in healthy
pt.
The continued elevation of GH despite glucose is
suggestive of acromegaly
23.1 In children, severe sleep apnoea is suggested by an apnoea-hypopnoea index
greater than
a. 10
b. 15
c. 20
d. 30
e. 40
a) 10
21.1 You are performing a regional block for analgesia following knee surgery. You have an ultrasound probe scanning the anterior mid-thigh. The muscle indicated by the arrow in the ultrasound image below is the
A: Sartorius
B: Vastus Medialis
C: Adductor Longus
D: Gracilis
E: Rectus femoris
B: Vastus Medialis
20.2 Elimination of remifentanil occurs following breakdown mainly by
a Plasma cholinesterase
b RBC esterases
c Hoffman degradation
d Hepatic Metabolism
e Plasma esterases
e Plasma esterases
Plasma esterases (not cholinesterase)
Esmolol metabolism is via RBC esterases.
22.1 You are planning to perform a peribulbar block and wish to check the axial length of the eye prior to proceeding. The average axial length of the globe in adults as measured by ultrasound is
a. 20mm
b. 23mm
c. 26mm
d. 29mm
e. 32mm
B 23mm
422.1 The current ANZCA guidelines for preoperative fasting of adult patients state that studies have shown that it is safe to administer
a) unlimited clear fluid 2 hours prior
b) 200ml clear fluid 2 hours prior
c) 300ml clear fluid 2 hours prior
d) 400ml clear fluid 2 hours prior
400mls of clear fluids pre op
Safe upper limit - definitely has not not been identified and will vary from patient to patient.
Clear fluids
Water / CHO rich fluids / pulp free fruit juice / clear cordial / black tea and coffee
20.2 The structure labelled A shows (gastric ultrasound image shown)
a. Empty stomach
b. Full stomach with Solids
c. Full stomach with liquids and Air
d. Gall Bladder
e. Abdominal Aorta
c. Full stomach with liquids and Air
22.1 An asymptomatic 65-year-old male with squamous cell carcinoma of the left lung has been referred for assessment of suitability for lung resection. There is no evidence of spread on computerised tomography scanning. PaCO2, electrocardiogram, full blood count and electrolytes are normal. His SpO2 on room air is 95%. His forced expiratory volume in one second is 2.3 litres (predicted 3.3 litres) and forced vital capacity is 3.4 litres (predicted 4.4 litres). The most appropriate course of action is to
a. Proceed with lobectomy or pneumonectomy
b. Proceed with lobectomy only
c. DLCO testing
d. Lung V/Q scan
e. CPET
a. Proceed with lobectomy or pneumonectomy
FEV1 surgical suitability:
- >80% or >2l pneumonectomy
○ no further testing required
- >80% or >1.5l lobectomy
○ no further testing required
- <80% or <2l for pneumonectomy
○ -> calculate ppoFEV1
- <80% or <1.5l for lobectomy
○ -> perform DLCO and express as % of predicted DLCO
○ Saturations on air
- ppoFEV1 < 40% and DLCO <40% = High Risk
- ppoFEV1 >40% and DLCO >40% and SaO2 >90% = Average risk (no further testing)
21.1 The composition of blood returned to the patient from intraoperative cell salvage shows
A. No evidence of haemolysis
B. Normal 2,3 DPG
C. Nil evidence of bone cement or some embolism type
D. Normal levels of coagulation factors
B. Normal 2,3 DPG
higher Hct-60%
No immunimodulation
require reinfusion within 6hrs
pause with sement, caution metal fragments
22.2 The nerve labelled by the arrow marked J in the diagram is the
- Ulnar Nerve
- Axillary Nerve
- Median Nerve
- Medial Cutaneous nerve of the forearm
- Long Thoracic Nerve
- Dorsal Scapular Nerve
- Radial Nerve
- Suprascapular nerve
- Musculocutaneous Nerve
- Medial Cutaneous nerve of the forearm
21.2, 20.1 The muscle or muscle group with the greatest resistance to the action of non-depolarising neuromuscular blocking agents is the
a) Adductor pollicis
b) Diaphragm
c) Orbicularis oculi
d) Pharyngeal
b) Diaphragm
22.2 The normal axial length of the globe of an adult eye is
a. 20mm
b. 23mm
c. 26mm
d. 29mm
e. 32mm
23mm
22.1 The nerve labelled by the arrow in the diagram (image of brachial plexus given) is the
a. Median nerve
b. MC nerve
c. Radial nerve
d. Ulnar nerve
Extensive Brachial plexus anatomy questions produced in 2022.2 paper on Brainscape
21.1 Toxicity of methylene blue is likely to be seen after single bolus dose (in mg/kg) greater than
a. 1mg/kg
b. 2mg/kg
c. 5mg/kg
d. 0.5mg/kg
e. 0.1mg/kg
c. 5mg/kg
Methylene blue due to its monoamine oxidase(MAO) inhibiting property may precipitate potentially fatal serotonin toxicity at doses >5mg/kg.
Source: STAT PEARLS - Methylene blue https://www.ncbi.nlm.nih.gov/books/NBK557593/
20.1 RFTS: Normal ratio, low FVC, low FEV1, Normal DLCO:
a) Sarcoid
b) Myasthenia Gravis
c) Asthma
d) Emphysema
b) Myasthenia Gravis
20.2 Of the following, the agent that causes the LEAST prolongation of the Thrombin Clotting Time (or Thrombin Time) is
a) Heparin
b) LMWH
c) Bivalirudin
d) Warfarin
e) Dabigatran
d) Warfarin
Warfarin – no effect on thrombin time
Heparin - causes considerable prolongation of TT.
LMWH, fondaparinux or direct factor Xa inhibitors have no effect on TT as the predominantly inhibit factor Xa.
-> However LMWH in very high concentration can affect TT.
Dabigatran, Bivalirudin and other direct thrombin inhibitors prolong TT considerably.
The thrombin time (TT), also known as the thrombin clotting time (TCT) is a blood test that measures the time it takes for a clot to form in the plasma of a blood sample containing anticoagulant, after an excess of thrombin has been added. Warfarin prevents thrombin synthesis but does not inhibit it, therefore no effect on TT.
23.1 Following the insertion of a peripherally inserted central catheter (PICC) into the cephalic vein in the upper arm, the patient complains of numbness in their forearm. It is likely that during insertion the operator has injured the
A. Median cutaneous antebrachial
B. Median antebrachial
C. Lateral antebrachial
D. Posterior brachial
E. Posterior cutaneous nerve (of the forearm)
c) lateral antebrachial
https://anatomytool.org/content/radiopaedia-drawing-contents-superficial-cubital-fossa-english-labels
double check - no reference
22.2 In a previously normal patient with cardiac failure secondary to acute pulmonary embolism, the best choice of vasoactive agent for initial treatment is
a. Dobutamine
b. Milrinone
c. Isoprenaline
d. Noradrenaline
d. Noradrenaline
Supportive Management of Massive PE
Coexisting left ventricular systolic dysfunction and diastolic dysfunction complicate the management of heart failure patients with massive PE. Although a common strategy in response to systemic arterial hypotension is to prescribe a fluid bolus, volume loading may worsen biventricular failure, pulmonary edema, and hypoxemia. An initial trial of volume expansion, limited to 250 to 500 mL, may be attempted in those heart failure patients without evidence of increased right-sided filling pressures or pulmonary edema.6
Although non–heart failure patients generally respond well to pure vasopressors for hemodynamic support in massive PE, many heart failure patients will not tolerate the isolated increase in systemic vascular resistance. PE patients with heart failure may require an agent with mixed vasopressor and inotropic properties such as norepinephrine, epinephrine, or dopamine. Whereas LV function often becomes hyperdynamic to compensate for RV failure, the presence of underlying LV systolic dysfunction in heart failure patients may limit the patient’s ability to maintain normal systemic cardiac output and may necessitate the addition of inotropes.
https://www.ahajournals.org/doi/10.1161/CIRCULATIONAHA.108.803965
21.2 A patient has return of spontaneous circulation (ROSC) but remains unresponsive after
cardiac arrest. ANZCOR Guidelines recommend all the following measures EXCEPT
a) Aiming SpO2 94%
b) Treating hyperglycaemia >10mmol/L
c) Normothermia
d) Cardiac catherisation
e) Amiodarone infusion
c) Normothermia
- TTM to 32-36 degrees is recommended for those that remain unresponsive
21.2 A 65 year old woman is dyspnoeic after a total hip replacement. A lung ultrasound is performed in the post-anaesthesia care unit, with a still image shown below. The likely cause of the dyspnoea is
a) Effusion
b) PE
c) Pneumothorax
d) Pneumonia
PTx
Commented from example image:
The stratosphere sign. Absent lung sliding on M-mode in a patient with a pneumothorax. Notice the absence of T lines (the lung pulse)
LITFL
The ultrasound appearance of pneumothorax
1. Loss of lung sliding and the movement artefact deep to the pleural line
- A pneumothorax lies deep to the smooth parietal pleural surface.
- The gas interface creates a highly reflective surface reflecting all ultrasound energy.
- This prevents imaging of structures lying below the pneumothorax. The movement of the lung, deep to the pneumothorax is completely hidden – lung sliding is lost.
- Loss of characteristic B-lines
- B-lines (vertical short path reverberation artefacts) are created by alveolar and interstitial fluid or fibrosis at the lung surface.
- In the same way that pneumothorax hides lung sliding it also hides any B-lines lying below. - Increased clarity of A-lines
- A-lines (horizontal long path reverberation artefacts) are echogenic horizontal artifactual lines deep to the pleural surface that are characteristic of pneumothorax.
- The mirror like, flat parietal pleura overlying the pneumothorax reflects the ultrasound which often then reverberates between the pleural surface and other horizontal reflecting surfaces above. These include fascial planes and the transducer surface itself.
- Multiple reflections cause horizontal linear artefacts mirroring the flat surfaces above the pleural surface, deep to the pleural surface. - Lung Point
- Pneumothorax separates the visceral and parietal pleural surfaces.
- The point at which these surfaces meet is known as the lung point
22.2 A 76-year-old man requires an emergency thoracotomy to treat an expanding haemothorax. He is mildly hypotensive and is not fasted. His plasma electrolytes and haemoglobin are below. The most appropriate strategy to employ to intubate him with a double lumen endotracheal tube is to (use)
K 6.3 Ur 7-ish Cr 174
a. Cisatracurium 0.5mg/kg
b. Rocuronium 1.2mg/kg
c. Suxamethonium 1mg/kg
d. Suxamethonium 0.5mg/kg (?was this an option)
b. Rocuronium 1.2mg/kg
Cis not appropriate for intubation
Sux with K 6.3 is risky. (I’ve never heard of reduced dose)
21.2 In a patient with anaemia of chronic disease, of the following the most likely to be elevated is
a. MCV
b. transferrin saturation
c. Increased soluble Transferrin Receptor
d. Ferritin
e. Total iron binding capacity
d. Ferritin
ANZCA blue book:
ACD caused primarily by inflammation
Mechanism:
1. Iron
- Inflammation reduces Iron availabilty as a protective mechanism whereby Iron is sequestered and stored in macrophages to limit availability to microbial pathogens
- Hepcidin expression is increased, this prevents the release of Iron by reticuloendothelial system resulting in “functional iron deficiency” with reduced tissue availability of iron, despite apparently normal total body iron stores. (hence increased Ferritin)
- Response to erythropoietin
- mechanism not clear suspect blunting of response to erythropoietin - Therapeutic agents
chemotherapies that impair bone marrow response to erythropoiesis
65% of patients with lung and gynae cancer treated with platinum based drug develop anaemia
RCPA advice on interpretation of Soluble Transferrin Receptor:
Soluble transferrin receptor levels in plasma are elevated if there is increased iron demand due to Iron deficiency, increased erythropoiesis (eg, Haemolysis) or dyserythropoiesis (eg, Megaloblastic anaemia), regardless of other, coexistent states.
Thus, it can be used to demonstrate iron deficiency in patients who also have an acute phase response and it can distinguish Iron deficiency from the Anaemia of chronic disease.
Patients with an acute phase response have reduced plasma iron and transferrin with elevation of Ferritin, making these usual indicators unreliable.
21.1 You have been asked to anaesthetise a patient with a history of severe depression which has been
well controlled on moclobemide. The most appropriate medications in combination with propofol are
a. Sevoflurane, morphine, phenylephrine
b. Sevoflurane, pethidine, phenylephrine
c. Midazolam, fentanyl, ephedrine
d. sevoflurane, oxycodone, ephedrine
a. Sevoflurane, morphine, phenylephrine
Moclobemide = MAOi
23.1 Desufflation after surgical pneumoperitoneum is NOT associated with an increase in
a) SVR
b) CI
c) EF
d) preload
e) LV work
a) SVR
When providing anaesthesia for endovascular treatment of acute ischaemic stroke, the Society of NeuroInterventional Surgery and the Neurocritical Care Society recommend
A. General anaesthesia
B. Hypervolaemia
C. Maintain temp <35
D. Maintain BGL 8-12
E. Maintain sBP 140-180
E. Maintain sBP 140-180
https://journals.lww.com/jnsa/Fulltext/2014/04000/Society_for_Neuroscience_in_Anesthesiology_and.1.aspx
We recommend that hemodynamic monitoring and management, as outlined below, should be started as soon as diagnosis of AIS has been made (class IIa, level of evidence C).
Heart rate and cardiac rhythm should be monitored continuously and blood pressure should be monitored continuously or measured at least once every 3 minutes.
We recommend that systolic blood pressure should be maintained >140mm Hg (fluids and vaso- pressors) and <180mm Hg (with or without IV tPA), and diastolic blood pressure <105 mm Hg (class IIa, level of evidence B).
Cause of hypotension should be investigated (volume depletion, myocardial infarction, cardiac arrhythmia, blood loss, retroperitoneal hemorrhage, and aortic dissection) and treated if possible.
We also recommend that blood pressure targets may be adjusted (lowered) in communication with the neuro- interventionalists and neurologists following successful recanalization of occluded vessel(s) (class IIb, level of evidence C), as reperfused brain often lacks autor- egulation leading to high risk of hyperperfusion leading potentially to hemorrhagic conversion.
20.1 Infrarenal aortic cross clamp will cause a(an):
a) Increased by 40% renal blood flow
b) Increased by 20% renal blood flow
c) Unchanged renal blood flow
d) Decreased 20% renal blood flow
e) Decreased 40%renal blood flow
e) Decreased 40%
Infra-renal aortic cross-clamping leads to a reduction in renal blood flow by up to 40%, as a result of an increase in renal vascular resistance of up to 75%.
The mechanism underlying this increased resistance is uncertain but may, in part, be a result of the associated decrease in cardiac output during aortic cross-clamping, as well as because of humoral mechanisms, which lead to increased release of renin. After declamping, there is a maldistribution of renal blood flow away from the cortex for at least 60 min.
21.2 A bleeding patient has ROTEM results including (results displayed) . The most appropriate treatment is
a) Cryoprecipitate
b) FFP
c) Platelets
d) TXA
e) Protamine
e) Protamine
The interpretation of this graph is not especially laborious. The cardinal abnormality is the massively prolonged CT and CF of the INTEM graph, which suggests that something has killed the intrinsic pathway of the clotting cascade. The CT returns to normal in the HEPTEM graph, which is essentially just an INTEM test with adde heparinase. The presence of heparinase seems to have reversed all of the coagulopathy - the CFT, alpha-angle and MCF have all returned to normal. Therefore, this patient has no coagulation problems other than the heparin.
https://derangedphysiology.com/main/required-reading/haematology-and-oncology/Chapter 1.2.0.1/intepretation-abnormal-rotem-data
22.1 Of the following, the drug most likely to cause pulmonary arterial vasodilation with systemic arterial vasoconstriction when used in low doses is
a) Adrenaline
b) Noradrenaline
c) Vasopressin
d) Dopamine
e) Dobutamine
c) Vasopressin
https://emcrit.org/ibcc/pressors/
- From UP TO DATE:
> At low doses of 1 to 3 mcg/kg per min, dopamine acts primarily on dopamine-1 receptors to dilate the renal and mesenteric artery beds
> At 3 to 10 mcg/kg per min (and perhaps also at lower doses), dopamine also stimulates beta-1 adrenergic receptors and increases cardiac output, predominantly by increasing stroke volume with variable effects on heart rate.
> At medium-to-high doses, dopamine also stimulates alpha-adrenergic receptors, although a small study suggested that renal arterial vasodilation and improvement in cardiac output may persist as the dopamine dose is titrated up to 10 mcg/kg per min
*clinically, the haemodynamic effects of dopamine demonstrate individual variability
Dobutamine (inodilator):
- selective β1-agonist that increases cardiac contractility and reduces pulmonary vascular and systemic vascular resistances
Vasopressin:
- vasopressin may have pulmonary vasodilatory effects in addition to a systemic vasoconstrictive effect
Milrinone (inodilator):
- the phosphodiesterase-3 inhibitors, milrinone and enxoimone, have positive inotropic effects combined with the capacity to reduce RV afterload (‘inodilators’) without significant chronotropic effect, but they can be associated with significant systemic hypotension
bonus neuro radiology questions
A 34 year-old woman developed the lightning onset of an explosive headache followed by nausea and vomiting.
She undergoes cerebral angiography.
Her cerebral aneurysm is in the
a. Anterior Cerebral Artery
b. Anterior communicating artery
c. MCA
d. PCA
e. Basilar artery
d. PCA
https://case.edu/med/neurology/NR/SubarachnoidHemorrhag3%20Pcom.htm
23.1 A nerve that does NOT provide sensory innervation to the shoulder joint is the
A. Axillary
B. Lateral pectoral
C. Subscapular
D. Supraclavicular
E. Suprascapular
d) Supraclavicular
Axillary nerve innervates skin to inferior deltoid (regimental badge)+ motor to terres minor and deltoid.
Lateral pectoral nerve innervates the anterosuperior part of the glenohumeral joint.
Subscapular nerves - upper subscapular nerve serves the upper portion of the subscapularis muscle; the middle subscapular nerve (thoracodorsal nerve) innervates latissiumus dorsi; lower subscapular nerve innervates subscapularis and terres major.
Supraclavicular nerve - sensory only and innervates skin across entire shoulder and trapezius in a ‘cape-like’ fashion - sometimes missed in interscalene block.
Suprascapular nerve sensory innervation to glenohumeral joint and acromiovlavicular joint + motor to supraspinatus/infraspinatous (rotator cuff)
https://pubmed.ncbi.nlm.nih.gov/32712453/
20.2 A patient with a purely metabolic acidosis has a serum bicarbonate of 14 mmol/L and a lactate of 3.8 mmol/L. The expected PaCO2 is
29mmHg
20.1 Patient with Fontan circulation and peritonism having induction for laparotomy. Drops sats on induction. Best move?
a. Decrease volatile
b. Reverse Trendelenberg
c. Decrease FiO2
d. Increase PEEP
e. Increase tidal volume
A 22-year-old man with a Fontan circulation is on your emergency list for an appendicectomy. He has had abdominal pain and vomiting for 3 days, and has a peritonitic abdomen. His preoperative arterial oxygen saturation is 95%. Shortly after induction he becomes hypotensive BP 80/45, and saturations fall to 75%. His condition is most likely to be improved by:
A. Increasing the inspiratory time.
B. Decreasing the ventilator tidal volumes.
C. Adding positive end-expiratory pressure (PEEP).
D. Positioning reverse trendelenberg.
A
Couldn’t find a clear source but we know;
A - will decrease venoplegia and improve venous return
B - Would not help, decrease VR
C - Don’t drop FiO2 when desatting…
D - increases PVR (unless below FRC) and reduces pulmonary flow
E - Same as above, increased PVR and reduces flow through pulmonary circuit
B. Decreasing the ventilator tidal volumes.
Patients who have undergone the Fontan procedure depend on blood flow through the pulmonary circulation without the assistance of the right ventricle. The difference between central venous pressure and systemic ventricular end-diastolic pressure (termed the “transpulmonary gradient”) is the primary force promoting pulmonary blood flow and, more importantly, cardiac output.
Circulation in the Fontan patient is promoted by low pulmonary vascular resistance. Positive-pressure ventilation with increased tidal volumes, as described above, can result in excessive intrathoracic pressures, leading to decreased venous return to the heart and increased pulmonary vascular resistance.
In periods of low oxygen saturation, 100% inspiratory oxygen is appropriate.
The addition of PEEP will increase intrathoracic pressure, reducing venous return.
Trendelenberg positioning would increase CVP and therefore bloodflow through pulmonary circulation.
22.2 Suxamethonium may be safely given to patients with (list of neuromuscular diseases given)
a. Becker muscular dystrophy
b. Myaesthenia gravis (new option)
c. Guillain Barre
d. Hypokalaemic periodic paralysis (new option)
e. Duchenne muscular dystrophy
b. Myaesthenia gravis
ED95 is 0.8mg/kg in a MG patient
20.1 The flow volume loop is most consistent with
a) Variable intra-thoracic obstruction
b) Variable extra-thoracic obstruction
c) Fixed large airway
d) Restrictive lung pattern
e) Mixed pattern
b) Variable extra-thoracic obstruction
21.1 A common electrolyte disturbance following the administration of ferric carboxymaltose is
a. hypophosphatemia
b. hypocalicaemia
c. hypokalaemia
d. hypercalicaemia
e. hypernatraemia
Hypophosphataemia
Ferric carboxymaltose (Ferinject) for iron-deficiency anaemia
https://www.nps.org.au/radar/articles/ferric-carboxymaltose-ferinject-for-iron-deficiency-anaemia
In this set of patients administered FCM (n = 5799), treatment-related side effects that occurred in more than 1% of the group included:
- nausea (3.1%)
- hypophosphataemia (1.9%)
- injection-site reactions (1.6%)
- headache (1.4%)
- hypertension (1.3%)
- dizziness (1.2%)
22.2 You will anaesthetise a 39-year-old woman for a laparoscopic cholecystectomy. She has a history of mastocytosis and has never had an anaesthetic in the past. A drug which you should avoid is
a. fentanyl
b. morphine
c. remifentanil
d. tramadol
B Morphine
Histamine-releasing
21.1 A four-year-old boy with a history of waddling gait, larger than normal calves and frequent falls receives a spontaneously breathing volatile-based anaesthetic with sevoflurane.
One hour into the case he develops peaked T waves and then the end-tidal CO2 begins to rise. The most appropriate immediate treatment is to
a. Temp probe, and go from there
b. Cool + dantrolene
c. Stop volatile, cool + dantrolene
d. Stop volatile, calcium
e. Stop volatile
d. Stop volatile, calcium
?Duchenne muscular dystrophy?
This patient most likely has Anaesthesia Induced Rhabdomyolysis (AIR) given the peaked Twaves and slow rise in ETCO2
Immediate MH Management:
Stop administering Sevo, flush machine (or new), charcoal filters. Dantrolene.
23.1 Expected features of Guillain-Barré syndrome include
A. Descending paralysis
B. Flaccid paralysis
C. Unilateral leg weakness
b) flaccid paralysis
Guillain–Barré syndrome (GBS) is an inflammatory disease of the PNS and is the most common cause of acute flaccid paralysis
22.1 When using cardioversion to revert a patient in atrial fibrillation to sinus rhythm, the direct current shock is synchronised with the ECG to coincide with the
a. Start of R wave
b. Start of Q wave
c. Middle of T wave
d. peak of R wave
R-wave peak
21.1 A 25-year-old man suffers a 30% total body surface area burn. A cardiovascular physiological change expected within the first 24 hours is
a. Decreased PVR
b. Increased SVR
c. Decreased SVR
d. Reduced PA pressure
e. Increased hepatic blood flow
increased SVR
EMSB handbook
CO is reduced after Burn injury 2ry to:
- myocardial depressant mediators
- decreased blood volume
- reduced venous return
- increased pulmonary and systemic vascular resistance due to increased levels of catecholamines
In the first 24hrs reduced cardiac output persists even after restoration of blood volume
Between 24-48hrs post burn a hyperdynamic state develops with reduced peripheral resistance, increased oxygen consumption and increased cardiac output
20.2 The part of the lung that is typically divided into superior and inferior segments is the
a) RUL
b) RML
c) RLL
d) LUL
e) Left lingula
e) Left lingula
LEFT LUNG: ASIA ALPS
Apical Posterior
Superior lingula
Inferior lingula
Apical Anterior
Anterior basal
Lateral basal
Posterior basal
Superior
RIGHT LUNG: A PALM Seed Makes Another little Palm
RUL:
Apical
Posterior
Anterior
RML:
Lateral
Medial
RLL
Superior
Medial basal
Anterior basal
Lateral basal
Posterior basal
20.2 The ANZCA guidelines regarding pre-operative oral intake for infants under 6 months of age having an elective procedure under anaesthesia are
a) Breast milk 2 hours before, clear fluids 1 hour before to max 3ml/kg
b) Breast milk 2 hours, clear fluids 1 hour before to max 5ml/kg
c) Breast milk 3 hours, clear fluids 1 hour to max 3ml/kg
d) Breast milk 3 hours, clear fluid 1 hour to max 5ml/kg
e) Breast milk 4 hours, clear fluids 1 hour to max 3ml/kg
Repeat
c) Breast milk 3 hours, clear fluids 1 hour to max 3ml/kg
Infants <6 months having elective procedure
* 4 hours for formula
* 3 hours for breast milk
* 1 hour for clear fluids (≤3 ml/kg/hr)
Children > 6 months having elective procedure
* 6 hours for limited solid food or formula
* 4 hours for breast milk
* 1 hour for clear fluids (≤ 3ml/kg/hr)
20.2 This lung ultrasound shows
a) Normal lungs
b) Pulmonary odema
c) Pneumothorax
d) Pleural effusion
e) Pneumonia
b) Pulmonary oedema
B-lines
> Vertical echogenic short path reverberation artefacts originating at the pleural line and extending to the deepest part of the ultrasound image.
They interrupt any horizontal A-lines.
Occasional B-lines are considered normal.
More than 3 B-lines in any single view is considered pathological.
Where there are numerous B-lines in close proximity they become confluent.
B-lines move with lung movement.
They are caused by ultrasound energy reverberating in a fluid filled focus that is surrounded by air. These foci may be interstitial or alveolar.
Cardiogenic and noncardiogenic oedema may have very similar appearances.
Interstitial thickening due to fibrosis or lymphangitis can also create the sonographic appearance of diffuse B-lines.
23.1 Pulse pressure variation is defined as
a. 100x SBP max - SBP min / SBP min
b. 100 x PPmax - PPmax / PPmin
c. 100x SBP max - SBP min/ SBP mean
d. 100 x PPmax - PPmin / PPmean
d) 100 x PPmax - PPmin / PPmean
20.1 What is the arrow pointing to?
a. Ilioinguinal
b. Iliohypogastric
c. Iliacus
d. Lateral Femoral Cutaneous
e. Obturator
d. Lateral Femoral Cutaneous
21.1 Perioperative overheating is most likely to cause worsening of symptoms of
a) Duchenne Muscular dystrophy
b) Myasthenia gravis
c) Multiple sclerosis
d) Myotonica dystrophia
e) Eaton Lambert syndrome
multiple sclerosis.
https://academic.oup.com/bjaed/article/11/4/119/266998
Anaesthetic considerations.
- Local anaesthetics may exacerbate symptoms due to the increased sensitivity of demyelinated axons to local anaesthetic toxicity.
- Non-depolarizing neuromuscular blocking agents may be used in normal doses.
Caution should be exercised when using depolarizing neuromuscular blocking agents if the patient is debilitated.
- Temperature maintenance is important as symptoms can deteriorate with an increase in temperature, as demyelinated axons are also more sensitive to heat.
22.2 The 2012 Berlin definition of the acute respiratory distress syndrome (ARDS) defines moderate disease as one with a PaO2 / FiO2 ratio (in mmHg) of
a) 50-100
b) 100-200
c) 200-300
d) 300-400
a) 100-200
2012 BERLIN DEFINITION OF ARDS
ARDS is an acute diffuse, inflammatory lung injury, leading to increased pulmonary vascular permeability, increased lung weight, and loss of aerated lung tissue…[with] hypoxemia and bilateral radiographic opacities, associated with increased venous admixture, increased physiological dead space and decreased lung compliance.
Key components
- acute, meaning onset over 1 week or less
- bilateral opacities consistent with pulmonary edema must be present and may be detected on CT or chest radiograph
- PF ratio <300mmHg with a minimum of 5 cmH20 PEEP (or CPAP)
- “must not be fully explained by cardiac failure or fluid overload,” in the physician’s best estimation using available information — an “objective assessment“ (e.g. echocardiogram) should be performed in most cases if there is no clear cause such as trauma or sepsis.
Severity
- ARDS is categorized as being mild, moderate, or severe:
20.2 When performing an infraclavicular block of the brachial plexus under ultrasound guidance, the structure indicated by the arrow is the (ultrasound image shown
a) Posterior Cord
b) Lateral cord
c) Median nerve
d) Superior trunk
e) Inferior trunk
a) Posterior Cord
20.2 This ECG rhythm strip represents
22.1 A patient presents for endoscopic retrograde cholangiopancreatography (ERCP) with a history of previous post-ERCP pancreatitis. The management most likely to reduce the likelihood of pancreatitis is
a. Gentamicin
b. PR indomethacin
c. Creon post op
d. Preop smoking cessation
Rectal indomethacin
APMSE 5th edition 8.6.1.3: Only rectal NSAIDs are effective for reducing post ERCP pancreatitis, particularly indomethacin. Epidural > PCA for severe acute pancreatitis
22.2 Of the following, all are useful for the treatment of status epilepticus EXCEPT
a. Calcium
b. isoflurane
c. ketamine
d. propofol
e. phenytoin
a. Calcium
(unless hyppocalcaemia is causing your seizures)
Deranged Physiology:
First line agents
- Benzodiazepines: boluses every 2-5 minutes
- Phenytoin: 20mg/kg loading dose
Phenytoin on its own is useless. Or rather, it is inferior to benzodiazepines as a solitary agent. Always, both must be used simultaneously.
Second line agents
- Midazolam infusion
- Phenytoin (well, rather, the American study recommends fosphenytoin)
- Phenobarbital and levetiracetam are also in this second line of attack
Third line agents: for refractory status epilepticus
- Propofol infusion, or midazolam infusion, or thiopentone infusion.
- At this stage, continuous EEG monitoring becomes mandatory
- The role of traditional antiepileptic drugs is also exhausted at this stage, as there will probably be no benefit from adding them into a situation where a constantly observed burst suppression is already achieved by high dose anaesthetic infusion.
Fourth line agents: for these, there is little evidence.
- Volatile anaesthetic agents
- Desflurane and Isoflurane
- Ketamine
- Lignocaine
- Magnesium
- Pyridoxine
Fifth line therapies:
- Hypothermia
- Ketogenic diet
- Deep brain stimulation
- Surgical management
20.1 A 22-year-old patient is scheduled for resection of a large extra-adrenal paraganglionoma. The tumour is secreting metanephrine. The most likely therapy to be commenced at the preassessment clinic prior to surgery is
a) Prazocin
b) Phentolamine
c) Magnesium
d) Phenoxybenzamine
e) Ca channel blocker
Phenoxybenzamine
UpToDate
Phenoxybenzamine is the preferred drug for preoperative preparation to control blood pressure and arrhythmia in most centers in the United States. It is an irreversible, long-acting, nonspecific alpha-adrenergic blocking agent.
With their more favorable side-effect profiles and lower financial cost, selective alpha-1-adrenergic blocking agents (eg, prazosin, t erazosin, or d oxazosin) are utilized in many centers or are preferred to phenoxybenzamine when long-term pharmacologic treatment is indicated (eg, for metastatic pheochromocytoma).
22.2 AA 15-year-old patient with a known prolonged QT interval has a ventricular tachyarrhythmia while being monitored postoperatively in the postanaesthesia care unit. The patient is alert, orientated and without chest pain but feels unwell. The best initial management is
A. Magnesium
B. Synchronised shock
C. Amiodarone
D. Adenosine
E. Metoprolol
A. Magnesium TdP
UTD - BB specifically propanolol or nadolol
if Hx of syncope / seizures or resus SCA
https://www.uptodate.com/contents/congenital-long-qt-syndrome-treatment
23.1 A technique which is NOT effective in providing analgesia for a sternal fracture is a
A. Pecs 1
B. Pecs 2
C. Thoracic transversus plane block
D. Subpectoral fascial plane block
A. Pecs 1
https://www.nysora.com/topics/regional-anesthesia-for-specific-surgical-procedures/thorax/pectoralis-serratus-plane-blocks/
21.2 The power board on the back of the anaesthesia machine has caught fire during an elective case. This should be extinguished with
a) CO2
b) Fire blanket
c) Wet chemical powder
d) Foam
e) Water
a) CO2
22.2 Non-anaesthetist practitioners wishing to provide procedural sedation should have training in sedation and/or anaesthesia for a minimum of
a) 6 weeks
b) 3 months
c) 6 months
d) 12 months
b) 3 months
ANZCA PS09 2014
NB: PG09 was updated in 2022 and no longer states a minimum timeframe, so this is unlikley to return as an MCQ
20.1 Which drug not metabolised by CYP2D6?
a) Oxycodone
b) Tramadol
c) Amitryptiline
d) Codeine
e) Hydromorphone
e) Hydromorphone
22.2 In Australia and New Zealand, the proportion of blood donors who are cytomegalovirus (CMV) seropositive is
(rough numbers in the options, can’t remember exactly)
a. 65 to 85 per million
b. 650 to 850 per million
c. 6.5 to 8.5 per hundred
d. 65 to 85 per hundred
d. 65 to 85 per hundred
85% of australians are CMV positive by the age of 40
https://www.blood.gov.au/system/files/documents/cmv-blood-components.pdf
22.1 The underlying trigger for the development of acute traumatic coagulopathy is
a. Acidosis
b. Hypothermia
c. Endothelial damage from ischaemia
d. Dilution of coagulation factors from resuscitation
e. Activation of fibrinolysis
Endothelial damage due to ischaemia
21.1 Major international guidelines recommend maintaining the core body temperature between 32°C and
36°C in comatose patients after
A. SAH
B. Stroke
C. Cardiac Arrest
Cardiac Arrest
Source: LITFL
23.1 For a woman who has a history of preeclampsia in a previous pregnancy, the intervention with the best evidence for prevention of preeclampsia during future pregnancies is
A. Aspirin 150mg daily (option was definitely 150mg not 100mg)
B. Mg
C. Heparin subcut
D. Ca
A. Aspirin 150mg daily (option was definitely 150mg not 100mg)
or
D. Ca
Aspirin should be given at a dose between 75 and 150 mg per day, started preferably before 16 weeks, possibly taken at night, and continued until delivery;
https://www.somanz.org/content/uploads/2023/06/SOMANZ_Hypertension_in_Pregnancy_Guideline_2023.pdf
Calcium supplementation (1.5g/day) should therefore be offered to women with moderate to high risk of preeclampsia, particularly those with a low dietary calcium intake (247)
22.1 A two-year-old boy with a history of respiratory tract infection one week previously has just undergone squint surgery. His airway was managed with a size 4.5 mm cuffed endotracheal tube.
The surgery was unremarkable. Twenty minutes after extubation he is awake and appears anxious, with stridor and a visible tracheal tug. His oxygen saturation is 96% on room air. The best initial management of this child is to administer
a) Dexamethasone 0.6mg/kg
b) Adrenaline nebulised 1:1000 - 0.5mL/kg
c) CPAP + T piece
d) Drugs for re-intubation
Nebulised Adrenaline
1mg
0.5ml/kg of 1:1000 Adrenaline nebulised
once adrenaline given consider dose of Steroid dexamethasone or hydrocortisone
22.1 Abnormal Q waves are NOT a feature of the electrocardiogram in
A. Digitalis toxicity
B. LBBB
C. Recent transmural MI
D. Wolff-Parkinson-White
E. Previous MI
A. Digitalis toxicity
Miller’s
The ECG made easy
http://lifeinthefastlane.com/ecg-library/pmi/
Normal Q waves
- Due to depolarisation of the interventricular septum from left to right
- Seen in the left-sided leads (I, aVL, V5, V6)
Pathological Q waves
- > 1 mm depth
- > 1 mm (= 40 ms) across
Digoxin ECG changes
- Therapeutic: prolonged PR interval (AV nodal delay), shortened QTC intervals (rapid ventricular repolarisation), ST depression (↓ slope of phase 3), T wave inversion
- Toxic: atrial or ventricular arrhythmias (↑ automaticity), prolonged PR interval → heart block, SA node inhibition → sinus arrest
- Atrial tachycardia with block = most common arrhythmia attributed to digoxin toxicity
- VF = most frequent cause of death
- QRS = normal!
Q waves in MI
- Occur with transmural infarctions, and are less likely with subendocardial infarctions
- Develops days after the onset of AMI, and is usually permanent
- Indicates the part of the heart that has been damaged
LBBB ECG changes
- Wide QRS
- Wide QS complex in lead V1
- Wide R wave in lead V6 with slight notching at the peak and TWI
- The axis is highly variable: can be normal or deviated to the left or right
Wolff-Parkinson-White syndrome
- Due to the presence of an accessory bundle between the atrium and ventricle, which has no AV node to delay conduction
- Short PR interval
- Early slurred upstroke of the QRS complex due to delta wave
- Reciprocal leads will have Q wave like appearance
23.1 The parameter that changes most with increasing age in the otherwise normal lung is the
a. Closing capacity
b. Residual volume
c. FRC
d. Lung capacity.
a) Closing capacity
see graph in Millers
21.2 The equipment shown in the picture below is a
a) Parker flex tip
b) Hunsaker
c) Laryngectomy tube
d) NIM tube
e) Reinforced tube
NIM tube: Neural Integrity Monitor Electromyogram Tracheal Tube
22.2 When using an endotracheal tube in an adult, the highest recommended cuff pressure to avoid mucosal ischaemia is
a. 10cmH2O
b. 20
c. 30
d. 40
e. 50
c. 30cmH2O
paeds 20cmH20
22.1 A patient with a haemopneumothorax has a chest drain in situ, which is attached to a three-bottle underwater seal drain apparatus. The system is attached to wall suction at -80 cmH20. This will cause
a) Failure of underwater seal
b) Water in suction chamber will enter drainage chamber
c) Reexpansion of haemopneumothorax
d) Oscillation in tube will diminish
e) Inability for stuff to drain into first bottle
Oscillations in the tube will be diminished
20.1 According to the Australian and New Zealand Resuscitation Guidelines the immediate treatment for an adult conscious victim with a severe airway obstruction due to a foreign body inhalation is
a) a single back blow
b) two back blow
c) up to 5 blows to back, then up to 5 chest thrusts
d) up to 5 chest thrusts
e) sweep mouth
c) 5 back, 5 chest
4.1 Assess Severity
The simplest way to assess severity of a FBAO is to assess for effective cough.
4.2 Effective Cough (Mild Airway Obstruction)
The person with an effective cough should be given reassurance and encouragement to keep coughing to expel the foreign material. If the obstruction is not relieved the rescuer should call an ambulance.
4.3 Ineffective Cough (Severe Airway Obstruction) Conscious person
If the person is conscious send for an ambulance and perform up to five sharp, back blows with the heel of one hand in the middle of the back between the shoulder blades. Check to see if each back blow has relieved the airway obstruction. The aim is to relieve the obstruction with each blow rather than to give all five blows. An infant may be placed in a head downwards position prior to delivering back blows, i.e. across the rescuer’s lap [Class B; LOE IV].1,2
If back blows are unsuccessful the rescuer should perform up to five chest thrusts. To perform chest thrusts, identify the same compression point as for CPR and give up to five chest thrusts. These are similar to chest compressions but sharper and delivered at a slower rate. The infant should be placed in a head downwards and on their back across the rescuer’s thigh, while children and adults may be treated in the sitting or standing position [Class B; LOE IV].1,2
With each chest thrust, check to see whether the airway obstruction has been relieved. The aim is to relieve the obstruction rather than deliver all five chest thrusts. If the obstruction is still not relieved and the person remains responsive, continue alternating five back blows with five chest thrusts.
Unconscious person
If the person becomes unresponsive a finger sweep can be used if solid material is visible in the airway.1,2 [Class A; LOE IV] Call an ambulance and start CPR.
22.2 Which of the following risk factors for preeclampsia in isolation would be sufficient to recommend commencing low-dose aspirin?
a. Age >40
b. >10 years since last pregnancy
c. Family hx of pre eclampsia
d. autoimmune disease
e. BMI >35
d. autoimmune disease (with potential vascular complications)
RANZCOG
Maternal characteristics that are associated with an increased likelihood of pre-eclampsia include:
- previous pre-eclampsia, particularly when more serious or early onset before 34 weeks
- pre-existing medical conditions (including chronic hypertension, underlying renal disease, or pre-gestational diabetes mellitus),
- underlying antiphospholipid antibody syndrome,
- multiple pregnancy
UTD: Preeclampsia: Prevention
https://www.uptodate.com/contents/preeclampsia-prevention
Based on the available data (see ‘Evidence of efficacy’ above), we recommend low-dose aspirin prophylaxis for women at high risk for preeclampsia. There is no consensus on the exact criteria that confer high risk. It is reasonable to use the US Preventive Services Task Force (USPSTF) high-risk criteria, which are also endorsed by the American College of Obstetricians and Gynecologists (ACOG).
The incidence of preeclampsia is estimated to be at least 8 percent for pregnant women with any one of these high risk factors:
●Previous pregnancy with preeclampsia, especially early onset and with an adverse outcome
●Multifetal gestation
●Chronic hypertension
●Type 1 or 2 diabetes mellitus
●Chronic kidney disease
●Autoimmune disease with potential vascular complications (antiphospholipid syndrome, systemic lupus erythematosus)
23.1 This Doppler trace obtained by transoesophageal echocardiography of the descending aorta suggests
a. AS
b. AR
b. AR
https://litfl.com/oesophageal-doppler/
20.1 What is the arrow pointing to?
a. Psoas
b. Femoral
c. Genitofemoral
d. Lateral cutaneous nerve of thigh
e. Lumbosacral trunk
b. Femoral
22.2 A patient has return of spontaneous circulation (ROSC) but remains unresponsive after cardiac arrest. ANZCOR Guidelines recommend all the following measures EXCEPT
a) Titrating FiO2 for SpO2 94-98%
b) Treating hyperglycaemia >10mmol/L
c) Targeted temp management at 32-36 degrees
d) Maintaining MAP >70
d) Maintaining MAP >70
Recommends maintaining equal or greater than pts usual, or at least a SBP> 100mmHg
https://www.resus.org.nz/assets/Uploads/ANZCOR-Guideline-11.7-Jan16.pdf
23.1 A multitrauma patient is being managed with a resuscitative endovascular balloon occlusion device of the aorta (REBOA) as part of damage control resuscitation. The
balloon has been inserted for intractable pelvic bleeding. The most appropriate location for the device placement is between the
A. Between artery of adamkiewicz to coeliac artery
B. Between coeliac artery to renal artery
C. Between lowest renal artery to bifurcation of aorta
D. Between coeliac and bifurcation
C. Between lowest renal artery to bifurcation of aorta
https://litfl.com/reboa-in-resuscitation/
Anatomy:
The aorta is divided into three separate zones for the purposes of REBOA (aortic length varies between individuals)
Zone I of the aorta extends from the origin of the left subclavian artery to the coeliac artery (approx 20cm long in a young adult male)
Zone II extends from the coeliac artery to the most caudal renal artery (approx 3cm long)
Zone III extends distally from the most caudal renal artery to the aortic bifurcation (approx 10cm long)
REBOA location based on injury:
> suspected or diagnosed intra-abdominal haemorrhage due to blunt trauma or penetrating torso injuries (Zone I REBOA), or
> blunt trauma patients with suspected pelvic fracture and isolated pelvic haemorrhage (Zone III REBOA), or
> patients with penetrating injury to the pelvic or groin area with uncontrolled haemorrhage from a junctional vascular injury (iliac or common femoral vessels) (Zone III REBOA)
Simplistic rendering of aorta. Zone 1 (from left subclavian artery to the upper border of the celiac trunk), Zone 2 (the upper border of the celiac trunk to the lower border of the distal take-off of the renal arteries), and Zone 3 (from the lower border of the lower renal artery to the aortic bifurcation). Zone 1 is occluded in the case of cardiac arrest or life-threatening intra-abdominal hemorrhage; Zone 2 has no current indication; and Zone 3 is occluded in the case of life-threatening pelvic or lower limb haemorrhage7. REBOA Resuscitative Endovascular Balloon Occlusion of the Aorta.
20.1 The radial artery pressure trace shown below is from a patient who has an intra-aortic balloon pump in situ. The device has been switched to 1:2 augmentation to assess the timing. The trace shows an augmented beat followed by an un-augmented beat. With respect to the augmentation, the trace shows
a. Correct timing
b. Early inflation
c. Late inflation
d. Early deflation
e. Late deflation
Bonus question
e. Late deflation
https://litfl.com/intra-aortic-balloon-pump-trouble-shooting/
Waveform features:
> The peak corresponding to diastolic augmentation (peak C) is widened.
> Assisted aortic end-diastolic BP (trough E) is the same as, not lower than, the unassisted aortic end-diastolic BP (trough G).
> The upstroke of assisted systolic BP (peak F) has a gentle gradient resulting in a prolonged rise.
Late deflation of the IAB has these effects:
> There is no afterload reduction. The inflated balloon may actually impede left ventricular ejection and increase the afterload.
> Myocardial oxygen consumption will increase because the left ventricle experiences a longer period of isovolumetric contraction (when most myocardial oxygen consumption occurs) and has to contract against greater resistance (afterload).
How to correct:
> Shorten the IAB inflation time, so that the IAB deflates at the end of diastole – just before isovolumetric contraction of the left ventricle.